You are on page 1of 109

For Free Study Material & Quizzes visit : www.mathsbyarunsir.

com
For any queries or suggestions email us @ mathsbyarunsir003@gmail.com or whatsapp @ 8881331466
For Free Study Material & Quizzes visit : www.mathsbyarunsir.com
For any queries or suggestions email us @ mathsbyarunsir003@gmail.com or whatsapp @ 8881331466
For Free Study Material & Quizzes visit : www.mathsbyarunsir.com
For any queries or suggestions email us @ mathsbyarunsir003@gmail.com or whatsapp @ 8881331466
For Free Study Material & Quizzes visit : www.mathsbyarunsir.com
For any queries or suggestions email us @ mathsbyarunsir003@gmail.com or whatsapp @ 8881331466
For Free Study Material & Quizzes visit : www.mathsbyarunsir.com
For any queries or suggestions email us @ mathsbyarunsir003@gmail.com or whatsapp @ 8881331466
1. x% of 800+348=980 7 5 3
17. 4 12 + 6 6 – 8 4 =? +1 3
2
(a) 81 (b) 76 (c) 83
(a) 1 (b) 2 (c) 3
(d) 79 (e) 73
1 𝑥
(d) 4 (e) 5
2. 𝑜𝑓 343 − 25% 𝑜𝑓 100 = 8 𝑜𝑓 24 18. 56% of 4750 + 475% of 12 – (?)² = 217
7
(a) 24 (b) 8 (c) 1 (a) 40 (b) 60 (c) 50
(d) 16 (e) 32 (d) 45 (e) 55
3. 9870 + 8790 + 6552 − 𝑥 = 9552 19. (87.98 + 56.64 + 123.58) – (47.6 + 28.6) – ? = (12)²
(a) 15660 (b) 15860 (c) 15460 (a) 42 (b) 40 (c) 45
(d) 15640 (e) 15606 (d) 38 (e) 48
4. 44 × 50% 𝑜𝑓 70 − 289 ÷ 17 × 2 = 𝑥 20. 19.65×14
3
+ 108.87 – 24.57 – ? = (13)2
(a) 1504 (b) 1508 (c) 1506 (a) 3 (b) 7 (c) 5
(d) 1510 (e) 1502 (d) 9 (e) 11
5. 18 + 512 − 128 + 𝑥 2 = 802 21. 37% 𝑜𝑓 ? + (11)² – 28.5 × 6 + 12 = (31)²
(a) 22 (b) 20 (c) 16 (a) 2650 (b) 2600 (c) 2750
(d) 24 (e) 18 (d) 2700 (e) 2760
4
22. (26% 𝑜𝑓 3800 + 25% 𝑜𝑓 620) ÷ 33 + 11 =?
6. √92 + 112 − 6 = 𝑥 × 56 ÷ 8
(a) 8 (b) 74 (c) 14 (a) 35 (b) 45 (c) 25
(d) 55 (e) 65
(d) 4 (e) 2 2.6 1
23. (27) × (81)0.7 + (32)0.8 = ? % 𝑜𝑓 1036
7. 80% 𝑜𝑓 𝑥 − 50% 𝑜𝑓 400 = 600
(a) 15 (b) 35 (c) 25
(a) 960 (b) 880 (c) 800
(d) 45 (e) 40
(d) 1000 (e) 500
(𝑥 + 4) = 8𝑥 2 + 97
2 2 24. 27.8 × 6 + 38.9 × 7 – ? = (13)² + √400
8.
(a) 240.1 (b) 250.1 (c) 230.1
(a) 3 (b) 9 (c) 1
(d) 235.1 (e) 260.1
(d) 27 (e) 18 4 3
2 4 1 𝑥 1 25. 9 × 486 + 35% 𝑜𝑓 120 + 8 × 208 = ?
9. − 3 + 18 = + 15
7 7 2 7 2 a) 412 b) 260 c) 398
(a) 7 (b) 5 (c) 57
d) 336 e) 284
(d) 15 (e) 6
26. (7800 ÷ 312) + 65% 𝑜𝑓 1120 = 9 × ? + 24
10. √961 + √1369 − √1444 = √361 + √? a) 81 b) 39 c) 27
(a) 196 (b) 169 (c) 144 d) 36 e) 24
(d) 121 (e) 81
27. ? × 15 + (3960 ÷ 12) = 262 + (350 ÷ 25)
11. 52500 ÷ 7 + 64680 ÷ 6 = 19500 − ? a) 56 b) 14 c) 24
(a) 1220 (b) 1260 (c) 1280 d) 36 e) 40
(d) 1340 (e) 1390 28. 45% 𝑜𝑓 1200 – 32% 𝑜𝑓 1500 = 15% 𝑜𝑓 ?
12. 28.5 × 34 + 2320 ÷ 8 = (36)2 − ? a) 600 b) 250 c) 500
(a) 51 (b) 47 (c) 43 d) 480 e) 400
(d) 34 (e) 37 29. (322 – 222) ÷ 32 = ?
13. 47 × 27 + 15600 ÷ 8 + 181 = ? a) 50 b) 60 c) 25
(a) 3320 (b) 3420 (c) 3370 d) 30 e) 15
(d) 3400 (e) 3460 30.
1 1 1
5 4 + 7 3 + 4 2 = 3 6 +? + 5 6
1 1

14. 112.5 × 5 + 4560 ÷ 6 − 175 × 7 = ? 1 1


(a) 8 2 (b) 8 4 (c) 8
(a) 103.5 (b) 91.5 (c) 97.5
3
(d) 110.5 (e) 115.5 (d) 8 4 (e) 9
3 3 5 7 1
15. ( √4096 + √5832) ÷ √289 = (? )0.2 31. of of 28 of 1600 = 260 + ? −499
2 8
(a) 64 (b) 16 (c) 27 a) 264 b) 480 c) 364
(d) 2 (e) 32 d) 342 e) 420
3 5 32
16. 𝑜𝑓 27 𝑜𝑓 45 𝑜𝑓 2187 = ? % of 400 32. √52 × 41 × 5 − 172 − 75 = ?
8
(a) 21 (b) 23 (c) 25 a) 69 b) 71 c) 79
(d) 27 (e) 29 d) 63 e) 89

For Free Study Material & Quizzes visit : www.mathsbyarunsir.com


For any queries or suggestions email us @ mathsbyarunsir003@gmail.com or whatsapp @ 8881331466
187 49. 252 + 520 ÷ 20 + 420 = 121 + ?
33. × (42)2 − 220% of 380 = 25% of ?
357 a) 587 b) 577 c) 527
a) 330 b) 358 c) 342
d) 477 e) 627
d) 352 e) 362 2
28 9 11 50. 35% of 5000 – 32 + 3 = ?
34. 252 × 22 × 3 ×? = (10)2 a) 825 b) 569 c) 729
a) 400 b) 500 c) 600 d) 689 e) None of these
d) 650 e) 550 51. 20% of 5000 – 25% of 3000 = ? × 52
35. 55% of 1400 + ? + 282 = (12)3
2 a) 10 b) 15 c) 8
a) 28 b) 24 c) 26 d) 16 e) None of these
d) 36 e) 16\ 52. 16 × 15 – 240 – 32 × 10 + 420 = ?2
a) 10 b) 15 c) 5
36. 42%of 500 + 22.5% of ? = 6 × √5041
d) 12 e) None of these
a) 840 b) 960 c) 800
53. 65% of 200 – 26% of 500 = ?
d) 600 e) 400
a) 5 b) 8 c) 4
37. ? + (13)3 = (59)2 − 681 d) 10 e) None of these
a) 603 b) 593 c) 613
54. 52 × 18 – √2304= (30)? - 5% of 240
d) 623 e) 583
a) 3 b) 2 c) 4
7.5% 𝑜𝑓 7200
38. + 450 = 15% of 3200 d) 0.5 e) 1
?
a) 15 b) 24 c) 20 55. 1669 – 5287 + 7982 – 2219 = ?
d) 18 e) 12 a) 2344 b) 2282 c) 2370
3 1 3
39. ? + 15 4 − 3 3 × 3 4 = 5] d) 2145 e) 2205
4860
a) 1.5 b) 2.25 c) 1.75 56. 62 + 5145 ÷ (7)3 − 47 × 3 = ?2
d) 2 e) 2.75 a) 9 b) 7 c) 3
40. 45% 𝑜𝑓 400 + ?2 = (16)2 + 120 d) 6 e) 2
52×36 952
(a) 12 (b) 14 (c) 16 57. 6 × ( )− + 59 × 2 = ?
17 68
(d) 18 (e) 8 a) 200 b) 170 c) 140
41. 12% of ? + 12.5% of 960 = 16 × 12 d) 120 e) 150
(a) 840 (b) 960 (c) 800 58. √1156 + √2209 = ? × √81
(d) 600 (e) 400 a) 9 b) 12 c) 7
42. 15 ×? + 20%of 450 = 360 d) 5 e) 11
(a) 12 (b) 20 (c) 24 59. 64%of 2500 + 75% of 1600 = ? × 112
(d) 16 (e) 18 a) 28 b) 21 c) 24
648
43. ? + 199.5 = 25%of 960 d) 32 e) 25
(a) 16 (b) 8 (c) 32
60. √40% 𝑜𝑓 900 − 421 + 350 + 272 = (? )2
(d) 18 (e) 12
1 5 a) 17 b) 19 c) 18
44. ? + 13 6 + 6 6 = 24
d) 16 e) 15
(a) 1 (b) 8 (c) 4
61. 43 × 322 ÷ 83 = (2)?
(d) 6 (e) 2
a) 9 b) 5 c) 7
45. 48% of 525 + ? % of 250 = 499
d) 6 e) 8
a) 88.8 b) 76.6 c) 82.6
62. 4 of 143 ÷ 26 × 12 – 200 = ?
d) 98.8 (e) 92.8
5 7 1 a) 58 b) 64 c) 72
46. of of of 1600 + 260+? – 499 d) 80 e) 74
2 8 28
a) 264 b) 480 c) 364 1 1
63. 5 2 − 4 3 = ? – 4
d) 342 (e)420 1 7 5
a) 54 b) 412 c) 46
47. √52 × 41 × 5 − 172 − 75 = ? 3 1
a) 69 b) 71 c) 79 d) 54 e) 56
d) 63 89 64. 49% of 180 – 70% of 120 = 9 –?
48. √256 × 49 + (19)2 + 11 = (? )2 a) 4.2 b) 5.2 c) 4.9
a) 484 b) 22 c) 24 d) 4.6 e) 4.8
d) 42 e) 26
For Free Study Material & Quizzes visit : www.mathsbyarunsir.com
For any queries or suggestions email us @ mathsbyarunsir003@gmail.com or whatsapp @ 8881331466
3 143
65. √?× √1225 + 240 = 520 81. 48%of 4800 + 169 = ?

a) 343 b) 512 c) 729 a) 2235 b) 2315 c) 2375
d) 1728 e) 216 d) 2300 e) 2345
66. 311 + (? )2 = 8%of 12800 + 16 82.
3
√1331 × 25%of 300 = ?/2
a) 24 b) 27 c) 25 a) 1600 b) 1550 c) 1650
d) 23 e) 28 d) 1425 e) 1775
67. 444×1.5 + 480 × 0.75 =? 3 1 1 1
a) 1028 b) 1026 c) 1034 83. 8 4 + 4 4 − 3 2 = ? + 7 5
3
d) 1036 e) 1032 a) 210 b) 2 c) 3
68. 42.55 – 21.75 – 24.11+15.55 =? 7 3
d) 2 10 e) 310
a) 14.65 b) 12.15 c) 13.55
3
d) 12.24 e) 12.35 84. 242 + 144 × 18 − 20%of ? = 152
69. 462 ÷ 5.25 + 24×12 =? a) 1925 b) 1650 c) 1850
a) 372 b) 390 c) 376 d) 1875 e) 2025
d) 370 e) 400 85. 3735 + 4565 + 6245 = ? + 6432
640
70. = (15)2 − 225 × 12) ÷ 33.75 (a) 8126 (b) 8113 (c) 6393
?
a) 18 b) 40 c) 45 (d) 7865(e) 8191
? 6 21
d) 32 e) 26 86. 𝑜𝑓 7 𝑜𝑓 36 𝑜𝑓 216 = 108 – 72 ÷ 2 – 36
3
71. ?2 + 224 − 96 = 85%of 280 + 34 (a) Multiple of 3 (b) Multiple of 2 (c) Multiple of 5
a) 18 b) 12 c) 16 (d) Both A & B (e) None of the Above
d) 14 e) 8 1
87. ? +√841 + –3 = 122 + 3 × 37
(3)
72. √1521 + (21)2 − 18 × 5 = 5 ×? (a) 113 < ?< 123 (b) 120 < ?> 122 (c) 133 > ? < 320
a) 80 b) 78 c) 64 (d) 143 < ?< 300 (e) Both B & D
d) 70 e) 96 88. 324 ÷ 12 + 29 × 3 – 48 + 39 = ?
73. 48% of 800 + 125% of 1200 – 120 = (? )2 (a) Multiple of 3 (b) Multiple of 2 (c) Multiple of 5
a) 42 b) 58 c) 36 (d) Both A & C (e) None of the above
d) 40 e) 50 89. 44% of 250 + 30% of 270 + 13² = ?
74. ?% of 700+18=751+332+83 (a) Multiple of 3 (b) Multiple of 2 (c) Multiple of 5
a) 172 b) 164 c) 128 (d) Both A & B (e) All of the Above
d) 142 e) 198
75. 15% of 256 + 35% of 280 = ?
90. (2.5)2 + 55% of 25 = ?2 − 5
a) 133.4 b) 136.4 c) 140.4
(a) 3 < ?< 2 (b) 1 < ?> 18 (c) 23 > ? < 3
d) 138.4 e) 135.4
3
(d) 3 < ?< 21 (e) None of these
76. √4096 + √12167 + 34 = (? )2 91. 121.75 + ? = 22 (63 − 91)
a) 11 b) 12 c) 13
(a) 376.25 (b) 378.25 (c) 384.25
d) 9 e) 8
4 3 19
(d) 386.25 (e) 390.86
77. 8 7 + 9 4 − ? = 6 28
92. 112% of 175 + ?3 × 7.5 = 162
9 8 17
a) 1114 b) 11 14 c) 1128 (a) Multiple of 3 (b) Multiple of 2 (c) Multiple of 5
3 11 (d) Both A & B (e) None of the above
d) 11 e) 11 1 1 1
7 14
93. 3 4 + ? = 7 8 + 2 12
78. 1.11 + 2.111 + 3.1 + ? = 8 1 23 1
a) 1.599 b) 1.669 c) 1.689 a) 4 24 b) 5 24 c) 8 24
d) 1.779 e) 1.679 1 1
d) 10 24 e) 2 24
2 11
79. 3 5 × 1 34 ×? = 108
94. 12.5 × ? + 123 = 40% of 4445
a) 18 b) 34 c) 12 (a) Multiple of 3 (b) Multiple of 2 (c) Multiple of 5
d) 36 e) 30 (d) Both A & B (e) Multiple of 7
1
3
80. 72.9 ÷ 81 × √100 =?2 ÷ √324 + √64 95. ? % of 144 + 16 × 12 = 182 − √576
a) 8100 b) 6400 c) 7200 (a) Multiple of 3 (b) Multiple of 2 (c) Multiple of 5
d) 1290 e) 3600 (d) Both A & C (e) None of these

For Free Study Material & Quizzes visit : www.mathsbyarunsir.com


For any queries or suggestions email us @ mathsbyarunsir003@gmail.com or whatsapp @ 8881331466
3 4
96. 156 + 376 − ? = 13% of 3000 d) 7 5 e) 8 5
(a) Multiple of 3 (b) Multiple of 2 (c) Multiple of 5 112. (1428 + 600 ÷ 2) ÷ (720 ÷ 5) =?
(d) Both A & B (e) None of the Above (a) Multiple of 3 (b) Multiple of 2 (c) Multiple of 4
97. 115% of 4880 – 85% of 1640 = 740% of ? (d) Both A & B (e) All of the Above
(a) Multiple of 3 (b) Multiple of 2 (c) Multiple of 5 1
113. ? % of 460 + 34% of 750 = of 2492
(d) Both B & C (e) All of the Above 4
3 (a) Multiple of 16 (b) Multiple of 13 (c) Multiple of 5
98. √9261 × √1681 = ? + 89 (d) Both A & C (e) All of the Above
(a) 772 (b) 166 (c) 663 2 2 5
(d) 652 (e) 762 114. 35 of 4 17 of 5 7 of 450 =?
1 2 1 3 (a) Multiple of 1000 (b) Multiple of 1200 (c) Multiple of 6000
99. 8 3 × 2 5 +? = 14 6 × 3 17
(d) Both A & C (e) All of the Above
(a) Multiple of 3 (b) Multiple of 2 (c) Multiple of 5 3 4 ?
(d) Both A & B (e) Multiple of 7 115. 5of7 of 3430 = 5 of 125
(a) 47.04 (b) 4.704 (c) 47.94
100. 45% of 80 + √841 + x² = 2121 ÷ 21
(d) 45.04 (e) 43.04
(a) Multiple of 3 (b) Multiple of 2 (c) Multiple of 5
116. 13% of 450 + 21% of 250 =? + 24% of 80
(d) Both A & B (e) Multiple of 7
36+3𝑥 (a) 9.18 (b) 91.8 (c) 93.8
101. + 28 ÷ 162 = 13 × 4 (d) 95.6 (e) 99.4
23
(a) 370 (b) 350 (c) 379 28 9 11
117. 252 × 22 × 3 ×? = (10)2
(d) 355 (e) 367
(a) Multiple of 10 (b) Multiple of 20 (c) Multiple of 30
102. 73 × 25 ÷ 43 + 175% 𝑜𝑓 350 =x2 (d) Both A & C (e) All of the Above
(a) Multiple of 7 (b) Multiple of 4 (c) Multiple of 2 118. 33% of 700 + 53% of 800 = ? % of 400
(d) Both A & B (e) All of the Above (a) 192.75 (b) 175.55 (c) 161.75
103. 23 × 24 + 23 × 47 – 23 × 54 = 𝑥 (d) 163.75 (e) 167.35
(a) 237 (b) 289 (c) 321 3 4
119. √729 + √784 + √256 = ? + √343
3

(d) 391 (e) 491 (a) 30 < x < 35 (b) 18 > x > 42 (c) 28 < x > 54
104. 120% of 650 + 320 + 255 ÷ 5 = 𝑥 (d) 22 < x > 70 (e) 24 > x < 39
(a) 1163 (b) 1363 (c) 1151 120. ? % of 125 + (15)2 = (16)2 – 2.5% of 640
(d) 1263 (e) 1051 a) 6 b) 8 c) 10
2 2 2 1
105. 11 9 + 12 9 – 13 9 – 4 4 = 𝑥 d) 12 e) 4
35 35 35 121. (12)3 + 24% of ? = 1830
a) 5 36 b) 7 c) 9 36 a) 400 b) 415 c) 425
36
31 35 d) 405 e) 350
d) 8 36 e) 3 36
122. 7.8 + 50% of 64.4 = (7)2 - ?2
106. 5220 + 1375 – 5364 + 𝑥 = 10288 a) 3 b) 2 c) 1
(a) 9263 (b) 9057 (c) 8024 d) 6 e) 7
(d) 7056 (e) 8824 123. ? = 6/5 of 8/7 of 420
107. ? × (1350 ÷ 112.5) = √5929 + √8281 a) 566 b) 576 c) 536
(a) Multiple of 3 (b) Multiple of 8 (c) Multiple of 5 d) 520 e) None of these
(d) Both A & B (e) Multiple of 7 124. (1350 ÷ 25) 𝑜𝑓 50% 𝑜𝑓 1700 = ? % 𝑜𝑓 2700
108. 18750 ÷ √? = 36 × 11 + 59 × 6 (a) 1500 (b) 1100 (c) 2100
(a) 252 (b) 52 (c) 42 (d) 1700 (e) 1900
(d) 32 (e) 62 125. 1330 + 26% 𝑜𝑓 1800 − 1190 = ? × 50 −
?
109. 3 = 729 ÷ 243 × 216 × 72 ÷ 576 1742
(a) 22 (b) 52 (c) 42 (a) 65 (b) 72 (c) 41
(d) 32
(e) 62
(d) 56 (e) 47
3 8
110. 456 ÷ 4 × 8 + 24 – (16× 5 of 15) =? 126. 1900 − (2600 × 13 + 1740) + ? = 80% 𝑜𝑓 700
(a) 992 (b) 892 (c) 692 (a) 2000 (b) 1100 (c) 1500
(d) 892 (e) 792 (d) 2300 (e) 1400
2 3 1 1 127. ? = 104% 𝑜𝑓 150 + 70% 𝑜𝑓 (503 % 𝑜𝑓 180 + 20) − 17 × 7
111. 3 3 + 4 5 + 2 2 = ? + 5 6
5 3 3
(a) 87 (b) 75 (c) 91
a) 6 6 b) 5 5 c) 4 4 (d) 72 (e) None of the above.
For Free Study Material & Quizzes visit : www.mathsbyarunsir.com
For any queries or suggestions email us @ mathsbyarunsir003@gmail.com or whatsapp @ 8881331466
128. 526 + (18)2 + 90% 𝑜𝑓 2400 + 640 =?3 − 21% 𝑜𝑓 2000 − 26 (a) 14 (b) 21 (c) – 1
(a) 26 (b) 16 (c) 36 (d) 2 (e) 11
(d) 46 (e) 56 145. 15% of 680 + 45% of 340 = ?
3
129. √1225 ÷ √343 × 45% of 760 = ? a) 255 b) 295 c) 345
(a) 1170 (b) 1710 (c) 1510 d) 325 e) 275
(d) 1710 (e) None of these 146. (?)2 + 93 = 382 – 39
a) 38 b) 30 c) 26
130. 175% 𝑜𝑓 460 + 110% 𝑜𝑓 170 + 2? = 1000 d) 22 e) 6
(a) 3 (b) 4 (c) 5 ? 3 60
(d) 2 (e) 1 147. 𝑜𝑓 √3375 = ? 𝑜𝑓 √144
5
(a) 18 (b) 12 (c) 33
131. 187.9 × 30.1 × 60.1 ÷ (34 × 64 ) = 18?
(d) 15 (e) 30
(a) 1 (b) 2 (c) 3
148. 40% of 400 + 15% of 620 = ?
(d) 4 (e) 5
1 3 1 3 18 35 (a) 273 (b) 243 (c) 283
132. 3 7 + 2 5 + 7 5 − 5 7 − 35 = (d) 193 e) 253
?
(a) 3 (b) 5 (c) 7 149. 2 5 + 3 3 − 1 2 = ? + 1 23
3 2 1
=?
30
(d) 9 (e) 11 a) 9 b) 4 c) 3
133. 36% 𝑜𝑓 245 − 40% 𝑜𝑓 10 = 10 − ? d) 1 e) 0
(a) 4.2 (b) 6.8 (c) 4.9 150. 15÷ 5× 4 – 2 = ? – 9
(d) 5.6 (e) None of these a) 23 b) 19 c) 17
134. 518 × ? × 9 = 303030 d) 15 e) 20
(a) 75 (b) 65 (c) 85 151. 23 × 46 ÷ 82 = (2)?−2
(d) 55 (e) None of these a) 7 b) 6 c) 9
135. 45% of 224 × ? % of 120 = 8104.32 d) 11 e) 8
(a) 67 (b) 62 (c) 59
152. √1521 ×
√1444
× (10)2 = √100 ×?
(d) 71 (e) None of these √169 √361
a) 40 b) 80 c) 70
136. (94) + (? ) = (145)2 − (56)2 − 3869
2 2
d) 50 e) 60
(a) 5184 (b) 72 (c) 84
(d) 7056 (e) None of these 153. 15% of 250+ 35% of 750 −75 = (? )2
137. 256 × 0.5 = 1024 ÷ ? × 4 a) 15 b) 16 c) 18
(a) 128 (b) 8 (c) 32 d) 10 e) 20
(d) 28 (e) None of these 154. 7.5× 8 – 10 = ? × 2.5
a) 15 b) 20 c) 25
138. [(35)3 ÷ 70 × 12] ÷ 25 = 58.8 × ? d) 30 e) 35
(a) 6 (b) 8 (c) 14 155. 7394+6295-3689 = ? × 40
(d) 22 (e) None of these a) 320 b) 240 c) 280
3 3 3
139. √8000 + √27 − √343 = ? d) 250 e) 300
a) 23 b) 16 c) 9 156. 9×9÷ 3+9× 123 = ? – 19 -23
d) 24 e) 19 a) 1176 b) 1174 c) 1177
140. 14400 ÷ 36 + 15600 ÷ 12 + 450 = 1800 + ? d) 1175 e) 1178
a) 410 b) 330 c) 390 157. 13× 23+ 27× 37 = (?) – 302
d) 350 e) 370 a) 1620 b) 1540 c) 1700
d) 1500 e) 1600
141. 7450 + 5880 − 6890 = 9000 − ? 158. 493+287-334 = -54 + ? × 5
a) 2560 b) 2760 c) 2460 a) 90 b) 100 c) 110
d) 2850 e) 2480 d) 95 e) 85
142. 32 × 25 + 44 × 18 + 348 ÷ 6 = ? 159. √5776 − √1444 + √729 = 43 + ?
a) 1550 b) 1620 c) 1650 a) 25 b) 20 c) 26
d) 1600 e) 1690 d) 24 e) 22
143. √1255 × 28 + 203 × 7 = (? )2 160. 78 × 26 ÷ 6 + 1262 = 1311 + (? )2
a) 47 b) 45 c) 49 a) 17 b) 22 c) 15
d) 51 e) 53 d) 13 e) 19
144. 28 × 16 - 28 = ? × 30

For Free Study Material & Quizzes visit : www.mathsbyarunsir.com


For any queries or suggestions email us @ mathsbyarunsir003@gmail.com or whatsapp @ 8881331466
161. √479 − 190 + 13 + √2209 = ? 178. (3)2 × (3)6 × (9)2 ÷ (27)2 = (3)?
a) 77 b) 79 c) 67 a) 4 b) 6 c) 7
d) 73 e) 81 d) 5 e) 8
3
162. 16 ÷ 8 + 4 × √2744 = (? )3
3 3 179. 123 + 447 – 170 + 500 =? – 200
a) 3 b) 4 c) 5 a) 1300 b) 1100 c) 1000
d) 2 e) 1 d) 1030 e) 1173
163. 756 ÷ 14 × 5 − √7921 = ? 180. (14)2 + 179 + (5)2 = (? )2
a) 183 b) 179 c) 178 a) 10 b) 20 c) 30
d) 181 e) 187 d) 40 e) 22
19 27
164. 12 − 16 = ? − 12 29 181. 1484 ÷ 28 + 1462 ÷ 34 − 12 × 7 = ?
a) 15/11 b) 13/7 c) 3 a) 12 b) 14 c) 18
d) 37/16 e) 37/14 d) 16 e) 20
165. 48% of 275 + 55% of 480 = ? 182. 42.5 × 15 + 37.5 × 25 = 1420 + ?
a) 394 b) 392 c) 398 a) 145 b) 165 c) 155
d) 386 e) 396 d) 170 e) 185
166. 42%of 650 + 243 - √? = 2 × (4)4 183. 2450 + 3760 − 3830 = 6000 − ?
(a) 18 (b) 16 (c) 20 a) 3610 b) 3620 c) 3580
(d) 24 (e) 12 d) 3600 e) 3520
167. ? % 𝑜𝑓 750 + (15)2 = (8)2 +356 184. 3√8000 + 3√27 − 3√343 = ?
(a) 42 (b) 16 (c) 22 a) 23 b) 16 c) 9
(d) 26 (e) 32 d) 24 e) 19
728 185. 33.33% of 189 + 66.66% of 99 = ?
168. ? × 15 + 24% of 550 – 38 =484
a) 153 b) 129 c) 123
(a) 20 (b) 36 (c) 32 d) 147 e) 135
(d) 24 (e) 28 186. (?)2 + 8 = 152 + 122
169. 36 × ? + 33 × 4 + √441 = (21)2 a) – 26 b) 124 c) 19
(a) 8 (b) 2 (c) 9 d) – 41 e) 94
(d) 11 (e) 14 187.√961 + √1225 + √576 = ?
170. 568 + 330 – (12)2 = 8% of ? a) 115 b) 90 c) 70
(a) 9255 (b) 9425 (c) 9755 d) 95 e) 65
(d) 9625 (e) 10225 188. 5% of 340 + 25% of 780 = ?
171. 26% of 250 + ?% of 640 = (15)² a) 312 b) 222 c) 122
(a) 35 (b) 25 (c) 20 d) 212 e) 162
(d) 15 (e) 45 189. 512÷?×6= 192
172. 23 × 13 + (11)² - ? = (19)² a) 11 b) 18 c) 12
(a) 49 (b) 39 (c) 59 d) 16 e) None of these
(d) 29 (e) 19 190. 432 ÷ 48 × 602 ÷ 86 = ?
?
173. 24 + 65% 𝑜𝑓 260 = 4 × 49.75 a) 63 b) 58 c) 53
(a) 640 (b) 840 (c) 960 d) 42 e) 54
(d) 720 (e) 800 1
191. 37 2 %of ?−√784 = 89
174. 2.46 × 15 + 25% of 92.4 = ? × 15 a) 320 b) 312 c) 325
(a) 10 (b) 2 (c) 8 d) 340 e) 310
(d) 6 (e) 4 192. 1 35 × 56 × 58 𝑜𝑓 32 ×? = 150
175. 460 + 927 – 433 + ? = (31)²
a) 120 b) 108 c) 100
(a) 1 (b) 9 (c) 3
d) 80 e) 132
(d) 7 (e) 5
√16
193. (2.16 × 31.25) ÷ ? =27
176. 50%𝑜𝑓 128 + 2 × 4 = ? + 10 a) 2.25 b) 2.5 c) 2
a) 64 b) 62 c) 60 d) 1.75 e) 1.5
d) 56 e) 82 194. 6080 ÷ 19 = ?% of 256
3
177. √1331
+ √81+? = 27 a) 137.5 b) 75 c) 150
11
a) 9 b) 8 c) 17 d) 125 e) 175
d) 6 e) 5
For Free Study Material & Quizzes visit : www.mathsbyarunsir.com
For any queries or suggestions email us @ mathsbyarunsir003@gmail.com or whatsapp @ 8881331466
195. (48% of 625) ÷ 0.75 = ? (a) 1.75 (b) 1.20 (c) 1.25
a) 800 b) None of these c) 40 (d) 1.45 (e) 1.80
d) 4000 e) 400 212. (24)² – (12)² – (10)² + (?)² = 207 × 3
196. 0.075% of 8800 – 1.9 = ? (a) 19 (b) 37 (c) 13
a) 4.6 b) 4.75 c) 4.7 (d) 23 (e) 17
d) 4.65 e) 4.5 213. (1285 + 215 – 720) ÷ (1620 + 1200 – 69.5 × 40) =?
4 (a) 23.5 (b) 13 (c) 21.5
197. 44 9 %of 1080 ÷ 5 = ? −28
a) 114 b) 118 c) 128 (d) 17.5 (e) 19.5
d) 124 e) 116 214. 25%of 360 + 12.5 × 4 = ?2 + 40
198. 65% of 480 - ? +175 =350 a) 9 b) 8 c) 10
a) 125 b) 129 c) 137 d) 12 e) 6
d) 147 e) 153 2
215. 324 +? = 40%of 1450
199. 3045 ÷ 87 = ? + 54 a) 18 b) 8 c) 12
a) – 27 b) – 14 c) – 19 d) 16 e) 14
d) – 21 e) – 25 56 3
216. ? + (8) = 13%of 4000
67 66
200. 264 × 201 ×? = 12 a) 7 b) 8 c) 14
a) 144 b) 156 c) 140 d) 28 e) None of these
d) 138 e) 152 217. √841 + √1296 − √1024 = √?
201. √5041 + √576 − √4096 = ? a) 1156 b) 1089 c) 1024
a) 36 b) 25 c) 31 d) 961 e) 1225
3 3
d) 39 e) 38 218. ? × 6 + √512 = 170
202. 250.26 × 250.24 = ? a) 2 b) 3 c) 4
a) 25 b) 1 c) 5 d) 5 e) 1
d) 125 e) 625 3 3 3
219. 4 12 + 1 4 − ? = √64
203. (34)2 - (38)2 + (31)2 = ? a) 5/2 b) 1 ½ c) 1
a) 694 b) 673 c) 678 d) 4 e) 2
d) 658 e) 701
3 3 3
220. 34 × 2 × 31 ÷ √961 = ? − 540 ÷ 18
204. √5832 − √2744 − √216 = ? (a) 98 (b) 83 (c) 93
a) – 5 b) 0 c) 3 (d) 95 (e) 82
d) – 2 e) -1
221. 3333 ÷ 33 + 4004 ÷ 26 + 650 ÷ 25 = ?
205. 29 × 31 − 24 = ? × 35 (a) 320 (b) 297 (c) 281
a) 𝟐𝟑 b) 𝟐𝟗 c) 𝟑𝟖 (d) 300 (e) 241
d) 𝟏𝟐 e) 𝟐𝟓 222. 15.5 × 28 ÷ 112 + 1230 ÷ 240 = ? ÷ 5
2 2 2
206. 36 - 28 - 39 = ? (a) 55 (b) 45 (c) 65
a) – 1089 b) – 969 c) - 1009 (d) 35 (e) 75
d) – 999 e) – 1039 9 4
207. (?)2 + 93 = 332 – 36 223. 13 × 221 + 1 9 × 378 = 241 + ?
a) 2 b) 38 c) 30 (a) 525 (b) 614 (c) 520
d) 18 e) 14 (d) 458 (e) 420
2
208. 55.55% of 45 + 57.14% of 91 = ? 224. 12.5% of 928 – 14 % of 980 + (12)2 ÷ (2)2 = ?
7
a) 74 b) 89 c) 92 (a) 10 (b) 8 (c) 12
d) 86 e) 77 (d) 14 (e) 16
5 1 1 5 1
209. ? + 2 + 1 4 = 1 8 + 2 2 + 7 4 225. (36% of 5000) ÷ (15% of 1800 + 90) = ?
a)
71
b)
73
c)
65
. (a) 7 (b) 5 (c) 8
8 8 8
63 1 (d) 4 (e) 3
d) 4
e) 7 8 525
226. + 2300 + √1600 = 1810 + 395 × 4
?
1725 3025 312 (a) 0.7 (b) 1.5 (c) 0.5
210. 25
+ ÷ 1248 = (? )2
121 (d) 0.3 (e) 1.2
(a) 17 (b) 23 (c) 15
227. (16)² + (21)² – (13)² + (?)² = 428 + 424
(d) 13 (e) 11
(a) 18 (b) 24 (c) 26
211. (180 × 170) ÷ 16 ÷ ? = 153 × 6 + 612
(d) 16 (e) 22
For Free Study Material & Quizzes visit : www.mathsbyarunsir.com
For any queries or suggestions email us @ mathsbyarunsir003@gmail.com or whatsapp @ 8881331466
228. 5 25 + 2 14 +? = 7 15 + 4 18 + 1 12 1 3 2
244. 2 4 + 3 5 – 1 3 =? −3 4
3

3 14 2
7 1 5 a) 9 14 b) 7 15 c) 6 15
a) 5 40 b) 3 5 c) 5 8 5
1 7 d) 7 14 e) None of these
d) 2 40 e) 40 245. 27% of 450 – ?% of 375 = 76.5
229. (14 × 21) ÷ 15 ÷ ? = 4 (a) 12 (b) 13 (c) 14
(a) 5.2 (b) 4.9 (c) 4.2 (d) 11 (e) 19
(d) 6.2 (e) 5.9 246. 311 × 17 = ? + 2482
2
230. 2652 − 441 + 928 − 6 3 % 𝑜𝑓 3375 = ? + (a) 2605 (b) 2715 (c) 2905
(31)2 (d) 2805 (e) 2875
3 1 2
(a) 1953 (b) 1853 (c) 1825 247. 12 + 4 × 3 =?
5 5 3
(d) 1935 (e) 1950 (a) 28 (b) 27 (c) 14
231. (3080 + 6160) ÷ ? = 330 (d) 19 (e) 22
3
(a) 27 (b) 25 (c) 28 248. √13824 × √? = 864
(d) 24 (e) 23 (a) 1225 (b) 1296 (c) 1349
15 (d) 1369 (e) 1156
232. 5 17 % 𝑜𝑓 4913 − (15)2 = (? )2
(a) 9 (b) 11 (c) 7 249. 1615 ÷ (50 × 0.85) = ?
(d) 6 (e) 8 (a) 35 (b) 34 (c) 38
(d) 37 (e) 28
233. ? × (523.5 + 687.5) = 24220
250. 19.5% of 78 = ?
(a) 31 (b) 20 (c) 42
(a) 16.21 (b) 11.56 (c) 12.25
(d) 18 (e) 24
(d) 15.21 (e) 17.25
234. (272 − 32) × (124 + 176) ÷ (17 × 15– 15) = ? + 15 × 16 251. 16.8 × 6.25 = 20% of ?
(a) 50 (b) 65 (c) 72 (a) 576 (b) 525 (c) 676
(d) 60 (e) 55 (d) 625 (e) 566
3
235. 125% of 92 – √4096 + ? = √10201 441 ?
252. = 49×9
(a) 5 (b) 2 (c) 4 ?
(d) 3 (e) 9 (a) 441 (b) 21 (c) 147
236. 3 12 + 4 34 + 9 34 + 6 78 =? +5 34 × 12 + 19 (d) 63 (e) 484
4 3
(a) 13 (b) 9 (c) 16 253. × 11 × 11979 =?
9
(d) 3 (e) 2 (a) 1756 (b) 1262 (c) 1452
1
2
1
237. (1156) − (1728) + 178 − (? )2 = √2025 + 55
3
(d) 1552 (e) 1652
(a) 100 (b) 12 (c) 144 254. √1444 ÷ 19 + 3.5 × √16 = (?)²
(d) 15 (e) 10 (a) 2 (b) 3 (c) 5
238. 111 × 36 – (61)2 − (? )2 = 3150 − (10)2 − 3000 (d) 6 (e) 4
3
(a) 15 (b) 16 (c) 18 255. 3375 + √1331 = (?)² – 19 × 5
(d) 14 (e) 13 (a) 59 (b) 49 (c) 39
239. (2744)1/3 + (17)2 − 3 = ? − 79 × 8 (d) 51 (e) 41
(a) 930 (b) 938 (c) 918 256. 60% of 300 + ? = (16)²
(d) 994 (e) 932 (a) 436 (b) 76 (c) 38
240. 323 × 15 + (?)² = 4989 (d) 68 (e) 86
(a) 12 (b) 11 (c) 13 257. 32.12 + 34.18 = 47 × ? – 27.7
(d) 14 (e) 17 (a) 3 (b) 4 (c) 2
241. 5760 ÷ 45 × 15 = ? (d) 1 (e) 5
(a) 1850 (b) 1875 (c) 1920 258. (384 ÷ 4)÷ 3 = (2)?
(d) 1925 (e) 1970 (a) 4 (b) 6 (c) 8
242. 81% of 2300 – 34% of 550 = ? (d) 7 (e) 5
(a) 1574 (b) 1676 (c) 1624 1 3 2 1
259. 1 2 + 2 4 – 5 3 = ? – 4 6
(d) 1596 (e) 1723 3 2 1
243. 25639 – 5252 – 3232 = ? a) 2 4 b) 3 3 c) 3 4
(a) 17254 (b) 16154 (c) 17155 d) 2 3
2 1
e) 2 4
(d) 16255 (e) 17815
For Free Study Material & Quizzes visit : www.mathsbyarunsir.com
For any queries or suggestions email us @ mathsbyarunsir003@gmail.com or whatsapp @ 8881331466
260. 4368 + 2158 – 596 - ? = 3421 + 1262 + 5² × 48 276. 460 × 15 – 5 × 20 = ?
(a) 35 (b) 47 (c) 51 (a) 92000 (b) 4600 (c) 137800
(d) 56 (e) 45 (d) 7000 (e) None of these
2 3 4 277. 5163 – 4018 + 3209 = ?
261. 3 of 5 of 5 of 125% of ? = 1112
(a) 4174 (b) 4264 (c) 4804
(a) 2780 (b) 2750 (c) 2650
(d) 4354 (e) None of these
(d) 2825 (e) 2675 1 1
278. 4 5 × 3 3 + ? = 20% of 120
262. 780 ÷ √676 + (? )2 = 1326 1
a) 35 (b) 38 (c) 34 (a) 10 15 (b) 10 (c) 5
(d) 36 (e) 32 (d) 15 (e) None of these
263. 3695.12 + 4458.02 - ? = 7592.14 279. 4848 ÷ 24 × 11 – 222 = ?
(a) 562.14 (b) 661.14 (c) 561 (a) 200 (b) 2444 (c) 2000
(d) 561.14 (e) 661 (d) 115 8
3
(e) None of these
3 5 1 19
264. 1 5 + 2 6 − 3 6 = ? × 15 280. 1750 ÷ 350 × 50 + 111 = (? )2 0
(a) 0.5 (b) 1 (c) 1.5 a) 19 b) 39 c) 29
(d) 2 (e) 2.5 d) 9 e) 49
34×4−12×8+61 2 15 7
265. 62 +√2025+(11)2 = ? 281. × ÷ + 88 × 26 + 562 =?
15 18 9
(a) 1 (b) 0.5 (c) 0.45 (a)28507
1
(b)2850 (c)27507
1
(d) 0.25 (e) 1.5 2
1 (d)25557 (e) None of these
266. 464 ÷ (16 × 2.32) + 4 2 = ?
3
(a) 15 (b) 16 (c) 20 282. of (20% of 920) = ?% of 650
4
(d) 17 (e) 10 (a) 19 (b) 22 (c) 25.26
267. (5863 − √2704) × 0.5 = ? + (51)2 (d) 21.23 (e) None of these
63000
(a) 304.5 (b) 303.75 (c) 302 285×
525
283. = ? −86
(d) 305 (e) 305.5 600
(a)141 (b)142 (c)143
268. 1231 – ?% of 500 = 145% of 780
(d)145 (e)149
(a) 15 (b) 18 (c) 20
(d) 25 (e) 30 110
1 ×?
100
269. 5 8 𝑜𝑓 208 + 786 = 2000 − ? 284. 112 = 4 × 8888
100
(a) 126 (b) 138 (c) 135 (a) 36198.4 (b) 405550 (c) 256892.8
(d) 156 (e) 148 (d) 548260 (e) None of these
270. √360 − 225 × 2 + 379 = ? 3 15 3276000
285. × 100 × = 5 ÷?
(a) 17 (b) 19 (c) 279 5 18

(d) 289 (e) None of these 5 1 3


a) 3276 b) 3276 c) 4286
271. 9 × 81 ÷ 273 = (3)?
3 2
1
(a) 3 (b) 4 (c) 5 d) 2516 e) None of these
(d) 6 (e) None of these 13% 𝑜𝑓 5700
286. =?
272. 572 ÷ 26 × 12 – 200 = (2)? 15% 𝑜𝑓 8550
26 28 22
(a) 5 (b) 6 (c) 7 (a)45 (b)45 (c)45
(d) 8 (e) None of these (d)45
41
(e) None of these
1 5 7
273. 4 2 − 2 6 =? − 1 12 50
1 5 7
287. 33 × 5200 – 35 × 2500 =? × 6
a) 3 4 b) 3 12 c) 2 12
3
(a)10052 (b)14528 (c)12212
d) 3 4 e) None of these (d)10092 (e) None of these
274. 36% of 245 – 40% of 210 = 10 – ? 3 7
288. (75 × 2) + (90 × 3) – 35 × 3 × 6 ÷ = ? – 105 – 696 ×
(a) 4.2 (b) 6.8 (c) 4.9 5 3
(d) 5.6 (e) None of these (a)1099 (b)1199 (c)1000
275. 40% of 265 + 35% of 180 = 50% of ? (d)1212 (e) (e)None of these
(a) 338 (b) 84.5 (c) 253.5 289. 13956 + 412 − 2704 = ? − (31)2
(d) 169 (e) None of these a) 28 b) 45 c) 65
d) 85 e) 98
For Free Study Material & Quizzes visit : www.mathsbyarunsir.com
For any queries or suggestions email us @ mathsbyarunsir003@gmail.com or whatsapp @ 8881331466
3840 1440 1330 (d) 120 (e) 80
290. √ + − =? ? 15×4 –40
60 40 70
306. =
a) 10 b) 9 c) 8 √25 2
d) 7 e) 11 (a) 20 (b) 45 (c) 25
4200 525 (d) 50 (e) 60
291. 25 × 18 + 40 − 105 = 740−?
307. 621 ÷ 27 × 2 – 37 = √?
a) 200 b) 220 c) 190 a) 9 b) 3 c) 81
d) 170 e) 150
d) 3√3 e) 21
292. 3845 + 43820 + 2640 − 5965 = (? )2 308. 36% of 250 × 18% of 50 = ? + 10
a) 75 b) 60 c) 80 (a) 820 (b) 810 (c) 790
d) 70 e) 72 (d) 800 (e) 700
293. 400 ÷ 20 × 35 + 6666 ÷ 33+? = 1100 309. [(7)² –(6)²]÷ 26 = 18 ÷ ?
a) 180 b) 198 c) 195 (a) 18 (b) 36 (c) 9
d) 205 e) 200 (d) 40 (e) 42
294. 28 × 14.5 + 1680 ÷ 15 + 445 = 1000−? 310. √(256) × √169 + 3600 ÷ 12 = 800 − ?
a) 27 b) 37 c) 47 (a) 312 (b) 280 (c) 292
d) 50 e) 40 (d) 324 (e) 296
295. √(256) × √169 + 3600 ÷ 12 = 800 − ? 311. 37.5 × 14 + 800 − (26)2 + 136 =?
a) 312 b) 280 c) 292 a) 785 b) 800 c) 810
d) 324 e) 296 d) 825 e) None of these
296. 37.5 × 14 + 800 − (26)2 + 136 =? 312. 5430+3780 - 6430 = 2260 + ?
a) 785 b) 800 c) 810 a) 530 b) 490 c) 500
d) 825 e) 765 d) 520 e) 510
297. 5430+3780 - 6430 = 2260 + ? 313. 2160÷12+5740÷14-3150÷15+ ? = 400
a) 530 b) 490 c) 500 a) 16 b) 32 c) 28
d) 520 e) 510 d) 24 e) 20
298. 2160÷12+5740÷14-3150÷15+ ? = 400
a) 16 b) 32 c) 28 314. √3481×7+√5625 ×4= 500+ ?
d) 24 e) 20 a) 213 b) 223 c) 203
d) 233 e) 243
299. √3481×7+√5625 ×4= 500+ ? 315. 48% of 525 + ? % of 250 = 499
a) 213 b) 223 c) 203 (a) 88.8 (b) 76.6 (c) 82.6
d) 233 e) 243 (d) 98.8 (e) 92.8
1 5
316. 52 𝑜𝑓 78 𝑜𝑓 28
1
300. 220 + 33 3 % of 1020 + ? = 5 9 % of 2700 + 400 𝑜𝑓 1600 = 260 + ? – 499
(a) –10 (b) 10 (c) 20 (a) 264 (b) 480 (c) 364
(d) –20 (e) 15 (d) 342 (e) 420
28 9 11
301. 252 × 22 × 3 ×? = (10)2 317. √52 × 41 × 5 – 172 – 75 =?
(a) 400 (b) 500 (c) 600 (a) 69 (b) 71 (c) 79
(d) 650 (e) 550 (d) 63 (e) 89
302. (1428 + 600 ÷ 2) ÷ (720 ÷ 5) 318. √256 × 49 + (19)2 + 11 = (? )2
(a) 14 (b) 144 (c) 18 (a) 484 (b) 22 (c) 24
(d) 16 (e) 12 (d) 42 (e) 26
4750
303. √1714 + 520 ÷ 2 – 190 × 5 = ? 319. 625 ÷ 2.5 + 283 – 157 + √576 + (10)² = ?
(a) 32 (b) 36 (c) 42 (a) 9.5 (b) 8.5 (c) 6.5
(d) 52 (e) 26 (d) 8 (e) 11.5
304. 25 × 5 ÷ 56.4 = 25(?)
2.7 4.2 320. 130% of 850 – 175% of 460 = 30% of ?
(a) 1.7 (b) 3.2 (c) 1.6 (a) 100 (b) 1000 (c) 10
(d) 3.6 (e) 2.8 (d) 30 (e) 300
321. ? × 13 3 = 6 11 × 7 29 × 9 13
1 6 4

305. √441– 41 × 42 ÷ 7 = ?
(a) 48 (b) 11 (c) 22
(a) 20 (b) 60 (c) 180 (d) 33 (e) 39

For Free Study Material & Quizzes visit : www.mathsbyarunsir.com


For any queries or suggestions email us @ mathsbyarunsir003@gmail.com or whatsapp @ 8881331466
322. √? = (1872 ÷ 12) ÷ 6.5 (a) 90 (b) 180 (c) 270
(a) 484 (b) 676 (c) 576 (d) 162 (e) 540
(d) 529 (e) 625 338. ? = 40% of 320 + 4³ ÷ 18 × 108 ÷ 8
(a) 176 (b) 172 (c) 168
323. 3? = √120 × 15 − 65 × 20 + 229 (d) 166 (e) 186
(a) 2 (b) 3 (c) 4 339. 60% of (152 ÷ 19) × ? = 3³ × 4²
(d) 6 (e) 9 (a) 80 (b) 60 (c) 45
324. 8973– 2378 + 1577 =?2 + 4572 (d) 90 (e) 30
(a) 40 (b) 45 (c) 50 340. √12.25 × 18 – (?)² = (6)² + √4
(d) 55 (e) 60 (a) 7 (b) 6 (c) 5
325. 52% of 180 × 45% of ? = 2106 (d) 4 (e) 3
(a) 40 (b) 50 (c) 60 341. (1250 + 1725) ÷ (825 + 365) = ?
(d) 55 (e) 65 (a) 1.5 (b) 2.5 (c) 1
1 2 1 2 1 3 (d) 2.25 (e) 2.75
326. 7 9 + 6 3 − 4 2 = ? − 5 9 + 4 4 − 4 4
2 8 342. √625 ÷ √16 × 6 = ?% of 300
a) 14 b) 14 9 c) 14 2
(a) 15 (b) 12.5 (c) 17.5
3
d) 14 4 e) 15 (d) 10 (e) 8.5
343. 26 × 15 + 310 – (15)² = 25% of ?
22 276 7 (a) 1600 (b) 1800 (c) 1900
327. ? = 10 23 𝑜𝑓 630 𝑜𝑓 4 12 𝑜𝑓 18
(d) 1500 (e) 1700
(a) 330 (b) 352 (c) 374
344. √81 × √625 + 1225 = (?)² – 150
(d) 385 (e) 396
407 372 (a) 50 (b) 45 (c) 35
328. 777 × 63 + 744 × 186 = ? (d) 30 (e) 40
(a) 116 (b) 126 (c) 128 345. 4900 ÷ 28 × 444 ÷ 12 – 6450 = (?)²
(d) 136 (e) 140 (a) 6 (b) 7 (c) 5
(512) 64 (d) 4 (e) 8
329. (2 )?+2 = 32 × (128) × 8
(a) 6 (b) 5 (c) 4 346. 38% of 250 – 85% of 560 + 13 × ?= 61
(d) 3 (e) 7 (a) 34 (b) 26 (c) 12
(d) 28 (e) 32
330. ? = (62 − 32 ) × (82 − 42 ) ÷ (10.52 − 1.52 )
347. 2 9 × 1 19 ÷ 2 3 − 12 =? – 1 12
1 2 1
(a) 6 (b) 8 (c) 9
5 3
(d) 12 (e) 15 a) 2 b) 4 c) 2
331. 35% of 540 + 410% of 30 = 39% of ? 1
d) 2 e) 2
(a) 900 (b) 800 (c) 700
(d) 950 (e) 850 348. √? × 12 – 26% of 1650 + 19 = 13 × 34
5 (a) 4900 (b) 5041 (c) 5329
332. ?² = 40% of 11 of 352
(d) 5476 (e) 5625
(a) 12 (b) 16 (c) 6 39
3

(d) 4 (e) 8 349 53.5% 𝑜𝑓 720 × [√676 √784


÷
√3969
√125
× 3 ] =?
√729
333. ? ÷ 15 × 11 = 195 ÷ 108 × 132 ÷ 65 (a) 281 (b) 342 (c) 298
(a) 5 (b) 15 (c) 25 (d) 321 (e) 441
(d) 55 (e) 12 350. 23 89 + 15 79 − ? = 12 13 × 23
4
334. ? = √60% 𝑜𝑓 850 + 80% 𝑜𝑓 550 − 83 × 2 a) 31 9 b) 32 c) 32 59
(a) 32 (b) 22 (c) 28 d) 32 79 e) 31 89
(d) 38 (e) 26 351. 16.5 × 18 ÷ √729 × 11 + 48 = (? )2
(a) 15 (b) 19 (c) 13
335. ?2 = (√1444 + √676) ÷ 4
(d) 5 (e) None of these
(a) 6 (b) 16 (c) 8
352. 4444 ÷ 44 + 3003 ÷ 3 = ? + 841
(d) 2 (e) 4
(a) 271 (b) 261 (c) 251
336. 2? = 64 × 288 ÷ 256 ÷ 36 × 16
(d) 281 (e) 291
(a) 2 (b) 3 (c) 4
353. (4080 + 5160) ÷ ? = 440
(d) 5 (e) 6
3
(a) 21 (b) 25 (c) 31
337. 90% of ?= √576 ÷ √64 × √729 (d) 28 (e) None of these
For Free Study Material & Quizzes visit : www.mathsbyarunsir.com
For any queries or suggestions email us @ mathsbyarunsir003@gmail.com or whatsapp @ 8881331466
354. ? × (623.5 + 587.5) × 5 = 24220] 1
370. 37 2 % of 4200 + (25)² + (10)³ – 700 = (?)²
(a) 5 (b) 6.5 (c) 5.5
(a) 50 (b) 60 (c) 55
(d) 4.5 (e) 4
(d) 75 (e) 80
355. ? = (19² - 11²) × (8² - 4²) ÷ (13.5² - 3.5²) ÷ 96 × 289 ?+62.5
(a) 201 (b) 204 (c) 402 371. 3 + 180 × 2 – (12)2 – 20 = (13)2 + 87
(d) 102 (e) 408 (a) 137.5 (b) 125.5 (c) 117.5
356. √√961 + √841 − √625 ÷ 5 × √35 = ? (d) 112.5 (e) 107.5
(a) 5 (b) 6 (c) 8 372. 22.5 × 12 + (11)² – √? = (19)²
(d) 9 (e) 7 (a) 800 (b) 750 (c) 825
5
357. ?² ÷ 352 = 40% of 11 × 400 (d) 900 (e) 950
624 440
(a) 16 (b) 150 (c) 15 373. + 3.5 × 6 + 27 = (6)2 +
? 2
(d) 160 (e) 140 (a) 5 (b) 3 (c) 4
358. 60% of (608 ÷ 19) × ? =3³ × 4² (d) 8 (e) 7
(a) 21 (b) 22 (c) 22.5 1 1
374. ?% of 625 + 33 3 % of 750 + 9 11 % of 5500 – √625 = (10)³
(d) 21.5 (e) 23 (a) 44 (b) 46 (c) 56
3
359. 12.5% of ?= √1089 ÷ √1331 × 24 (d) 54 (e) 64
(a) 625 (b) 576 (c) 600 3
375. 763 + 882 + 13 of 6500 + ? = (60)²
(d) 529 (e) 676
(a) 465 (b) 445 (c) 425
360. 4.4 + 14.44 + 41.14 = ? – 14.41 – 1.4
(d) 455 (e) 545
(a) 74.21 (b) 75.79 (c) 76.19
376. 2450 – 1540 + 1700 – 710 = ?% of 1900
(d) 78.79 (e) 73.79
(a) 120 (b) 160 (c) 125
361. 81% of 2300 - ? = 34% of 596
(d) 200 (e) 100
(a) 1751.36 (b) 1680.36 (c) 1791.36 1626 1
(d) 1660.36 (e) 1691.36 377. 40% of 1325 + 299 + 6 = 33 3 % of 3300 + ?
362. (7921 ÷ 178) – 5.5 = √? (a) 10 (b) 20 (c) 0
(a) 39 (b) 1369 (c) 1521 (d) 15 (e) 25
4750
(d) 1225 (e) 1444 378. 625 ÷ 2.5 + 283 – 157 + √576 + (10)² = ?
363. √484 + √8649 + 192 = 212 + √1089+? (a) 9.5 (b) 8.5 (c) 6.5
(a) 2 (b) 3 (c) 4 (d) 8 (e) 11.5
(d) 5 (e) 7 379. 2250 + 270 – √400 + 3125 ÷ 25 + ? = (60)²
3
364. √13824 × √? = 864 (a) 875 (b) 945 (c) 935
(a) 36 (b) 38 (c) 1225 (d) 975 (e) 925
(d) 1521 (e) 1296 380. (2/5){(3/5) × 750 + 40% of 375} = ? × 16
365. 4368 + 2158 – 596 -? = 3421 + 1262 a) 20 b) 24 c) 15
(a) 1247 (b) 1157 (c) 1367 d) 12 e) 18
(d) 1457 (e) 947 381. (42% of 1500 – 230) ÷ 25 = ? ÷ 4
3 14 21 a) 64 b) 32 c) 48
366. 25 × 17 ÷ 34 = (? )2
d) 24 e) 76
(a) 1 (b) 0.4 (c) 0.5 382. {(420 ÷ 28)% of 1400} ÷ 7 = ?
(d) 0.8 (e) 1.2 a) 35 b) 20 c) 15
367. 34% of 250 – 45% of 340 = ? – 150 d) 30 e) 25
(a) 85 (b) 82 (c) 91 383. (15 × 16) + 242= ? × 16
(d) 101 (e) 92 a) 31 b) 45 c) 51
17 53 91 252
368. 19 × 221 × 106 ×? = 4 d) 24 e) 40
(a) 342 (b) 441 (c) 542 384. (2/5)(32% of 4500 – 440) = ? × 8
(d) 384 (e) 424 a) 50 b) 45 c) 30
4 2 2 1 1
369. 1 5 + 3 5 + 2 3 + 1 6 = 1 9 + 1 6 + ?
1 d) 25 e) 20
41 31 34
385. (6/7) × 1050 – (3/8) × 1200 = ?% of 750
a) 6 45 b) 6 35 c) 6 45 a) 40 b) 60 c) 20
34 41 d) 35 e) 25
d) 7 45 e) 5 45

For Free Study Material & Quizzes visit : www.mathsbyarunsir.com


For any queries or suggestions email us @ mathsbyarunsir003@gmail.com or whatsapp @ 8881331466
386. 35% of 1600 – 142 = ? × 14 यदि ‘x 'के 35% के 5/3 के 20% और 46 का योग 74 है, तो ‘x’ का मान
a) 16 b) 32 c) 24 ज्ञात करें?
d) 26 e) 34 a) 120 b) 150 c) 180
387. (5/6){(122 + 221) – (212 + 112)} = ? ÷ 24 d) 240 e) 200
a) 360 b) 420 c) 380 404. What will come in the place of question mark (?) in the given
d) 240 e) 260 expression?
388. (1600 – 160 – 16) ÷ 16 = ? िी गई अदिव्यदि में प्रश्न दिह्न (?) के स्थान पर क्या आएगा?
a) 89 b) 93 c) 84 22.5% of 400 + 32% of 625 = ? + 100
d) 74 e) 69 a) 280 b) 250 c) 190
389. (16 × 15) + (45 × 12) + (30 × 24) = ? d) 150 e) 320
a) 1800 b) 1240 c) 1200 405. (315 × 81) ÷ 45 – 147 = ? × 3
d) 1250 e) 1500 a) 160 b) 180 c) 120
390. (22% of 1500 + 15% of 2200) = ? × 11 d) 140 e) None of these
a) 60 b) 45 c) 80 406. √1521 + √1681 = ?2 – 41
d) 50 e) 95 a) 11 b) 15 c) 9
391. (342 – 20% of 5280) = ? ÷ 3 d) 17 e) 21
a) 250 b) 450 c) 300 407. 964 + 256 - ? = 465 + ? + 121
d) 650 e) 300 a) 256 b) 297 c) 357
392. 321 + 52 × 5 – 125 = ? × 12 d) 305 e) None of these
a) 64 b) 46 c) 54 408. (1/2 × 4/5 + 3/5 – 1/5) × 245 = ?
d) 38 e) 30 a) 142 b) 196 c) 184
393. (78 × 15) ÷ 52 × 8 = ? + 55 d) 154 e) 212
a) 200 b) 175 c) 160 409. ? = 45% of 2400 – 152 – 282
d) 145 e) 125 a) 71 b) 121 c) 51
394. (48 × 12) ÷ 18 × 8 = 4? d) 141 e) None of these
a) 8 b) 6 c) 3 410. 23% of ? = 25% of 448 + 12 ×√(841) – 20% of 460
d) 5 e) None of these a) 1200 b) 1400 c) 1600
395. 645 + 255 + 365 – 575 = ? d) 2400 e) 1800
a) 580 b) 720 c) 540
411. √3481 + √4356= ?% of 500
d) 690 e) 500
a) 25 b) 20 c) 15
396. 62.5% of 640 + 45% of ? = 252
d) 35 e) None of these
a) 750 b) 600 c) 500
412. (4/9) × 486 + 35% of 120 + (3/8) × 208 = ?
d) 800 e) None of these
a) 412 b) 260 c) 398
397. 7/8 + 7/16) × 256 – 12 = ?2
d) 336 e) 284
a) 25 b) 18 c) 30
413. (7800 ÷ 312) + 65% of 1120 = 9 × ? + 24
d) 8 e) 12
a) 81 b) 39 c) 27
398. 16/15 of 21/28 of 321.5 = ?
d) 36 e) 24
a) 257.2 b) 251.4 c) 264.5
414. ? × 15 + (3960 ÷ 12) = 262 + (350 ÷ 25)
d) 224.3 e) None of these
a) 56 b) 14 c) 24
399. √4096 + √3249 = (?)2 d) 36 e) 40
a) 21 b) 9 c) 17 415. 45% of 1200 – 32% of 1500 = 15% of ?
d) 13 e) 11 a) 600 b) 250 c) 500
400. 24% of 35% of 150% of 2500 = ? d) 480 e) 400
a) 325 b) 305 c) 315 416. (322 – 222) ÷ 32= ?
d) 335 e) None of these a) 50 b) 60 c) 25
401. 32% of 3500 + 28% of 2500 = ?% of 7280 d) 30 e) 15
a) 15 b) 20 c) 25 417. {(45 × 220) ÷ 33} + 325 = ?2
d) 30 e) None of these a) 25 b) 15 c) 35
402. 37.5% of 2016 + 25% of 3648 = ? × 12 d) 20 e) None of these
a) 160 b) 120 c) 170 418. (111 + 121 + 131 + 141) ÷ 21 = ?
d) 145 e) None of these a) 19 b) 24 c) 35
403. If the sum of 20% of 5/3 of 35% of ‘x’ and 46 is 74, find the d) 28 e) 14
value of ‘x’.

For Free Study Material & Quizzes visit : www.mathsbyarunsir.com


For any queries or suggestions email us @ mathsbyarunsir003@gmail.com or whatsapp @ 8881331466
419. 22% of 3500 – 152 = 20% of 725 + ? (d) 2 (e) 11
a) 125 b) 320 c) 480 437. 15% of 680 + 45% of 340 = ?
d) 240 e) 400 (a) 255 (b) 295 (c) 345
420. ?% of 4200 + 1380 × 5/46 = 25% of 3120 (d) 325 (e) 275
a) 40 b) 35 c) 20 438. (?)2 + 93 = 382 – 39
d) 15 e) 25 (a) 38 (b) 30 (c) 26
421. 13 × 8 + 13 × 12 + 13 × 16 + 13 × 20 = ? × 8 (d) 22 (e) 6
a) 85 b) 102 c) 77 439. 2(2/3) x 11(3/4) = 44/5 x 2a/6 x 114+b
d) 85 e) 91 (a) a = -28/5, b = -13/4
422. 737 × 3 + 363 × 2 + 11 × 33 = ? × 11 (b) a = -28/5, b = -9/4
a) 120 b) 300 c) 240 (c) a = -23/5, b = -13/4
d) 280 e) None of these (d) a = -23/5, b = -9/4
423. ? × 32 – 25% of (480 × 128) = 0 (e) None of the Above
a) 80 b) 240 c) 64 440. 5? 𝑜𝑓 3√3375 = 60? 𝑜𝑓 √144
d) 480 e) None of these (a) 18 (b) 12 (c) 33
424. 93 × (8910 ÷ 110) = (14580 ÷ 20) × ? (d) 15 (e) 30
a) 27 b) 42 c) 81 441. √625+ ? = 3 36 of 132
d) 729 e) None of these (a) 421 (b) 440 (c) 437
425 262 + 113 - √49 = 2 × ?3 (d) 425 (e) 401
a) 8 b) 10 c) 12 442. 472 + 602 - 572 = ?
d) 5 e) None of these (a) 2590 (b) 2490 (c) 2560
426. 65% of 1200 – 15% of 400 = ?3 × 45% of 200 (d) 2650 (e) 2480
a) 4 b) 1 c) 3 443. (?)2 + 53 = 192 – 11
d) 2 e) None of these (a) 35 (b) 23 (c) 11
427. 35% of 5000 – 322 + 3 = ? (d) 15 (e) 27
a) 825 b) 569 c) 729 444. 12.5% of 200 + 20% of 100 + 55.55% of 45 = ?
d) 689 e) None of these (a) 67 (b) 70 (c) 88
428. 20% of 5000 – 25% of 3000 = ? × 52 (d) 64 (e) 82
a) 10 b) 15 c) 8 445. 12(3/4) x 13(3/4) = 1442/4 x 12a/4 x 134+b
d) 16 e) None of these (a) a = -1, b = -13/4 (b) a = -1, b = -9/4
429. 16 × 15 – 240 – 32 × 10 + 420 = ?2 (c) a = 0, b = -13/4 (d) a = 0, b = -9/4
a) 10 b) 15 c) 5 (e) None of the above
d) 12 e) None of these 446. (0.064)113 ÷ 0.1654 = (0.064)?
430. 65% of 200 – 26% of 500 = ? (a) 86 (b) 74 (c) 65
a) 5 b) 8 c) 4 (d) 77 (e) 92
d) 10 e) None of these 447. 10(3/4) x 5(3/4) = 1002/6 x 10a/5 x 56+b
431. 45% of 1500 – 2 × 52= ?2 (a) a = 17/12, b = -21/4 (b) a = 5/12, b = -17/4
a) 25 b) 35 c) 5 (c) a = 5/12, b = -21/4 (d) a = 17/12, b = 17/4
d) 20 e) None of these (e) None of the Above
432. (32 × 25) – 202 = 20% of ? 448. 150 x 11 = ? - 230 x 9
a) 1000 b) 3200 c) 2000 (a) 3640 (b) 3650 (c) 3720
d) 2500 e) None of these (d) 3660 (e) 3680
900
433. 22% of 5000 + 14% of 400 = ?2 449. 6
= 14 × 11 + √225 = ?
a) 26 b) 14 c) 24 (a) 3 (b) 11 (c) 21
d) 36 e) None of these (d) 1 (e) 25
3 2
434. 7 – 13 – ? = 25% of 120 450. 66.66% of 33 + 77.77% of 261 = ?
a) 216 b) 144 c) 156 (a) 231 (b) 228 (c) 204
d) 246 e) None of these (d) 249 (e) 225
435. 80% of 2500 - ?% of 2500 = 103 451. (?)2 + 55 = 162 + 322
a) 20 b) 60 c) 30 (a) 35 (b) 155 (c) 95
d) 40 e) None of these (d) 125 (e) 185
436. 28 x 16 - 28 = ? x 30 452. (0.027) ÷ 0.09 = (0.3)?
107 59

(a) 14 (b) 21 (c) – 1 (a) 212 (b) 203 (c) 194


(d) 206 (e) 221
For Free Study Material & Quizzes visit : www.mathsbyarunsir.com
For any queries or suggestions email us @ mathsbyarunsir003@gmail.com or whatsapp @ 8881331466
? 120 3 6
453. of √400 = ? Of √2197 470. √256 + ? = 3 3 7 of 168
78
(a) 93 (b) 87 (c) 78 a) 632 b) 642 c) 612
(d) 99 (e) 84 d) 636 e) 621
454. 195 x 16 = ? - 280 x 4 471. 142 + 152 - 342 = ?
(a) 4290 (b) 4240 (c) 4180 a) -755 b) -795 c) -735
(d) 4320 (e) 4310 d) -655 e) -675
2 3 2
455. 71.42% of 189 + 77.77% of 306 = ? 472. (?) + 9 = 32 – 6
(a) 358 (b) 385 (c) 373 a) 17 b) 1 c) 13
(d) 370 (e) 349 d) 9 e) 5
456.
660
- 15 x 12 + √256 = ? 473. (?) + 56 = 14 + 342
2 2

6 a) 111 b) 54 c) 24
(a) -50 (b) -38 (c) -46
d) 21 e) 36
(d) -40 (e) -54 ? 3 16
457. 115 x 15 = ? - 150 x 11 474.. of√64 = ? Of √100
10
(a) 3385 (b) 3375 (c) 3315 a) 20 b) 23 c) 26
(d) 3435 (e) 3395 d) 44 e) 41
458.
3 3
√343 − √2197 + √2744 = ?
3 475. 135 x 15 = ? - 190 x 3
(a) 7 (b) 10 (c) 11 a) 2515 b) 2605 c) 2585
(d) 8 (e) 14 d) 2635 e) 2595
2 ⅓ ¼
459. 2 2
36 - 45 - 24 = ? 2 476. 25 - 64 + 6561 = ?
(a) -1395 (b) -1405 (c) -1255 a) 636 b) 642 c) 621
(d) -1305 (e) -1275 d) 639 e) 630
19 13
460. (?)2 + 31 = 202 + 162 477. 35 × 95 × ? = 15
(a) 70 (b) 85 (c) 55 a) 229 b) 209 c) 225
(d) – 5 (e) 25 d) 213 e) 239
0.18 0.22
461. 1024 x 1024 = ? 478. √7396 + √400 + √6400 = ?
(a) 64 (b) 1 (c) 4 a) 191 b) 184 c) 182
(d) 16 (e) 256 d) 186 e) 181
2 2 2
462. (19) + (36) + (37) = ? 479. 320.03 x 320.57 = ?
(a) 3053 (b) 3031 (c) 3047 a) 16 b) 1 c) 2
(d) 3030 (e) 3026 d) 4 e) 8
4
463. √961+? = 4 6 of 36 480. √1681+ ? = 1 26of 72
(a) 127 (b) 148 (c) 137 a) 75 b) 60 c) 55
(d) 123 (e) 143 d) 67 e) 79
464. 35 × 34 - 30 = ? × 40 481. 33 x 20 - 20 = ?x 40
(a) 21 (b) 29 (c) 28 a) 9 b) 16 c) 32
(d) 30 (e) 27 d) 7 e) 3
465. 36 × 16 - 48 × 50 = ? 482. 142 + 152 - 592 = ?
(a) -1833 (b) -1811 (c) -1823 a) -3080 b) -3070 c) -2960
(d) -1824 (e) -1814 d) -3060 e) -3130
466. 45% of 580 - 30% of 530 = ? 483. 19 x 44 - 14 x 40 = ?
(a) 32 (b) 2 (c) 92 a) 276 b) 285 c) 283
(d) 62 (e) 102 d) 280 e) 302
467. √1764 + √9025 + √3844 = ? 484.. 35% of 460 + 80% of 685 = ?
(a) 200 (b) 201 (c) 196 a) 609 b) 629 c) 619
(d) 193 (e) 199 d) 759 e) 709
468. (30)2 - (33)2 - (14)2 = ? 485. (?)2 + 4 = 132 + 262
a) – 371 b) – 397 c) - 378 a) -16 b) 104 c) 29
d) – 385 e) – 372 d) -31 e) -76
? 3 24
3 3 3 486. of √216 = 𝑜𝑓 √4
469. √6859 + √1728 + √2197 = ? 2 ?

a) 44 b) 47 c) 50 a) 16 b) 28 c) 25
d) 46 e) 45 d) 7 e) 4

For Free Study Material & Quizzes visit : www.mathsbyarunsir.com


For any queries or suggestions email us @ mathsbyarunsir003@gmail.com or whatsapp @ 8881331466
487. 175 x 8 = ? - 310 x 5 4
504. × 9646 − 24% of 1900 + 1548 ÷ 1.5 = ?
a) 3030 b) 2990 c) 2920 13
a) 3500 b) 3544 c) 3600
d) 2950 e) 3010

d) 4000 e) 4500
488. 25 - 1000 + 1296¼ = ?
2

a) 615 b) 621 c) 633 505. √115600 + 14 × 102 − 1312 + 11 ÷ 2.75 = ?


d) 603 e) 639 a) 400 b) 420 c) 460
489. (80% of 1400 +70% of 2800) = ? d) 500 e) 560
3
a) 3080 b) 3060 c) 2980 506. 158 × √1331 − 500 × 3.5 + 1131 + 13 × 17 =
d) 3140 e) None of these ?
2 2
490. (12 × 2 – 16 ) = ? a) 1340 b) 1300 c) 1500
a) 25 b) 32 c) 34 d) 1200 e) 1000
d) 22 e) 38 507. 1225 ÷ 2.5 − 1500% of 122 + 3160 + 121 = ?
491. 232 + (242 ÷ 16) + 40% of 600 = ? a) 1000 b) 1611 c) 1800
a) 780 b) 805 c) 795 d) 1455 e) 1708
d) 820 e) None of these 4 3 1
508. 17 + 1 51 − 6 + 343 × 7 = ?
2
492. (20% of 35 ) + 186 ÷ 6 – 27 × 6 = ?
a) 128 b) 114 c) 116 5 12 5
d) 103 e) 106 a) 4817 b) 4817 c) 4717
493. (16 × 9 of 2 + 152) = ? d) 4717
12 4
e) 48 17
a) 513 b) 565 c) 547
509. 29 + 17 – 18 = ? – 124 + 12
d) 429 e) 501
4
a) 140 b) 130 c) 135
494. ? × (7.5 + √256) + 102 = 132 d) 120 e) 145
a) 10 b) 2 c) 8 510. 11 19 % of 990 +15 – 25 (? )3 − 17 − 8
d) 4 e) 6 a) 4 b) 3 c) 6
495. ? % of 250 + 122 = 80% of 230 d) 5 e) 7
a) 12 b) 16 c) 14 511. 21 × 17 + 11 × 13 + (10)2 = (? )2 ÷ 6
d) 18 e) 20 a) 62 b) 60 c) 55
496. 4√62.5% 𝑜𝑓 284 + 12.5% 𝑜𝑓 628 = 2? d) 50 e) 54
a) 2 b) 1 c) 3 512. 9801 + 1137 – 1898 + 2346 = ? + 2446
d) 4 e) 5 a) 8940 b) 8848 c) 8954
497. 84%of ? + 123 = 145 × 18 d) 8972 e) 8936
a) 1200 b) 1000 c) 1250 513. 21 13 × 3 + 28 47 % 𝑜𝑓 525 + 3√1331 = (? )2
d) 1050 e) 950 a) 13 b) 14 c) 16
896
498. ? + 21% of 6400 = 382 − 44 d) 17 e) 15
514. 36 × 15 – 56 × 784 ÷ 112 =?
a) 8 b) 18 c) 24
a) 138 b) 238 c) 158
d) 12 e) 16
d) 258 e) 148
499. (? )% of 350 – (19)2 + 28 × 45 = 1032 515. 28.314 – 31.427 + 113.928 =? + 29.114
a) 48 b) 38 c) 26 (a) 81.711 (b) 80.701 (c) 71.711
d) 44 e) 56 (d) 81.701 (e) 80.107
500. (? )2 − 31 × 7 = √1521 + 12 × 35 516. 6573 ÷ 21 × (0.2)² =?
a) 26 b) 38 c) 21 (a) 18.25 (b) 22.6 (c) 15.65
d) 24 e) 18 (d) 12.52 (e) 12.89
501. 46% of 1500+36% of 750=?+28×15 517. 475 + 64% of 950 = 900 +?
a) 570 b) 500 c) 450 (a) 183 (b) 233 (c) 198
d) 540 e) 640 (d) 186 (e) 283
5
502. ? × 13 = 26 6 × 6 23
18 518. (0.064) × (0.4)7 = (0.4)? × (0.0256)²
(a) 7 (b) 2 (c) 8
a) 20 b) 18 c) 21
(d) 3 (e) 4
d) 14 e) 15 1 3
503. (?+26)% of 1200+375÷75×4=500 519. 13 × 3237 + 14 × 5362 + 200% 𝑜𝑓 1 =? + 1335
a) 18 b) 12 c) 14 (a) 72 (b) 70 (c) 68
d) 16 e) 10 (d) 65 (e) 85

For Free Study Material & Quizzes visit : www.mathsbyarunsir.com


For any queries or suggestions email us @ mathsbyarunsir003@gmail.com or whatsapp @ 8881331466
520. 33 ÷ 37 × 27 × 11.25 + 75% of 45 =? 536. 12.28 × 1.5 – 36 ÷ 2.4 = ?
(a) 37.5 (b) 3.75 (c) 375 (a) 3.24 (b) 7.325 (c) 6.42
(d) 35.7 (e) 32.5 (d) 4.32 (e) 3.42
521. 36.5% of 140 ÷ 12.5% of 80 =? 537. 56% of 958 + 67% of 1008 = ?% of 2000
(a) 6.12 (b) 4.71 (c) 5.11 (a) 60.592 (b) 47.622 (c) 42.86
(d) 5.91 (e) 8.11 (d) 91.455 (e) 49.592
5 4 538. 515.15 – 15.51 – 1.51 – 5.11 – 1.11 =?
522. of 616 × 12 ÷ 8+? = 13 × 21 + 71 + 3 ×? (a) 491.91 (b) 419.91 (c) 499.19
8
(a) 7005 (b) 7.005 (c) 70.05 (d) 411.19 (e) 391.91
(d) 700.5 (e) 600.5 4 1

523. 3.5% of 40 + 3.5% of 80 =? % of 10 539. 16 5 % of 3500 ÷ √784 = (? )3


(a) 49 (b) 56 (c) 64 (a) 9,571 (b) 9,921 (c) 9,252
(d) 66 (e) 42 (d) 9,261 (e) 9442
3 540. 225% of 5468 – 87% of 3700 =?
524. √1225 ÷ √343 × 45% of 760 = ?
(a) 9,218 (b) 9,368 (c) 9,084
(a) 1170 (b) 1710 (c) 1510
(d) 9,628 (e) 9448
(d) 1700 (e) 1720
541. 1532+ 857 + 496 – ?= 2005
525. 175% of 460 + 110% of 170 + 2? = 1000 (a) 880 (b) 870 (c) 980
(a) 3 (b) 4 (c) 5 (d) 788 (e) 780
(d) 2 (e) 1 542. 19.2 × 6 + 8.9 – 13.3 =? + 37.9
526. 18 × 3 × 60.1 ÷ (34 × 64 ) = 18?
7.9 0.1
(a) 75.7 (b) 81.6 (c) 74.8
(a) 1 (b) 2 (c) 3 (d) 72.9 (e) 85.2
(d) 4 (e) 5 67
543. 3 % of 2100 + 32% of 350 =? – 207
1 3 1 3 18 35
527. 3 + 2 + 7 − 5 − = (a) 878 (b) (b)788 (c) 877
7 5 5 7 35 ?
(a) 3 (b) 5 (c) 7 (d) 792 (e) 787
(d) 9 (e) 11 544. 40% of 265 + 35% of 180 = 50% of ?
528. 36% 𝑜𝑓 245 − 40% 𝑜𝑓 10 = 10− ? a) 338 b) 184 c) 253
(a) 84.2 (b) 6.8 (c) 74.2 d) 169 e) 181
(d) 75.6 (e) -74.2 545. 460 × 15 – 5 × 20 = ?
529. 24.424 + 5.656 + 1.131 + 0.089 = ? (a) 6500 (b) 4600 (c) 6200
(a) 31.003 (b) 31.3 (c) 31.03 (d) 7000 (e) 6800
(d) 31.0003 (e) 3.1 546. 5163 – 4018 + 3209 = ?
530. 21? × 216.5 = 2112.4 (a) 4174 (b) 4264 (c) 4804
(a) 18.9 (b) 4.4 (c) 5.9 (d) 4354 (e) 4345
1 1
(d) 16.4 (e) 6.9 547. 4 5 × 3 3 + ? = 20% of 120
531. 12.25 × ? × 21.6 = 3545.64 (a) 20 (b) 10 (c) 5
(a) 14.8 (b) 12.6 (c) 15.8 (d) 15 (e) 8
(d) 13.4 (e) 11.4 548. 4848 ÷ 24 × 11 – 222 = ?
532. 15.5% of 646 + 24.5% of 298 = ? (a) 3200 (b) 2444 (c) 2000
(a) 184.22 (b) 173.14 (c) 168.26 (d) 2500 (e) 2440
(d) 137.41 (e) 183.14 2 1 1
1 1 1 1 2  ? 2
533. 2 3 + 1 5 + 2 4 =? 549. 5 7 2

a) 8 60
23 53
b) 4 60
47
c) 5 60 3 4 3
(a) 3 (b) 3 (c) 2
37 47 70 35 70
d) 6 60 e) 360 17 3
(d) 1 (e) 1
70 70
534. 534.596 + 61.472 – 496.708 = ? + 27.271
550. 3136  625  ?2
(a) 126.631 (b) 72.089 (c) 62.069
(d) 132.788 (e) 82.089 (a) 8 (b) 11 (c) 9
535. 16 × 12 – 672 ÷ 21 = ? – 211 (d) 7 (e) 15
(a) 381 (b) 347 (c) 371 551. 53457 + 19743 – 49850 =?
(d) 311 (e) 391 (a) 24,350 (b) 23,350 (c) 25,330
(d) 23,550 (e) 23,840

For Free Study Material & Quizzes visit : www.mathsbyarunsir.com


For any queries or suggestions email us @ mathsbyarunsir003@gmail.com or whatsapp @ 8881331466
552. 13.57 + 29.49 + 23.46 =? + 50.79 (d) 8 (e) None of these
(a) 16.73 (b) 13.73 (c) 12.73 1 5 7
569. 4 2 − 2 6 =? −1 12
(d) 15.73 (e) 17.37 1 5 7
553. 1331 × 121 × 0.11 = (1.1)? × 11000 a) 3 4 b) 3 12 c) 2 12
(a) 5 (b) 7 (c) 4 3
d) 3 4 e) None of these
(d) 3 (e) 8 570. 36% of 245 – 40% of 210 = 10 – ?
554. 9.6 ÷ 0.24 × 2.5 + 150 =? (a) 4.2 (b) 6.8 (c) 4.9
(a) 255 (b) 250 (c) 260 (d) 5.6 (e) None of these
(d) 265 (e) 245 571. 40% of 265 + 35% of 180 = 50% of ?
555. ? % of 325  26 (a) 338 (b) 84.5 (c) 253.5
(a) 211 (b) 210 (c) 208 (d) 169 (e) None of these
(d) 212 (e) 216 572. 460 × 15 – 5 × 20 = ?
556. 999 ÷ 3000 + 8888 ÷ 4400 =? (a) 92000 (b) 4600 (c) 137800
(a) 2.353 (b) 23.53 (c) 0.2353 (d) 7000 (e) None of these
(d) 235.3 (e) 3.253 573. 5163 – 4018 + 3209 = ?
557. 21.16  6.25  ?  10.24 (a) 4174 (b) 4264 (c) 4804
(a) 3.8 (b) 8.3 (c) 9.6 (d) 4354 e) None of these
1 1
(d) 7.6 (e) 6.8 574. 4 5 × 3 3 + ? = 20% of 120
1
1331 11 (a) 10 15 (b) 10 (c) 5
0
558. ?3 – ? (d) 15 (e) None of these
(a) 8 (b) 9 (c) 14 575. 4848 ÷ 24 × 11 – 222 = ?
(d) 12 (e) 11 (a) 200 (b) 2444 (c) 2000
3
559. 233% of 30 + 153% of 70 – 87% of 200 =? (d) 115 8 (e) None of these
(a) 8 (b) 6 (c) 4 7 2 2
576. 77 9% of 3456 + 66 3% of 1881 – 16 3% of 12354=?
(d) 5 (e) 3
(a) 1838 (b) 1883 (c) 1683
560. 5.6 × 2.8 + 6.3 × 0.9 – 2.5 × 1.5 =?
(a) 15.6 (b) 16.7 (c) 19.4 (d) 202 (e) 1388
(d) 17.6 (e) 1.76 577. 5√108% of 75 + 32 × 5 + 2 + 45% of 460 =?
561. 115% of 4880 – 85% of 1640 =? (a) 410 (b) 120 (c) 210
(a) 4218 (b) 4368 (c) 4448 (d) 310 (e) 190
3 2
(d) 4628 (e) 4828 578. 4 5 of 6 3 of 360 – 32% of 950 =?
562. 3251+ 587 + 369 –? = 3007 (a) 12215 (b) 11736 (c) 9736
(a) 1250 (b) 1300 (c) 1375 (d) 10736 (e) 13376
(d) 1400 (e) 1200 579. 981 ÷ 3 × 5 + 105 – 54 % of 1645 =?
3
563. √9261 × √7056 = ? +89 (a) 851.7 (b) 85.17 (c) 8.517
(a) 1475 (b) 1875 (c) 1680 (d) 8517.7 (e) 751.7
3
(d) 1675 (e) 1275 580. 5 of? = 48% of 550 + 36% of 750 – 40% of?
564. 18.6 × 3 + 7.2 – 16.5 =? + 21.7 (a) 438 (b) 544 (c) 534
(a) 35.7 (b) 21.6 (c) 24.8 (d) 435 (e) 634
(d) 27.6 (e) 23.6 581. (2 × 3)3 ÷ (4 × 9)2 × (27 × 8)2 = (6)?
565. 5.35 + 4.43 + 0.45 + 45.34 + 534 =? (a) 5 (b) 6 (c) 3
(a) 597.27 (b) 589.57 (c) 596.87 (d) 8 (e) 7
(d) 596.67 (e) 696.67
582. 454.58 − 376.89 + 121.45 − 95.42 = ?
566. √360 − 225 × 2 + 379 (a) 102.22 (b) 103.72 (c) 91.72
(a) 17 (b) 19 (c) 279 (d) 92.32 (e) 104.42
(d) 289 (e) None of these 583. √576 ÷ (4)2 × 7.4 + (7)3 − 231 = ?
567. 9 × 81 ÷ 273 = (3)?
3 2
(a) 123.9 (b) 121.1 (c) 111.4
(a) 3 (b) 4 (c) 5 (d) 122.1 (e) 123.1
(d) 6 (e) None of these 584. [(84) 2
÷ 28 × 12] ÷ 24 = 7 ×?
568. 572 ÷ 26 × 12 – 200 = (2)? (a) 15 (b) 17 (c) 18
(a) 5 (b) 6 (c) 7 (d) 21 (e) 24
For Free Study Material & Quizzes visit : www.mathsbyarunsir.com
For any queries or suggestions email us @ mathsbyarunsir003@gmail.com or whatsapp @ 8881331466
585. (7.9% of 134) – (3.4% of 79) = ? 11
601. 79% of 500 + 113% of 700 = ?2 + 13 of 1170
(a) 8.1 (b) 7.9 (c) 8.6
a) 14 b) 15 c) 13
(d) 7.3 (e) 6.8
2 5 4 6
d) 11 e) 16
586. 3 5 of 5 17 + 5 13 × 2 23 =? 602. 65% 𝑜𝑓 240+ ? % 𝑜𝑓 150 = 210
(a) 25 (b) 30 (c) 40 (a) 45 (b) 46 (c) 32
(d) 20 (e) 15 (d) 36 (e) 38
400 300 2 2
587. √81% of 3
+ 78% of 2
=? 603. 3 𝑜𝑓 1 5 of 75% of 540 = ?
(a) 17 (b) 18 (c) 15 (a) 378 (b) 756 (c) 252
(d) 13 (e) 19 (d) 332 (e) 368
? 3 1 5 7
588. ? % of 300 + 45% of 440 = 7 of 63 – 17 of 578 604. 4 2 − 2 6 = ? −1 12
(a) 50 (b) 55 (c) 60 1 5 7
(a) 3 (b) 3 (c) 2
(d) 45 (e) 40 4 12 12
3 1
(d) 3 (e) 4
589. √2401 + √1024 = ?2 4 4
(a) 7 (b) 9 (c) 8 605. 99.9 + 9.99 + 0.99 + 99.09 + 9.09 + 999 = ?
(d) 6 (e) 11 (a) 1123.06 (b) 1218.06 (c) 1345.06
590. 37750 ÷ 50 + 41455 ÷ 5 + 27540 ÷ 20 = ? (d) 1440.06 (e) 1216.06
(a) 10652 (b) 10523 (c) 10423 606. ? = 4200 + 3/8 of 480
(d) 11020 (e) 9985 (a) 4390 (b) 4480 (c) 4380
591. 63% of 210 + 47% of 310 – 30% of 175 = ? (d) 4480 (e) 4580
(a) 224 (b) 225.5 (c) 220 607. 580 + 24 × 0.25 − 3 = ?
(d) 227.5 (e) 228.5 (a) 580 (b) 586 (c) 583
3 (d) 593 (e) 563
592. ? of 7 – 42% of 250 = 315
(a) 970 (b) 1000 (c) 980 608. 75% 𝑜𝑓 1228 + 45% 𝑜𝑓 400 = ?
(d) 985 (e) 975 (a) 1201 (b) 1101 (c) 1301
7 102 (d) 1501 (e) 1001
593. 17 of 133 of 304 = ?
609. 1520 + 18420 + 1680 ÷ 80 =?
(a) 92 (b) 88 (c) 94
(a) 19951 (b) 19971 (c) 11981
(d) 96 (e) 98
(d) 11941 (e) 19961
594. 341.46 + 273.43 + 583.11 – 881 = ?
(a) 321 (b) 317 (c) 319 610. ? % 𝑜𝑓 6300 = 225 − 44% 𝑜𝑓 225
(d) 313 (e) 314 (a) 4 (b) 5 (c) 2
1 (d) 3 (e) 6
595. √? – 423 ÷ 9 × 13 + 11% of 5000 = 3 of 57 3
611. √5832 × √361 = 18% 𝑜𝑓 190 × ?
(a) 8100 (b) 3600 (c) 6400
(a) 12 (b) 10 (c) 8
(d) 6436 (e) 6600
(d) 9 (e) 15
596. (0.7) × (4.9) × (0.343) = (0.7)? × 10
(a) 6 (b) 7 (c) 8 612. ? % 𝑜𝑓 50 + 25% 𝑜𝑓 444 = 202
(d) 4 (e) 5 (a) 182 (b) 122 (c) 142
597. 9 78 + 5 16
7 5
– 4 24 = ?
(d) 162 (e) 172
5
(a) 9 48 (b) 17 41
2
(c) 11 24
5 613. (0.064) ÷ (0.16)3 × (0.0256) = (0.4)?+1
7 5 (a) 0 (b) 1 (c) –1
(d) 13 48 (e) 11 48
1 2
598. 33 3 % of 243 + 66 3% of 345 =? 614. 1425 + 8560 + 1680 ÷ 200 = ?
(a) 315 (b) 311 (c) 309 (a) 58.325 (b) 9973.4 (c) 56.425
(d) 317 (e) 320 (d) 9939.4 (e) 9993.4
6÷60+66÷6000+66÷600000 615. 75% of 1240 = 35% of 1560 + ?
599. =?
2÷20+222÷20000+1 (a) 384 (b) 394 (c) 456
(a) 10 (b) 1 (c) 0.1 (d) 364 (e) 374
(d) 100 (e) 0.01
616. 555.05 + 55.50 + 5.55 + 5 + 0.55 = ?
600. 9 − 7 + 67 = x + 43
2 2
a) 621.65 b) 655.75 c) 634.85
a. 43 b. 39 c. 33 d) 647.35 e) 631.65
d. 47 e. 35

For Free Study Material & Quizzes visit : www.mathsbyarunsir.com


For any queries or suggestions email us @ mathsbyarunsir003@gmail.com or whatsapp @ 8881331466
617. 2151.46 + 5437.54 – 6795 =? (d) 25 (e) 32
(a) 974 (b) 794 (c) 796 634. 18 × 3 × 60.1 ÷ (34 × 64 ) = 18?
7.9 0.1
(d) 790 (e) 792 (a) 1 (b) 2 (c) 3
2 3 4
618. 5 of 215 + 4 of 128 – 7 of 147 =? (d) 4 (e) 5
1 3 1 3 18 35
(a) 94 (b) 96 (c) 98 635. 3 + 2 + 7 − 5 − =
7 5 5 7 35 ?
(d) 92 (e) 100 (a) 3 (b) 5 (c) 7
619. 56% of 700 + 64% of 900 – 40% of 290 =? (d) 9 (e) 11
(a) 848 (b) 852 (c) 850 636. 36% 𝑜𝑓 245 − 40% 𝑜𝑓 210 = 10− ?
(d) 854 (e) 846 (a) 4.2 (b) 6.8 (c) 4.9
620. 7777 ÷ 11 + 888 ÷ 6 =? (d) 5.6 (e) 5.8
(a) 855 (b) 857 (c) 853 637. 39% of 450 + 51% of 340 – 40% of 255 =?
(d) 850 (e) 852
3 3
(a) 264.9 (b) 246.9 (c) 236.9
621. √1331 × 11 % of 14300 =? (d) 242.9 (e) 251.7
638. 3 5 + 4 7 + 5 10 =? +7 27
2 5 7
(a) 426 (b) 427 (c) 431
(d) 429 (e) 432
47 47 37
622. √?× √3025 = 2695 a) 9 20 b) 7 70 c) 6 70
43 41
(a) 2401 (b) 2209 (c) 2601 d) 11
69
e) 5
70
(d) 2304 (e) 2400 ?
639. 67% of 700 + 73% of 510 – 52% of 220 = 10
623. (98)2 + (? ) = (150)2 − (80)2 − 737 (a) 6369 (b) 6269 (c) 7629
(a) 6084 (b) 5759 (c) 5777 (d) 7269 (e) 7379
(d) 6724 (e) 5658 640. 3564 + 3486 ÷ 10 + 57434 ÷ 100 – 67385 ÷ 100 =?
624. 48 + 8 × 0.75 – 5 =? (a) 3813.09 (b) 3138.09 (c) 3613.09
(a) 22 (b) 36 (c) 49 (d) 3713.09 (e) 3513.09
(d) 56 (e) 46
641. 5 of 4 of 17 of 6 of 544 = 7 37
? 3 15 13
625. 18.657 – 7.549 – 4.111 – 1.630 =? 2 8 1
(a) 4.673 (b) 6.893 (c) 6.562 a) 21 b) 21 c) 21
1 5
(d) 5.367 (e) 6.367 d) 23 e) 23
626. 2950 ÷ 12.5 + 160 =? 642. 346.64 ÷ 4 + 440 ÷ 2.2 + 46% of 550 =?
(a) 392 (b) 390 (c) 396 (a) 544.66 (b) 539.66 (c) 439.66
(d) 394 (e) 400 (d) 343.66 (e) 493.66
627. 999 ÷ 3000 + 8888 ÷ 4400 =? 643. √(40% of 640 + 208.6 + 264.4) + 56% of 900 = 45 of ?
3

(a) 2.353 (b) 23.53 (c) 0.2353 (a) 441.25 (b) 541.25 (c) 645.25
(d) 235.3 (e) 3.253 (d) 641.25 (e) 741.25
628. 21.16  6.25  ?  10.24 4
644. 3 of 5 of 2 = 55
3 ? 5
7 5 6 9
(a) 3.8 (b) 8.3 (c) 9.6 2 2 2
(d) 7.6 (e) 6.8 a) 4 3 b) 6 3c) 8 3
2 1
1331 11 d) 7 3e) 4 3
3
629. ? – 0 3
? 645. 49 × 546 ÷ 6 + 243 – 534 =? – 5 of 465
(a) 8 (b) 9 (c) 14 (a) 4474 (b) 4847 (c) 4347
(d) 12 (e) 11 (d) 4664 (e) 4447
630. 233% of 30 + 153% of 70 – 87% of 200 =? 646. 5555 ÷ 11+ 6666 ÷ 33 + 888888 ÷ 220 =? – 1.121 × 4
(a) 8 (b) 6 (c) 4 (a) 4651.884 (b) 4571.884 (c) 4751.884
(d) 5 (e) 3 (d) 4951.864 (e) 4451.488
631. 5.6 × 2.8 + 6.3 × 0.9 – 2.5 × 1.5 =? 647. 7² + 34 – 4³ =? – 11²
(a) 15.6 (b) 16.7 (c) 19.4 (a) 55 (b) 196 (c) 172
(d) 17.6 (e) 1.76 (d) 187 (e) 178
632. 56% of 225 + 20% of 150 = ? – 109 648. 1 34 + 2 23 + 2 12 = ?
(a) 49 (b) 103 (c) 53
(d) 47 (e) 265 7 11 1
a) 9 12 b) 6 12 c) 3 12
633. 68% of 625 + ? % of 185 = 499
5 5
(a) 42 (b) 40 (c) 28 d) 4
12
e) 712

For Free Study Material & Quizzes visit : www.mathsbyarunsir.com


For any queries or suggestions email us @ mathsbyarunsir003@gmail.com or whatsapp @ 8881331466
6 17 27 21
649. 35% of 42% of 7th of 500 =? (a) (b) 56 (c) 42 23
8
(a) 36 (b) 44 (c) 52 2 23
(d) 18 3 (e) 1 36
(d) 60 (e) 63
650. 8888 + 848 + 88 - ? = 7337 + 737 666. √52 × 14 − 6 × 7 + (4)? = 18
(a) 1750 (b) 1650 (c) 1550 (a). 1 (b) 3 (c) 4
(d) 1450 (e) 1250 (d) 5 (e) 2
651. 414? × 7 = 127512 667. (𝟐 × 𝟑)𝟑 ÷ (𝟒 × 𝟗)𝟐 × (𝟐𝟕 × 𝟖)𝟐 = (𝟔)?
(a) 36 (b) 40 (c) 44 (a) 5 (b) 6 (c) 3
(d) 48 (e) 52 (d) 8 (e) 7
652. 3.2% of 500 × 2. 4% of ? = 288 668. 454.58 − 376.89 + 121.45 − 95.42 = ?
(a) 650 (b) 700 (c) 600 (a) 102.22 (b) 103.72 (c) 91.72
(d) 750 (e) 850 (d) 92.32 (e) 104.42
653. (– 251 × 21 × (– 12)) ÷ ? = 158.13 669. √𝟓𝟕𝟔 ÷ (𝟒)𝟐 × 𝟕. 𝟒 + (𝟕)𝟑 − 𝟐𝟑𝟏 = ?
(a) 250 (b) 400 (c) 300 (a) 123.9 (b) 121.1 (c) 111.4
(d) 15 (e) 18 (d) 122.1 (e) 123.1
2
654. [(130) ÷ 25 × 15] ÷ 30 = ? 670. [(𝟖𝟒)𝟐 ÷ 𝟐𝟖 × 𝟏𝟐] ÷ 𝟐𝟒 = 𝟕 ×?
(a) 352 (b) 314 (c) 326 (a) 15 (b) 17 (c) 18
(d) 338 (e) 426 (d) 21 (e) 24
655. √√4900 + √5476= ? 671. (7.9% of 134) – (3.4% of 79) =?
(a) 576 (b) 144 (c) 256 (a) 8.1 (b) 7.9 (c) 8.6
(d) 16 (e) 12 (d) 7.3 (e) 6.8
656. (6.5% of 375) – (0.85% of 230) =? 672. (252 ÷ 21 × 12) ÷ ? = 16
(a) 23.42 (b) 24.24 (c) 21.64 (a) 11 (b) 9 (c) 8
(d) 25.76 (e) 22.42 (d) 13 (e) 17
657. 13.141 + 31.417 – 27.118 =? 673. 36865 + 12473 + 21045 – 44102 =?
(a) 16.441 (b) 17.543 (c) 17.490 (a) 114485 (b) 28081 (c) 26281
(d) 17.440 (e) 17.590 (d)114845 (e) 79821
658. If √80656 = 284, then √8.0656 =? 674. (21.6)² ÷ (– 7.2)² × ? = 15483.36 – 15276.09
(a) 28.4 (b) 2.84 (c) 0.0284 (a) 23.03 (b) 23.3 (c) 32.03
(d) 0.284 (e) 4.084 (d) 32.3 (e) 42.3
𝑥 675. 348 ÷ 29 × 15 + 156 = (?)³ + 120
659. If 𝑥 = 3.2 and 𝑦 = 4, what is the value = 5𝑥 + 4𝑦 + 3 (𝑦) ? (a) 12 (b) 6 (c) 36
(a) 35.2 (b) 34.4 (c) 32.8 (d) 9 (e) 13
𝟐
(d) 18.4 (e) 28.4 676. (𝟐√𝟑𝟗𝟐 – 𝟐𝟏) + (√𝟖– 𝟕) = (? )𝟐
660. ?2 + (14)2 × 18 ÷ 6 − 1029 = 80 × (12 − 7) (a) 6 (b) 4 (c) 12
(a) 25 (b) 841 (c) 729 (d) 2 (e) 8
(d) 27 (e) 29 677. 46% of 450 +? % of 19.07 = 359.56
? √162 (a) 795 (b) 805 (c) 815
661. =
√128 ? (d) 800 (e) 820
(a) 12 (b) 144 (c) 14 4
(d) 196 (e) 28 678. (? ) ÷ 27 = 243
(a) 8 (b) 12 (c) 9
662. [(5√5 + √5) × (4√5 + 8√5)] − (19)2 =? (d) 11 (e) 15
(a) – 1 (b) 1√5 (c) 1 2 3 1 1
679. 2 3 + 3 7 × 4 6 ÷ 7 7 =?
(d) −1√5 (e) 11 17 2 1
(a) (b) 4 3 (c) 4 3
8
663. (4444 ÷ 40) + (645 ÷ 25) + (3991 ÷ 26) =? 2 1
(a) 280.4 (b) 290.4 (c) 295.4 (d) 8 3 (e) 3 4
(d) 285.4 (e) 258.5 680. 0.03 × 0.01 – 0.003 ÷ 100 + 0.03 =?
3
664. (? ) ÷ 32 = 54 (a) 0.03027 (b) 0.0327 (c) 0.3027
a) 318 b) 12 c) 14 (d) 1.03027 (e) 0.003027
d) 16 e) 24 681. 17 × 25 – 240 +? ÷ 41 = 354
1 5 5 1 (a) 6299 (b) 6929 (c) 9629
665. 1 4 + 1 9 × 1 8 ÷ 6 2 =? (d) 7129 (e) 6992
For Free Study Material & Quizzes visit : www.mathsbyarunsir.com
For any queries or suggestions email us @ mathsbyarunsir003@gmail.com or whatsapp @ 8881331466
682. [(𝟏𝟗𝟐)𝟐 ÷ 𝟔𝟒 × 𝟐𝟒] ÷ 𝟒𝟖 = √? (a) 49,742 (b) 47,942 (c) 44,942
(a) 83000 (b) 82944 (c) 82954 (d) 47,429 (e)49,427
(d) 82950 (e) 82590 698. ? ÷ 13 + 2262 ÷ 39 × 243 + 273 ÷ 3 =
𝟐 𝟐 𝟐 𝟐
683. 30% of 𝟕 of 𝟗 of 𝟓 of 𝟑 of 9450 =? 5 × (1481 + 1376)
(a) 1250 (b) 1100 (c) 1300
(a) 32 (b) 36 (c) 42
(d) 1400 (e) 1350
(d) 48 (e) 52
−𝟐 −𝟐 699. 3.4 × 2.6 × 5.4 ÷ 1.8 ÷ 0.17 =? ÷ 0.8
𝟏 𝟓 𝟏 𝟑
684. (𝟏𝟎𝟐𝟒) + (𝟑𝟒𝟑) = ? × 𝟓 (a) 12.48 (b) 124.8 (c) 1248.8
(a) 65 (b) 42 (c) 13 (d) 1.248 (e) 12.84
(d) 21 (e) 27 700. √
𝟒𝟎𝟎 𝟏𝟔
% 𝐨𝐟 𝟏𝟑𝟕𝟐 + 𝟏𝟕 𝐨𝐟 𝟐𝟒𝟗𝟗 = ?
685. 3.5% of 40 + 3.5% of 80 =? % of 10 𝟕
(a) 30 (b) 32 (c) 36
(d) 40 (e) 42 (a) 58 (b) 54 (c) 56
𝟕 𝟓 𝟏 𝟏 (d) 52 (e) 48
686. 𝟏 𝟗 + 𝟐 𝟑 + 𝟑 𝟗 − 𝟒 𝟓 = ?
701. 1701 ÷ 2100 + 3087 ÷ 6300 + 832 ÷ 5200 =?
13 17 16
(a) 2 45 (b) 3 45 (c) 4 45 (a) 1.0019 (b) 101.9 (c) 0.1019
17 13 (d) 10.19 (e) 1.46
(d) 4 (e) 4
45 45 702. 185% 𝑜𝑓 400 + 35% 𝑜𝑓 240 =? % 𝑜𝑓 1648
687. 7960 + 2956 − 8050 + 4028 = ? (a) 85 (b) 75 (c) 125
(a) 6984 (b) 6884 (c) 6894 (d) 50 (e) 55
(d) 6954 (e) 7894
703. √𝟐𝟒𝟒 + 𝟐𝟐𝟒 = ? × 𝟐𝟎²
688. 25 × 3.25 + 50.4 ÷ 24 = ?
(a) 20 (b) 4 (c) 2
(a) 84.50 (b) 83.35 (c) 83.53
(d) 16 (e) 24
(d) 82.45 (e) 92.84
704. 2.8 × 1.5 + 8% of 250 =?
689. 350% 𝑜𝑓 ? ÷ 50 + 248 = 591 (a) 24.2 (b) 24.02 (c) 242.2
(a) 4900 (b) 4890 (c) 4850 (d) 2.42 (e) 22.24
(d) 4950 (e) 4750 705. 160% of 250 +? = 120% of 400
1
690. 2 𝑜𝑓 3842 + 15% 𝑜𝑓 ? = 2449 (a) 160 (b) 40 (c) 80
(a) 3520 (b) 3250 (c) 3350 (d) 120 (e) 140
(d) 3540 (e) 2850 706. 65% of 780 + 78% of 650 =?
691. 833.25 − 384.45) ÷ 24 = ?
( (a) 507 (b) 1014 (c) 1011
(a) 1.87 (b)20.1 (c) 2.01 (d) 817 (e) 705
(d) 18.7 (e) 16.7 707. 5431 + 4687 – 8462 =?
(a) 1565 (b) 1656 (c) 1645
692. (252 ÷ 21 × 12) ÷ ? = 16
(d) 1675 (e) None of these
(a) 11 (b) 9 (c) 8
708. 31% of 500+ 41% of 1100 – 50% of 790 = ?
(d) 13 (e) 17
(a) 230 (b) 221 (c) 211
693. 36865 + 12473 + 21045 – 44102 =? (d) 255 (e) None of these
a) 114485 b) 28081 c) 26281 709. 1107 ÷ 9 × 6 + 236 – 874 = ?
d) 114845 e) 79821 (a) 110 (b) 105 (c) 100
694. (21.6)² ÷ (– 7.2)² × ? = (d) 120 (e) 112
3 4 18
15483.36 – 15276.09 710. 4 𝑜𝑓 27 𝑜𝑓 11 𝑜𝑓 2673 = ?
(a) 23.03 (b) 23.3 (c) 32.03 (a) 470 (b) 450 (c) 465
(d) 32.3 (e) 42.3 (d) 486 (e) None of these
695. 348 ÷ 29 × 15 + 156 = (? )³ + 120 4 3 7 3
711. 17 𝑜𝑓 √4913 + 15 𝑜𝑓 √3375 = ?
(a) 12 (b) 6 (c) 36
(a) 13 (b) 11 (c) 9
(d) 9 (e) 13
𝟐
(d) 15 (e) None of these
696. (𝟐√𝟑𝟗𝟐 – 𝟐𝟏) + (√𝟖– 𝟕) = (? )𝟐 3
712. 8 𝑜𝑓 168 × 15 ÷ 5+? = 549 ÷ 9 + 235
(a) 6 (b) 4 (c) 1 (a) 163 (b) 199 (c) 107
(d) 2 (e) 8 (d) 126 (e) 136
𝟒 𝟐 𝟐
697. 𝟓 𝒐𝒇 𝟕 𝒐𝒇 𝟑 𝒐𝒇 𝟒𝟗𝟎 =?
𝟕 𝟑

For Free Study Material & Quizzes visit : www.mathsbyarunsir.com


For any queries or suggestions email us @ mathsbyarunsir003@gmail.com or whatsapp @ 8881331466
1 (d) 9 (e)11
713. 11 × 34 + ? 𝑜𝑓 385 − 1698 ÷ 6 = 685
2 5 4 6
(a) 5 (b) 4 (c) 8 729. 3 5 𝑜𝑓 5 17 + 5 13 × 2 23 =?
(d) 6 (e) 9 (a) 25 (b) 30 (c) 40
2
714. (? ) − 364 ÷ 7 × 6 + 289 = 26 × (121 + 72) (d) 20 (e)15
(a) 95 (b) 89 (c) 83 730. ? % 𝑜𝑓 300 + 45% 𝑜𝑓 440 =
(d) 71 (e) None of these ? 3
𝑜𝑓 63 – 𝑜𝑓 578
1 2 7 17
715. 1 𝑜𝑓 1411 + 583 ×? = 16 % 𝑜𝑓 14490 (a) 50 (b) 55 (c) 60
3
(4913) 3
(a) 5 (b) 6 (c) 7 (d) 45 (e)40
(d) 8 (e) 4 731. √2401 + √1024 = ?2
2 1 (a) 7 (b) 9 (c) 8
716. 14 7 % 𝑜𝑓 27048 ÷ √576 = (? )2 (d) 6 (e) 11
(a) 25571 (b) 25921 (c) 25252 732. 63% 𝑜𝑓 210 + 47% 𝑜𝑓 310 – 30% 𝑜𝑓 175 = ?
(d) 25481 (e)24481 (a) 224 (b) 225.5 (c) 220
717. 668 ÷ 167 × 284 = ? (d) 227.5 (e) 228.5
(a) 1156 (b) 1136 (c) 1096 7 102
733. 17 𝑜𝑓 133 𝑜𝑓 304 = ?
(d) 1116 (e) 2116
(a) 92 (b) 88 (c) 94
718. 60% 𝑜𝑓 25% 𝑜𝑓 5/6 𝑜𝑓 ? = 630 (d) 96 (e) 98
(a) 5060 (b) 5200 (c) 4880
734. 341.46 + 273.43 + 583.11 – 881 = ?
(d) 4500 (e) 5040
(a) 321 (b) 317 (c) 319
719. (85410 + 36885 + 24705) ÷ 1600 =? (d) 313 (e) 314
(a) 90.25 (b) 94.386 (c) 95.50 1
735. √? – 423 ÷ 9 × 13 + 11% 𝑜𝑓 5000 = 3 𝑜𝑓 57
(d) 91.875 (e)85.275
(a) 8100 (b) 3600 (c) 6400
720. 4.5 + 23.50 + 14.58 − 17.68 × 0.5 =?
(d) 6436 (e) 6600
(a) 33.74 (b) 21.29 (c) 35.06 7 7 5
(d) 24.48 (e) 28.48 736. 9 8 + 5 16 – 4 24 = ?
5 2 5
721. (3675 ÷ 75) × (7480 ÷ 80) =? (a) 9 48 (b) 17 41 (c) 11 24
7 5
(a) 4394.5 (b) 4301 (c) 4581.5 (d) 13 48 (e) 11 48
(d) 4114 (e)3774 11
737. 79% of 500 + 113% of 700 = ?2 + 13 of 1170
722. 1 + 222 ÷ 20 + 4444 ÷ 400 =? (a) 14 (b) 15 (c) 13
(a) 25.21 (b) 23.21 (c) 29.23 (d) 11 (e) 16
(d) 31.23 (e) 33.21
𝟏 𝟏
738. 150% 𝑜𝑓 220 + 120% 𝑜𝑓 140+? = 850
723. 𝟏𝟐 + 𝒐𝒇 𝟗 ­ =? (a) 342 (b) 355 (c) 352
𝟑 𝟐
(d) 360 (e) 370
1 1 1
(𝑎) 15 2 (𝑏) 13 2 (𝑐) 14 2 739. 30% 𝑜𝑓 340 − 70% 𝑜𝑓 110 = 65− ?
3 1
(𝑑) 14 4 (𝑒) 16 2 (a) 30 (b) 50 (c) 45
724. 36% 𝑜𝑓 125 + 44% 𝑜𝑓 225 = ? (d) 40 (e) 55
(a) 144 (b) 152 (c) 140 740. [7 × 4.4 + 8.6 − (8)2 =?
( )]
(d) 136 (e) 126 (a) 27 (b) 30 (c) 23
725. 235.4 + 371.8 + 253.8 = ? (d) 35 (e) 33
(a) 851 (b) 881 (c) 861 2 1 1
1 2  ? 2
(d) 867 (e) 651 741. 5 7 2
726. ? % 𝑜𝑓 400 + 31% 𝑜𝑓 600 = 50% 𝑜𝑓 820 3 4 3
3 3 2
(a) 52 (b) 56 (c) 64 (a) 70 (b) 35 (c) 70
(d) 48 (e)36 17 3
727. 14.2% 𝑜𝑓 11000 + 15.6% 𝑜𝑓 ? = 3590 1 1
(d) 70 (e) 70
(a)12000 (b)13000 (c)14560
742. 64 + 60 ÷ (30 ÷ 3) = (?) ÷ 20
(d)12250 (e)13500
𝟏 𝟑 𝟏 𝟑 𝟏𝟖 𝟑𝟓
(a) 1440 (b) 1400 (c) 1600
728. 𝟑 𝟕 + 𝟐 𝟓 + 𝟕 𝟓 − 𝟓 𝟕 − 𝟑𝟓 = (d) 1540 (e) 1660
?
(a) 3 (b) 5 (c) 7

For Free Study Material & Quizzes visit : www.mathsbyarunsir.com


For any queries or suggestions email us @ mathsbyarunsir003@gmail.com or whatsapp @ 8881331466
743. 1251/3 × 271/3 × 1254/3 = 15 × 5? (a) 3 (b) 4 (c) 5
(a) 5 (b) 3 (c) 1 (d) 2 (e) 1
(d) 2 (e) 4 759. 187.9 × 30.1 × 60.1 ÷ (34 × 64 ) = 18?
744. 125% of 3060 = 408 + 85% of (?) (a) 1 (b) 2 (c) 3
(a) 4020 (b) 4010 (c) 4818 (d) 4 (e) 5
(d) 4221 (e) 4515 1 3 1 3 18
760. 3 7 + 2 5 + 7 5 − 5 7 − 35 = ?
35

745. [(37.5 × 19.2) ÷ 18 + 21.50 ÷ 4.3] = ? × 15 (a) 3 (b) 5 (c) 7


(a) 5 (b) 4 (c) 6 (d) 9 (e) 11
(d) 2 (e) 3
761. 36% 𝑜𝑓 245 − 40% 𝑜𝑓 10 = 10− ?
746. 338.40 ÷ 42.3 + 25% of 4200 ÷ 100 = ? + 246.5 ÷ 29
(a) 84.2 (b) 6.8 (c) 74.2
(a) 12 (b) 11 (c) 10
(d) - 75.6 (e) – 74.2
(d) 1 (e) 13 𝟏 𝟑
762. 𝟏𝟑 × 𝟑𝟐𝟑𝟕 + 𝟏𝟒 × 𝟓𝟑𝟔𝟐 + 𝟐𝟎𝟎% 𝒐𝒇 𝟏 =? + 𝟏𝟑𝟑𝟓
747. √𝟑𝟗𝟔𝟗 − √𝟐𝟐𝟎𝟗 + 𝟗 = 𝟑√𝟔𝟖𝟗𝟐𝟏−?
(a) 72 (b) 70 (c) 68
(a) 12 (b) 13 (c) 14
(d) 15 (e) 16 (d) 65 (e) 85
748. 𝟏𝟖𝟕.𝟗 × 𝟑𝟎.𝟏 × 𝟔𝟎.𝟏 ÷ (𝟑𝟒 × 𝟔𝟒 ) = 𝟏𝟖? 763. 𝟑𝟑 ÷ 𝟑𝟕 × 𝟐𝟕 × 𝟏𝟏. 𝟐𝟓 + 𝟕𝟓% 𝒐𝒇 𝟒𝟓 =?
(a) 1 (b) 2 (c) 3 (a) 37.5 (b) 3.75 (c) 375
(d) 4 (e) 5 (d) 35.7 (e) 32.5
1 3 764. 36.5% 𝑜𝑓 140 ÷ 12.5% 𝑜𝑓 80 =?
749. 13 × 3237 + 14 × 5362 + 200% 𝑜𝑓 1 =
(𝑎) 6.12 (𝑏) 4.71 (𝑐) 5.11
? + 1335 (𝑑) 5.91 (𝑒) 8.11
(a) 72 (b) 70 (c) 68
765. 𝟖 𝒐𝒇 𝟔𝟏𝟔 × 𝟏𝟐 ÷ 𝟖+? = 𝟏𝟑 × 𝟐𝟏 + 𝟕𝟏 + 𝟒𝟑 ×?
𝟓
(d) 65 (e) 85
750. 𝟑𝟑 ÷ 𝟑𝟕 × 𝟐𝟕𝟐 × 𝟏𝟏. 𝟐𝟓 + 𝟕𝟓% 𝒐𝒇 𝟒𝟓 =? (a) 7005 (b) 7.005 (c) 70.05
(d) 700.5 (e) 600.5
(a) 135 (b) 133 (c) 132
(d) 134 (e) 131 766. 3.5% 𝑜𝑓 40 + 3.5% 𝑜𝑓 80 =? % 𝑜𝑓 10
7 4 (a) 49 (b) 56 (c) 64
751. 8 𝑜𝑓 616 × 12 ÷ 16+? = 17 × 19 + 81 + 3 ×? (d) 66 (e) 42
3 2 4 767. 36 × 15 – 56 × 784 ÷ 112 =?
(𝑎) (𝑏) (𝑐) (a) 138 (b) 238 (c) 158
5 5 5
3 4
(𝑑) (𝑒) (d) 258 (e)148
4 3
752. [(𝟏𝟗𝟐)𝟐 ÷ 𝟔𝟒 × 𝟐𝟒] ÷ 𝟒𝟖 = √? 768. 28.314 – 31.427 + 113.928 =? + 29.114
(a) 83000 (b) 82944 (c) 82954 (a) 81.711 (b) 80.701 (c) 71.711
(d) 82950 (e)82590 (d) 81.701 (e) 80.107
𝟐 𝟐 𝟐 𝟐
753. 𝟑𝟎% 𝒐𝒇
𝟕
𝒐𝒇
𝟗
𝒐𝒇
𝟓
𝒐𝒇
𝟑
𝒐𝒇 𝟗𝟒𝟓𝟎 =? 769. 6573 ÷ 21 × (0.2)² =?
(a) 32 (b) 36 (c) 42 (a) 18.25 (b) 22.6 (c) 15.65
(d) 48 (e) 52 (d) 12.52 (e) 12.89
−𝟐 −𝟐
𝟏 𝟓 𝟏 𝟑 770. 475 + 64% 𝑜𝑓 950 = 900 +?
754. (𝟏𝟎𝟐𝟒 ) + (𝟑𝟒𝟑) = ? × 𝟓
(a) 183 (b) 233 (c) 198
(a) 65 (b) 42 (c) 13 (d) 186 (e) 283
(d) 21 (e) 27
755. 𝟑. 𝟓% 𝒐𝒇 𝟒𝟎 + 𝟑. 𝟓% 𝒐𝒇 𝟖𝟎 =? % 𝒐𝒇 𝟏𝟎 771. (0.064) × (0.4)7 = (0.4)? × (0.0256)²
(a) 7 (b) 2 (c) 8
(a) 30 (b) 32 (c) 36
(d) 3 (e) 4
(d) 40 (e) 42
𝟕 𝟓 𝟏 𝟏 772. 3.6 + 36.6 + 3.66 + 0.36 + 3.0 =?
756. 𝟏 𝟗 + 𝟐 𝟑 + 𝟑 𝟗 − 𝟒 𝟓 = ?
(a) 44.22 (b) 77.22 (c) 74.22
13 17 16
(𝑎) 2 45 (𝑏) 3 45 (𝑐) 4 45 (d) 47.22 (e) 57.22
17 13 773. 23 × 45 ÷ 15 =?
(𝑑) 4 (𝑒 )4
45 45
3
(a) 69 (b) 65 (c) 63
757. √1225 ÷ √343 × 45% 𝑜𝑓 760 = ? (d) 71 (e) 81
(a) 1170 (b) 1710 (c) 1510 𝟐𝟏𝟎 𝟏𝟕
(d) 1700 (e) 1720 774. × 𝟏𝟓 × ? = 𝟒𝟎𝟒𝟔
𝟏𝟒
(a) 202 (b) 218 (c) 233
758. 175% 𝑜𝑓 460 + 110% 𝑜𝑓 170 + 2? = 1000
(d) 227 (e) 238
For Free Study Material & Quizzes visit : www.mathsbyarunsir.com
For any queries or suggestions email us @ mathsbyarunsir003@gmail.com or whatsapp @ 8881331466
𝟓 𝟏 𝟖 (d) Can’t be determined (e)1.0404
775. 𝟒 𝟔 + 𝟕 𝟐 − 𝟓 𝟏𝟏 = ?
2
10 20 20 790. [(140) ÷ 70 × 16] ÷ 8 = 14 × ?
(𝑎) 2 23 (𝑏) 6 33 (𝑐) 2 33 (a) 38 (b) 22 (c) 55
10 20
(𝑑) 6 33 (𝑒 )8 (d) 40 (e) 36
33
776. 83% of 2350 = ? 791. 𝟕𝟕 𝟕𝟗 % 𝒐𝒇 𝟑𝟒𝟓𝟔 + 𝟔𝟔 𝟐𝟑 % 𝒐𝒇 𝟏𝟖𝟖𝟏 – 𝟏𝟔 𝟐𝟑 % 𝒐𝒇 𝟏𝟐𝟑𝟓𝟒 =?
(a) 1509.5 (b) 1950.5 (c) 1905.5 (a) 1838 (b) 1883 (c) 1683
(d) 1590.5 (e) 1850.5 (d) 2021 (e) 1388
777. [5 × 8 + 60 ÷ 4 – 45 + 2] = ? 792. 𝟓√𝟏𝟎𝟖% 𝒐𝒇 𝟕𝟓 + 𝟑𝟐 × 𝟓 + 𝟐 + 𝟒𝟓% 𝒐𝒇 𝟒𝟔𝟎 =
(a) 6 (b) 8 (c) 10 ?
(d) 12 (e) 16 (a) 410 (b) 120 (c) 210
778. (9/2 + 4) × 8 = ? × 10 (d) 310 (e)190
(a) 7.8 (b) 8.8 (c) 6.8 𝟑 𝟐
(d) 5.4 (e) 3.5 793. 𝟒 𝒐𝒇 𝟔 𝒐𝒇 𝟑𝟔𝟎 – 𝟑𝟐% 𝒐𝒇 𝟗𝟓𝟎 =?
𝟓 𝟑
779. 83.33% of 90 + 78 – 168 × 8 ÷ 14 = ? + 34
(a) 23 (b) 32 (c) 34 (a) 12215 (b) 11736 (c) 9736
(d) 52 (e) None of these (d) 10736 (e) 13376
780. 1920/?2 + 96/6 – 10 = 1/30 + 42/7 794. 981 ÷ 3 × 5 + 105 – 54 % 𝑜𝑓 1645 =?
(a) 120 (b) 430 (c) 123 (a) 851.7 (b) 85.17 (c) 8.517
(d) 320 (e) 240 (d) 8517.7 (e) 751.7
781. 41% of 400+ 51% of 700 – 40% of 890 =? 3
795. 5 of? = 48% of 550 + 36% of 750 – 40% of?
(a) 170 (b) 165 (c) 160
(a) 438 (b) 544 (c) 534
(d) 155 (e) 145
(d) 435 (e) 634
782. 836 ÷ 4 × 5 + 128 – 768 =?
796. (1/5) of 1670 + (1/4) of 2456 – (1/0.5) of 677 = ?
(a) 410 (b) 405 (c) 400
A)-432 B)-684 C)-406
(d) 420 (e) 412
𝟑 𝟒 D)568
783. 𝒐𝒇 𝟕 𝒐𝒇 𝟐𝟏𝟎𝟎 =? 797. (441/?) = (?/3364)
𝟓
(a) 560 (b) 680 (c) 820 A)1218 B)1456
(d) 640 (e) 720 C)2457 D)6546
784.
𝟐 𝟑 𝟓
𝒐𝒇 𝟒 𝒐𝒇 𝟖 𝒐𝒇 𝟏𝟗𝟐𝟎 = ? 798. (0.0144)1/2 + (2.5)2 = ?
𝟓 A)5 B)11
(a) 370 (b) 350 (c) 365
C)6 D)9
(d) 360 (e) 240
799. (13824) + (5776) =x2
1/3 1/2
2 1 1
785. 1  2  ?  2 A)20 B)100
5 7 2 C)15 D)10
3 4 3 800. 23% 520 + ? % 730 = 354
3 3 2
(a) 70 (b) 35 (c) 70 A)40% B)32%
17 3 C)25% D)37%
1 1 801. (16)2 + (27)2 – 24 = ?2
(d) 70 (e) 70 A)961 B)41
786. 3136  625  ?2 C)29 D)31
(a) 8 (b) 11 (c) 9 802. 3((2116)1/2 – 16)) + (62 – 3) 2 = ?
(d) 7 (e) 15 A)968 B)1179
1 0 1 4 C)867 D)1289
787. (64) + (64)−2 + (−32)5 =? 803. (550/25) × (2232/17) × (2074/9) = ? × 22
137 139 143
(𝑎) (𝑏) (𝑐) A)30256 B)40657
8 8 8
141 C)34934 D)34678
(𝑑) (𝑒) 15 804. (6×5)2 ÷ 3+300×10 = ?
8
34 48 68
788. × 10 ÷? = 3 A)3300 B)3100
3
(a) 2.4 (b) 4.2 (c) 2.6 C)3000 D)3030
(d) 2.8 (e) 3.2 805. 645×(256)1/2 × 85.74 = 63 × 8?
2
789. (1.06 + 0.04) −? = 4 × 1.06 × 0.04 A)1 B)4
(a) 1.04 (b) 1.4 (c) 1.5 C)2 D)3

For Free Study Material & Quizzes visit : www.mathsbyarunsir.com


For any queries or suggestions email us @ mathsbyarunsir003@gmail.com or whatsapp @ 8881331466
806. (25% of 890 + 45% of 980) ÷ 6 = ? 822. 5431 + 4687 – 8462 =?
A)110.6 B)110.4 (a) 1565 (b) 1656 (c) 1645
C)116.4 D)114.6 (d) 1675 (e) 1665
807. (16/100) × 1440 + (49/100)×549 = 100 + ? 823. 31% of 500+ 41% of 1100 – 50% of 790 = ?
A)399.41 B)499.41 (a) 230 (b) 221 (c) 211
C)349.41 D)494.41 (d) 255 (e) 121
808. 4(2/3)+6(7/5)-11(5/3)+3(4/7) = ? 824. 1107 ÷ 9 × 6 + 236 – 874 = ?
A) 2(103/105) B) 2(102/105) C) 2(105/103) (a) 110 (b) 105 (c) 100
D) 2(105/102) E) None of these (d) 120 (e)112
809. (124÷23) + (56×76) – (45÷ 2.4) = ? 3 4 18
A) 4242.61 B) 4242.62 C) 4242.63 825. 4 𝑜𝑓 27 𝑜𝑓 11 𝑜𝑓 2673 = ?
D) 4242.64 E) None of these (a) 470 (b) 450 (c) 465
810. (7.84)1/2 × (47.61)1/2 – 92161/2 = ? (d) 486 (e) 468
A)- 78.68 B)- 78.86 C)- 76.68 826. (0.064) × (0.4)7 = (0.4)? × (0.0256)2
D)- 78.68 E) None of these (a) 7 (b) 2 (c) 8
811. 7/11 of 6908 − 4/9 of 7704 = ? × 72 (d) 3 (e) 1
A. 9.33 B. 13.5 C. 17 827. 572 ÷ 26 × 12 – 200 = (2)?
D. 21.5 E. 22.5 A. 5 B. 6 C. 8
812. (50² + 30² ) ÷ 6400 = ? D. 7 E. 11
A. 17/20 B. 17/32 C. 17/36 828. 350% of ? ÷ 50 + 248 = 591
D. 17/46 E. 13/41 A. 4700 B. 4900 C. 4100
813. (5)³ – (3.75)³ – (1.25)³– 3(3.75) (6.25) = ? D. 4600 E. 5100
A. 0 B. 1 C. -1 829. 534.596 + 61.472 – 496.708 = ? + 27.271
D. -2.5 E. 2.5 A. 126.631 B. 62.069 C. 72.089
814. 8(2/7) + 30% of 60 + 10(5/9) = ? D. 132.788 E. 145.25
A. 32 B. 34 C. 37 830. (√3 – 2)2 = ? – √12 – √36
D. 44 E. 41 A. 13 – 2√3 B. 13 – 4√3 C. 13 – 6√3
815. √224 x 12.05 + ³√342 x 4 = ? D. 13 – 9√3 E. None of these
(a) 2018 (b) 168 (c) 208 (𝟗 ÷ 𝟐 × 𝟐𝟕 ÷ 𝟗)
831. (𝟏𝟖 ÷ 𝟕.𝟓 × 𝟓 ÷ 𝟒)= ?
(d) 226 (e) 145
816. (216)^1.3 × (36)^1.8 ÷ (6)^0.5 = (6)^? A. 3.5 B. 2.5 C. 5.5
a) 5 b) 4 c) 7 D. 4.5 E. 6.5
d) 3 e)_ None of these 832. (?)2 + (123)2 = (246)2 – (99)2 – 2462
817. 23 × 2 (45 – 15) + 168 ÷ 4 ÷ 2- (11)2 =? A. 184 B. 186 C. 182
(a) 685 (b) 1410 (c) 1080 D. 172 E. 192
(d) 1280 (e) None of these 833. 572 ÷ 26 x 12 – 200 = 2?
A. 5 B. 3 C. 6
D. 7 E. 9
818. 834. [(3√8 + √8) x (8√8 + 7√8)] – 98 = ?
(a) 1050 (b) 1100 (c) 1500 A. 352 B. 382 C. 362
(d) 1000 (e) 1200 D. 372 E. 272
3
819. √123 + 2744 ÷ 72 – 29 + 232 + 50 = ? 835. √(7 x 24 x 2 -(11)³ + 3) = ?
2

A. 52 B. 32 C. 62
(a) 8 (b) 7 (c) 9
D. 72 E. 64
(d) 5 (e) 6
836. 65% of √3136 x 5 = ? + 154
820. 36% of 450 + √3136 – 18 × 13 = ?
A. 56 B. 28 C. 35
(a) – 16 (b) 78 (c) - 34
D. 32 E. 18
(d) 16 (e) 5
837. √[(√7396+ √2401)+?2] = 100
4

A.7 B.10 C.12


821. D.14 E. 16
(a) 1500 (b) 200 (c) 1000 838. (45)2 + (21)2 = (?)2 + 257
(d) 40 (e) None of these A. 51 B. 49 C. 45
D. 47 E. 61
For Free Study Material & Quizzes visit : www.mathsbyarunsir.com
For any queries or suggestions email us @ mathsbyarunsir003@gmail.com or whatsapp @ 8881331466
839. 65% of 400 + √? = 44% of 800 – 12% of 400 856. ?2 = (√1444 + √676) ÷ 4
A. 1936 B. 44 C. 2115 (𝑎) 6 (𝑏) 12 (𝑐) 8
D. 46 E. 48 (𝑑) 2 (𝑒) 4
840. 40% of 375 ÷ 512 × 30 = 4500 ÷ ? ?
857. 2 = 64 × 288 ÷ 256 ÷ 36 × 16
(a) 8 (b) 10 (c) 9 (a) 2 (b) 3 (c) 4
(d) 12 (e) 15 (d) 5 (e) 6
841. 299 ÷ 169 × 11 – 361 = ?2 + 38 3
(a) 11 (b) 8 (c) 16 858. 90% 𝑜𝑓 ? = √576 ÷ √64 × √729
(d) 14 (e) 24 (𝑎) 90 (𝑏) 180 (𝑐) 270
842. 94.5 × 512 ÷ 343 ÷ 6.4 = ?2 + 56 (𝑑) 162 (𝑒) 540
(a) -56 (b) 44 (c) 25 859. ? = 40% 𝑜𝑓 320 + 4³ ÷ 18 × 108 ÷ 8
(d) -32 (e) 35 (a) 176 (b) 172 (c) 168
843. 86 – (86)2 + 86 × (86 + 86 ÷ 0.86) = ? (d) 166 (e) 186
A. 9696 B. 8486 C. 8686 860. 60% 𝑜𝑓 (152 ÷ 19) × ? = 3³ × 4²
D. 6844 E. None of these (a) 80 (b) 60 (c) 45
3
844. 57% of 350 + ?% of 170 = – 89.5 – √512 – 9 (d) 90 (e) 30
(a) 210 (b) -165 (c) -180 861. (3375)3.7 ÷ (225)4.3 × (15)2.5 ÷ (3)5 =
(d) 190 (e) -155 (5)?
(a) 3 (b) 3.5 (c) 4.5
845. √529 ÷ ?% of 270 = 115 × √81 ÷ ?2
(d) 5 (e) 4
(a) 125.6 (b) 121.5 (c) 130.4
(d) 117.2 (e) 23.5 862. 1396 + 412 − 2704 = ? −(31)²
846. ? = (6 − 3 ) × (82 − 42 ) ÷ (10.52 − 1.52 )
2 2 (a) 64 (b) 71 (c) 65
(d) 75 (e) 68
(a) 6 (b) 8 (c) 9
(d) 12 (e) 15 863. 45% 𝑜𝑓 280 + 72% 𝑜𝑓 550 = 90% 𝑜𝑓 ?
847. 35% of 540 + 410% of 30 = 39% of ? (a) 540 (b) 550 (c) 580
(a) 900 (b) 800 (c) 700 (d) 600 (e) 630
(d) 950 (e) 850 864. (14 – 8 ) ÷ ? × (72 – 42 ) = ? × 400
2 2

5
848. ?² = 40% of 11 of 352 (a) 33 (b) 66 (c) 6.6
(d) 4.4 (e) 3.3
(a) 12 (b) 16 (c) 6 4 6 8
(d) 4 (e) 8 865. 8 11 × 5 23 × ? = √484 + √4356
849. ? ÷ 15 × 11 = 195 ÷ 108 × 132 ÷ 65 (a) 11 (b) 8 (c) 6
(a) 5 (b) 15 (c) 25 (d) 4 (e) 2
4 1 2 3 1 2
(d) 55 (e) 12 866. 7 3 + 3 2 + 5 3 = ? +4 5 − 7 2 + 11 5
2
850. ? = (√1444 + √676 ) ÷ 4 (a) 10 (b) 9 (c) 95
(a) 6 (b) 16 (c) 8 (d) 105 (e) 11
(d) 2 (e) 4 5
867. 66% 𝑜𝑓 350 + ? = 8 𝑜𝑓 1256
851. 2? = 64 × 288 ÷ 256 ÷ 36 × 16
(a) 2 (b) 3 (c) 4 (a) 521 (b) 496 (c) 554
(d) 5 (e) 6 (d) 568 (e) 544
1 2 3
852. 90% of ?= √576 ÷ 3√64 × √729 868. 33 3 % of 360 + 66 3 % of 120 =?× 7 11
a) 90 (b) 180 (c) 270 (a) 22.5 (b) 27.5 (c) 2.25
(d) 162 (e) 540 (d) 2.75 (e) 225
853. ? = 40% of 320 + 4³ ÷ 18 × 108 ÷ 8 869. (1600 + 1970) ÷ (825 + 365) = ?
(a) 176 (b) 172 (c) 168 a) 5 b) 3 c) 1
(d) 166 (e) 186 d) 2.25 e) 2.75
854. 60% of (152 ÷ 19) × ? = 3³ × 4² 200
870. 65 = 150% 𝑜𝑓 200 × 40% 𝑜𝑓
?
(a) 80 (b) 60 (c) 45 a) 1 b) 2 c) 4
(d) 90 (e) 30 d) 6 e) 37
855. ? ÷ 15 × 11 = 195 ÷ 108 × 132 ÷ 65 871. 𝟓√𝟏𝟎𝟖% 𝐨𝐟 𝟕𝟓 + 𝟑𝟐 × 𝟓 + 𝟐 + 45% of 460 =?
(a) 5 (b) 15 (c) 25 (a) 410 (b) 120 (c) 210
(d) 55 (e) 12 (d) 310 (e)190
For Free Study Material & Quizzes visit : www.mathsbyarunsir.com
For any queries or suggestions email us @ mathsbyarunsir003@gmail.com or whatsapp @ 8881331466
𝟑 𝟐
872. 4 𝟓 of 6 𝟑 of 360 – 32% of 950 =? 886. 6(3/4) + 4 (3/8) – 6 (¼) = ? – 5 (½)
(a) 12215 (b) 11736 (c) 9736 a) 11 (1/8) b) 12 (3/8) c) 10 (3/8)
(d) 10736 (e) 13376 d) 9 (2/3) e) None of these
(242 −142 ) 46 887. ? = (3745 + 1780) ÷ 17 + 162 + (5/
873. (120% 𝑜𝑓 200 × 2.4) + − 2 =?
2
a) 757 b) 743 c) 780 11) 𝑜𝑓 880 – 15
a) 824 b) 872 c) 938
d) 792 e) 634
d) 966 e) 866
874. ? 2 % 𝑜𝑓 11.11% 𝑜𝑓 256 × 1872 ÷ 2704 = 888. 2516 ×? + 16 × 11 = 2526 + 5198
81 a) 3 b) 5 c) 7
a) 9.75 b) 10.50 c) 11.25 d) 8 e) 8
d) 12.75 e) None of these
889. 156 ÷ 13 + ∛19683 – (12 × 15) =? − 83
875. 𝟑√(𝟒𝟎% 𝒐𝒇 𝟔𝟒𝟎 + 𝟐𝟎𝟖. 𝟔 + 𝟐𝟔𝟒. 𝟒) + 𝟓𝟔% 𝒐𝒇 𝟗𝟎𝟎 =
𝟒
a) 432 b) 415 c) 371
𝒐𝒇 ?
𝟓 d) 384 e) None of these
(a) 441.25 (b) 541.25 (c) 645.25 1 1 4
890. 7 × 5 × 9 × 3150 + 40% 𝑜𝑓 7800 =? – 540
(d) 641.25 (e) 741.25
𝟑 𝟐 𝟏 a) 3700 b) 5600 c) 4400
876. 𝟏 𝟒 + 𝟐 𝟑 + 𝟐 𝟐 = ? d) 3200 e) None of these
7 11 1
(𝑎) 9 12 (𝑏) 6 12 (𝑐) 3 12 891. 318 × 5160 ÷ 43 % 𝑜𝑓 2400 = 46 × 990 ÷
5 5
(𝑑) 4 12 (𝑒) 7 12 15 + ? – 2400
6 a) 1026 b) 954 c) 1288
877. 35% 𝑜𝑓 42% 𝑜𝑓 7 𝑡ℎ 𝑜𝑓 500 =? d) 1342 e) 820
(a) 36 (b) 44 (c) 52 892. 98 ÷ 14 × 49 − 294 = (? )2
(d) 60 (e) 63
(a) –14 (b) √7 (c) –7
878. 8888 + 848 + 88 − ? = 7337 + 737 (d) 196 (e) 156
(a) 1750 (b) 1650 (c) 1550
(d) 1450 (e) 1250 893. (2 × 3)3 ÷ (4 × 9)2 × (27 × 8)2 = (6)?
(a) 5 (b) 6 (c) 3
879. 414 × ? × 7 = 127512 (d) 8 (e) 12
(a)36 (b) 40 (c) 44
(d) 48 (e) 52 894. 454.58 − 376.89 + 121.45 − 95.42 =?
(a) 102.22 (b) 103.72 (c) 91.72
880. 2⁵ + 2¹⁰ = ? × 12 (d) 92.32 (e) 97
(a) 68 (b) 72 (c) 80
(d) 88 (e) 96 895. √576 ÷ (4)2 × 7.4 + (7)3 − 231 =?
(a) 123.9 (b) 121.1 (c) 111.1
881. √256 + √784 = ? × √121 (d) 122.1 (e) 123.1
(a) 4 (b) 5 (c) 3 2
(d) 6 (e) 2 896. (√3 − 2) =? −√12 − √36
5 1 1 5
882. ? + 2 + 1 4 = 1 8 + 2 2 + 7 4
1 (a) 13-2√3 (b) 13 (c) 13-4√3
(d) 1 (e) 6
71 73 65 897. [(30)3 ÷ 60 × 24] ÷ 25 = 36 ×?
(𝑎) (𝑏) (𝑐) (a) 12 (b) 15 (c) 18
8 8 8
63 1
(𝑑) (𝑒) 7 (d) 21 (e) 28
4 8
898. 8493 + 3148 + 955 − ? = 8727 + 963
883. 115% 𝑜𝑓 360 + 180% 𝑜𝑓 270 = ? × 225 (a) 2932 (b) 2923 (c) 2906
(a) 4.5 (b) 3 (c) 3.25 (d) 2942 (e) 2755
(d) 3.5 (e) 4 899. 80% 𝑜𝑓 24000 – 16% 𝑜𝑓 7200 =
? ? – 15% 𝑜𝑓 2000
884. 9 × 729 = 3⁴ × 9⁶ ÷ 81
(a) 2 (b) 3 (c) 4 a) 12360 b) 18348 c) 20650
(d) 5 (e) 6 d) 16408 e) 14055
12 54 7 900. 292 + 52 % 𝑜𝑓 900 – (3/7) 𝑜𝑓 1750 = ? – 113
885. 40 % 𝑜𝑓 (18) 𝑜𝑓 (147) 𝑜𝑓 (12) 𝑜𝑓175 =?
a) 2150 b) 1560 c) 1890
a) 25 b) 40 c) 15
d) 2780 e) 2940
d) 10 e) None of these

For Free Study Material & Quizzes visit : www.mathsbyarunsir.com


For any queries or suggestions email us @ mathsbyarunsir003@gmail.com or whatsapp @ 8881331466
901. ∛35937 + √5184 ÷ 2? – 4/5 𝑜𝑓 65 + 52 = 42 (a) 6 (b) 4 (c) 12
a) 1 b) 2 c) 3 (d) 2 (e) 8
2
d) 4 e) 5 917. 2652 − 441 + 928 − 6 3 % 𝑜𝑓 3375 = ? +(31)2
902. 46% 𝑜𝑓 450 + 84% 𝑜𝑓 525 – 55% 𝑜𝑓 880 =? (a) 1953 (b) 1853 (c) 1825
a) 164 b) 166 c) 170 (d) 1935 (e) 1950
d) 162 e) 160 918. (272 − 32) × (124 + 176) ÷ (17 × 15– 15) = ? +15 × 16
(a) 50 (b) 65 (c) 72
903. √256 + 45 𝑜𝑓 400 + 3 3
𝑜𝑓 ? = 𝑜𝑓 208
4 13 (d) 60 (e) 55
a) 36 b) 32 c) 34 3
919. 125% 𝑜𝑓 92 – √4096+ ? = √10201
d) 30 e) 28
(−2) (−2)
(a) 5 (b) 2 (c) 4
904. (−3)(−2) ÷ (3)(−2) =? (d) 3 (e) 9
(a) 1 (b) –1 (c) 3 920. 34 × 2 × 31 ÷ √961 = ? − 540 ÷ 18
(d) –3 (e) None of these
(𝑎) 98 (𝑏) 83 (𝑐) 93
905. ? % 𝑜𝑓 250 – 48% 𝑜𝑓 250 = ? (𝑑) 95 (𝑒) 82
(a) 70 (b) 75 (c) 85 921. 3333 ÷ 33 + 4004 ÷ 26 + 650 ÷ 25 = ?
(d) 72 (e) 80 (a) 320 (b) 297 (c) 281
1
906. √46225 ÷ (3125)5 =? −215 (d) 300 (e) 241
(a) 258 (b) 255 (c) 218 922. 15.5 × 28 ÷ 112 + 1230 ÷ 240 = ? ÷ 5
(d) 235 (e) 248 (a) 55 (b) 45 (c) 65
2 1 9 188 (d) 35 (e) 75
907. 3 7 + 4 14 − 14 = ? 9 4
(a) 14 (b) 28 (c) 35 923. 13 × 221 + 1 9 × 378 = 241+ ?
(d) 7 (e) 24 (a) 525 (b) 614 (c) 520
908. √15 × 22 − 40%𝑜𝑓602 + 19 × 39 = ?2
2 (d) 458 (e) 420
2
(a) 81 (b) 21 (c) 19 924. 12.5% 𝑜𝑓 928 – 14 7 % 𝑜𝑓 980 + (12)2 ÷ (2)2 = ?
(d) 11 (e) 9 (a) 10 (b) 8 (c) 12
909. 40% 𝑜𝑓 ? + 55% 𝑜𝑓 360 = 36% 𝑜𝑓 450 + 102 (d) 14 (e) 16
(a) 64 (b) 320 (c) 160 925. √18 × 8 + 37.5% 𝑜𝑓 216−? = √1444 − √529
(d) 80 (e)20 (a) 4 (b) 0 (c) 6
(54)2
910. √144 × √324 ÷ 4 (13 ÷ 24) = ? (d) 12 (e) 2
3 5 2
(a) 432 (b) 0.75 (c) 243 926. 𝑜𝑓 9 𝑜𝑓 7 𝑜𝑓 9450 = ?
5
3
(d) 64 (e) 1.5
3 (a) 960 (b) 480 (c) 450
911. 34 ÷ 362 × 243 = ?2
(d) 900 (e) None of these
(a) 16 (b) 14 (c) 6
(d) 18 (e) 12 927. 41% 𝑜𝑓 400 + 51% 𝑜𝑓 700 – 40% 𝑜𝑓 890 =?
(a) 170 (b) 165 (c) 160
912. 72.3 × 494.7 × 633.4 × 815.85 = 63?
(d) 155 (e) 135
(a) 16.25 (b) 15.1 (c) 13.4
(d) 18.9 (e) None of these 928. 836 ÷ 4 × 5 + 128 – 768 =?
913. (? )2 + (164)2 = (307)² − 272 (a) 410 (b) 405 (c) 400
(a) 151 (b) 189 (c) 211 (d) 420 (e) 412
(d) 259 (e) 281 929. 𝟓𝟔 % 𝒐𝒇 𝟐𝟓𝟎 + 𝟑𝟑 𝟏𝟑 % 𝒐𝒇 𝟗𝟔𝟑 =?
(a) 367 (b) 473 (c) 428
914. (21.6)² ÷ (– 7.2)² × ? =
(d) 385 (e) 461
15483.36 – 15276.09 𝟐 𝟑 𝟓
(a) 23.03 (b) 23.3 (c) 32.03 930. 𝟓 𝒐𝒇 𝟒 𝒐𝒇 𝟖 𝒐𝒇 𝟏𝟗𝟐𝟎 = ?
(d) 32.3 (e) None of these (a) 370 (b) 350 (c) 365
(d) 360 (e) 340
915. 348 ÷ 29 × 15 + 156 = (? )³ + 120 𝟓 𝟓 𝟒 𝟏
(a) 12 (b) 6 (c) 36 931. 𝟑 𝟕 𝒐𝒇 𝟓 𝟏𝟑 + 𝟒 𝟏𝟏 × 𝟐 𝟏𝟔 =?
(d) 9 (e) None of these (a) 25 (b) 29 (c) 30
2 (d) 20 (e) None of these
916. (2√392 – 21) + (√8– 7) = (? )2

For Free Study Material & Quizzes visit : www.mathsbyarunsir.com


For any queries or suggestions email us @ mathsbyarunsir003@gmail.com or whatsapp @ 8881331466
𝟐𝟎𝟎 𝟒𝟎𝟎 (d) 78 (e) 72
932. √𝟕𝟐% 𝒐𝒇 + 𝟏𝟏𝟏% 𝒐𝒇 =?
𝟑 𝟑 947. ? ² = 40% 𝑜𝑓 420 + 44% 𝑜𝑓 200
(a) 14 (b) 13 (c) 15 (a) 24 (b) 12 (c) 8
(d) 12 (e) 16 (d) 16 (e) 416
933. ? % 𝒐𝒇 𝟏𝟑𝟎𝟎 + 𝟓𝟓% 𝒐𝒇 𝟒𝟔𝟎 = 𝟖? 𝒐𝒇 𝟏𝟔𝟖 – 𝟏𝟒 𝒐𝒇 𝟗𝟎𝟖 948. 343 + 243 + 512 = 20% 𝑜𝑓 ?
(a) 50 (b) 55 (c) 60 (a) 4590 (b) 5490 (c) 6490
(d) 45 (e) 65 (d) 6140 (e) 5290
2 5 4 6
934. 3 5 𝑜𝑓 5 17 + 5 13 × 2 23 =? 949. (0.3)3 × (0.09) × (0.027) = (0.0081)?+2
(a) 0 (b) 1 (c) 2
(a) 25 (b) 30 (c) 40
(d) 3 (e) 4
(d) 20 (e) 15
950. 3√1728 + √784 + √1296 = ? + 3√512
935. ? % 𝑜𝑓 300 + 45% 𝑜𝑓 440 =
? 3
(a) 86 (b) 68 (c) 74
𝑜𝑓 63 – 17 𝑜𝑓 578 (d) 72 (e) 64
7
1 3 4 3
(a) 50 (b) 55 (c) 60 951. 14 11 + 16 + 14 + 12 = ?
11 121 11
(d) 45 (e) 40 (a) 59
54
(b) 39
23
(c) 61
82
3 121 121 99
936. ? 𝑜𝑓 7 – 42% 𝑜𝑓 250 = 315 (d) 56 121
81 81
(e) 57 121
(a) 970 (b) 1000 (c) 980 952. 16.5% 𝑜𝑓 300 + 70.5% 𝑜𝑓 1400 – 10% 𝑜𝑓 480 = ?
(d) 985 (e) 975 (a) 1280.75 (b) 1084.5 (c) 986.25
937. 79% 𝑜𝑓 500 + 113% 𝑜𝑓 700 = ?2 + (d) 1175.5 (e) 988.5
11
𝑜𝑓 1170 What should come in place of (x) in the following questions and
13
(a) 14 (b) 15 (c) 13 you have to check each and every option carefully.
(d) 11 (e) 16 निम्िनिनित प्रश्नों में (x) के स्थाि पर क्या आिा चानिए और आपको प्रत्येक
1 निकल्प को ध्याि से देििा िोगा।
938. √? – 423 ÷ 9 × 13 + 11% of 5000 = 3 of 57 5
(a) 8100 (b) 3600 (c) 6400 953. 35% 𝑜𝑓 180 + 182 = (27)3 + 𝑥 2
(d) 6436 (e) 6600 (a) 3√1728 (b) 2√196 (c) 3√2744
939. 3735 + 4565 + 6245 = ? + 6432 (d) prime number (e) None of these
323
(a) 8126 (b) 8113 (c) 6393 954. 357 × 441 − 15 × 21 = 𝑥
(d) 7865 (e) 8191 (a) multiple of 21 (b) 2√7056 (c) 2√7396
? 6 21
940. 3 𝑜𝑓 7 𝑜𝑓 36 𝑜𝑓 216 = 108 – 72 ÷ 2 – 36 (d) both (a) and (b) (e) both (a) and (c)
(a) 2 (b) 1.5 (c) 1
955. √52 × 41 × 5 – 172 – 75 = 𝑥
(d) 3 (e) 3.5
1 (a) tens place value of answer is multiple of 3
941. ? +√841 + –3 = 122 + 3 × 37 (b) tens place value of answer is multiple of 2
(3)
(a) 217 (b) 176 (c) 203 2
(c) √4761
(d) 199 (e) 165 (d) both (a) and (c)
942. 324 ÷ 12 + 29 × 3 – 48 + 39 = ? (e) All (a), (b) and (c)
(a) 136 (b) 219 (c) 91 2
(d) 110 (e) 105 956. 𝑥 3 = 64% 𝑜𝑓 150 + 7 × 3 − 92
943. 44% of 250 + 30% of 270 + 13² = ? (a) even number (b) cube of 8 (c) cube of 6
(a) 260 (b) 360 (c) 355 (d) both (a) and (b) (e) both (a) and (c)
(d) 336 (e) 349 957. 𝑥 × 4 − 402 = 142 − 36 × 44
3
√729 –√729+√324 (a) prime number
944. =?
√1444 (b) tens place value of answer is multiple of 5
(a) 0 (b) 1 (c) √123 (c) 2√2601
1
(d) √17 (e) 19 (d) all (a), (b) and (c)

2 108 21 27 (e) both (a) and (b)
945. √961 + 3 of of 36 of 42 = ? Directions :
9
(a) 27 (b) 29 (c) 23 What should come in place of (x) in the following questions and
(d) 34 (e) 38 you have to check each and every option carefully.
946. (841 ÷ 29) + (34 × 5) – (23 × 5) =? निम्िनिनित प्रश्नों में (x) के स्थाि पर क्या आिा चानिए और आपको प्रत्येक
(a) 88 (b) 81 (c) 84 निकल्प को ध्याि से देििा िोगा।

For Free Study Material & Quizzes visit : www.mathsbyarunsir.com


For any queries or suggestions email us @ mathsbyarunsir003@gmail.com or whatsapp @ 8881331466
958. 45% of 80 + √841 + x² = 2121 ÷ 21 C. Quantity II ≤ Quantity I / मात्राII≤मात्राI
(a) multiple of 3 (b) even number (c) multiple of 2 D. Quantity II ≥ Quantity I / मात्राII ≥मात्राI
(d) both (a) and (b) (e) all (a) , (b) and (c) E. Quantity II = Quantity I or no relation / मात्राII = मात्राI
36+3𝑥 याकोईसंबंधििीं
959. 23 + 28 ÷ 162 = 13 × 4
(a) even number (b) odd number (c) prime number 969. 8888 + 848 + 88 – x = 7337 + 737
(d) multiple of 3 (e) None of these And
618 + 62 × 0.50 – 29 = y
960. 73 × 25 ÷ 43 + 175% 𝑜𝑓 350 = x2 Quantity I (मात्राI): value of x.
(a) multiple of 7 (b) multiple of 2 (c) both (a) and (b) Quantity II (मात्राII): value of 3y.
(d) prime number (e) None of these A. Quantity II < Quantity I / मात्राII <मात्राI
961. 23 × 24 + 23 × 47 – 23 × 54 = 𝑥 B. Quantity II > Quantity I / मात्राII>मात्राI
(a) multiple of 19 (b) odd number (c) multiple of 17 C. Quantity II ≤ Quantity I / मात्राII≤मात्राI
(d) both (a) and (b) (e) both (b) and (c)
D. Quantity II ≥ Quantity I / मात्राII ≥मात्राI
962. 120% of 650 + 30 + 255 ÷ 5 = 𝑥 E. Quantity II = Quantity I or no relation / मात्राII = मात्राI
(a) odd number (b) prime number (c) multiple of 3 याकोईसंबंधििीं
(d) multiple of 7 (e) both (a) and (d) 970. 3.2% of 500 × 2.4% of x = 288
4 2 8 234
963. 7 − 3 + 4 = And
5 3 15 𝑥
(a) odd number (b) multiple of 3
3
(c) √19683 y % of 800 = 293 – 22% of 750
Quantity I (मात्राI): value of y3.
(d) both (a) and (b) (e) all (a), (b) and (c)
Quantity II (मात्राII): value of x.
964. 96% of 1050 − ? % of 500 = 998 A. Quantity II < Quantity I / मात्राII <मात्राI
(a) prime number (b) even number (c) can’t say B. Quantity II > Quantity I / मात्राII>मात्राI
(d) 1 < x < 3 (e) both (a) and (d)
C. Quantity II ≤ Quantity I / मात्राII≤मात्राI
965. x × (1350 ÷ 112.5) = √5929 + √8281
D. Quantity II ≥ Quantity I / मात्राII ≥मात्राI
(a) 13 > x > 15 (b) multiple of 7 (c) x is even number
E. Quantity II = Quantity I or no relation / मात्राII = मात्राI
(d) All (a), (b) and (c) (e) both (b) and (c)
याकोईसंबंधििीं
966. 18750 ÷ √𝑥 = 36 × 11 + 59 × 6 971. √𝟏𝟐𝟐𝟓 ÷ 𝟑√𝟑𝟒𝟑 × 45% of 760 = ?
(a) 53 (b) 54 (c) 63 (a) Multiple of 5 (b) Multiple of 10 (c) Multiple of 2
2
(d) 5 (e) None of these (d) Multiple of 3 (e) All of the Above
𝑥
967. 3 = 729 ÷ 243 × 216 × 72 ÷ 576 972. 175% 𝑜𝑓 460 + 110% 𝑜𝑓 170 + 2? = 1000
(a) 8 < x < 9 (b)5> x > 7 (c) 3> x >5
(a) Prime No. (b) Positive no. (c) Non Prime No.
(d) 0 > x > 1 (e) None of these
(d) Both A & B (e) Both B & C
Directions : 1 3 1 3 18 35
In these questions two quantity numbered I and II are given. You 973. 3 7 + 2 5 + 7 5 − 5 7 − 35 = ?
have to solve both the equations and mark the appropriate option. (a) Prime No. (b) Positive no. (c) Non Prime No.
Given answer- (d) Both A & B (e) Both B & C
इि प्रश्नों में दो मात्रा I और II दी गई िैं आप को दोिों समीकरणों को िि करिा 1 3 2
974. 52 ÷ 2 𝑜𝑓 8 + 5 × 1 3 = ?
िोगा और उनचत निकल्प को नचनहित करिा िोगा (a) Multiple of 5 (b) Multiple of 10 (c) Multiple of 4
A. Quantity II < Quantity I / मात्राII <मात्राI (d) Multiple of 3 (e) Multiple of 7
B. Quantity II > Quantity I / मात्राII>मात्राI 975. ? % of 400 + 310% of 60 = 500% of 82
C. Quantity II ≤ Quantity I / मात्राII≤मात्राI (a) Multiple of 8 (b) Multiple of 7 (c) Multiple of 5
D. Quantity II ≥ Quantity I / मात्राII ≥मात्राI (d) Both A & B (e) All of the Above
E. Quantity II = Quantity I or no relation / मात्राII = मात्राI 𝟑
976. ? ³ = 𝟕 of 1771-864 ÷ 16 × 𝟗
𝟓
याकोईसंबंधििीं
(a) Multiple of 5 (b) Multiple of 10 (c) Multiple of 2
968. (46351–36418–4505) ÷ x = 1357
(d) Multiple of 3 (e) All of the Above
And
y % of 800 = 293 – 22% of 750 977. 156 + 376 − ? = 13% of 3000
Quantity I (मात्राI):value of x. (a) Multiple of 3 (b) Multiple of 2 (c) Multiple of 5
Quantity II (मात्राII):value of √𝑦. (d) Both A & B (e) None of the Above
978. 115% of 4880 – 85% of 1640 = 740% of ?
A. Quantity II < Quantity I / मात्राII <मात्राI
(a) Multiple of 3 (b) Multiple of 2 (c) Multiple of 5
B. Quantity II > Quantity I / मात्राII>मात्राI
(d) Both B & C (e) All of the Above

For Free Study Material & Quizzes visit : www.mathsbyarunsir.com


For any queries or suggestions email us @ mathsbyarunsir003@gmail.com or whatsapp @ 8881331466
1 2 1
979. 8 3 × 2 5 +? = 14 6 × 3 17
3 989. 132 - 24 + 789 ÷ 263 = 46 * ? ÷ 23
A. 77 B. 74 C. 78
(a) Multiple of 3 (b) Multiple of 2 (c) Multiple of 5
D. 68 E. 64
(d) Both A & B (e) Multiple of 7
980. (21 - 12 ÷ 3 × 2) ? [13 − (12 − 6 ÷ 3)] 990. 80% of 45 ÷ 12% of 50 + (?) = 18
A. 12 B. 10 C. 4
a) > (Greater than) b) < (Less than)
D. 8 E. 6
c) ≤ (either lesser than or equal to )
991. (76 + 24 x 5 - 11 x 8) ÷ 4 + √144 = ?
d) ≥ (either greater than or equal to) A. 25 B. 37 C. 46
e) = (equal to) D. 41 E. 39
981. (54 ÷ 3 of 6 + 9) ? [13 − (12 − 6 ÷ 3) ]
992. (3214 + 886 - 1073 - 1731)1/4 + (?) = √361
a) > (Greater than)
A. 21 B. 13 C. 19
b) < (Less than)
D. 18 E .11
c) ≤ (either lesser than or equal to )
993. √1225 + √5776 − √3249 = √? + √1024
d) ≥ (either greater than or equal to) A. 576 B. 400 C .529
e) = (equal to) D. 484 E. 441
982. 8[1 - {3 + (5 - 6 + 7)}] ? 0.8{0.75 ÷ (1.35 - 0.6)} - 0.8 994. 56% of 3150 - 130% of 630 = 900% of ?
a) > (Greater than) A. 85 B. 125 C. 115
b) < (Less than) D 105 E. 95
c) ≤ (either lesser than or equal to ) 995. 78 * 34 ÷ 221 - 121 * 209 ÷ 1331 = ?
d) ≥ (either greater than or equal to) A. 4 B. 5 C. 6
e) = (equal to) D. -9 E. -7
983. (37 - 6 × 4 + 32 ÷ 8) ? (64 - 48 ÷ 6 × 4 + 8) 996. 12.5% of 64 + 80% of 25 - ?% of 180 = 19
a) > (Greater than) A. 8 B. 7 C. 6
b) < (Less than) D .5 E. 4
c) ≤ (either lesser than or equal to ) 997. 425 - 68% of 350 + ? = 162 - √2809
d) ≥ (either greater than or equal to) A. 12 B. 26 C. 24
e) = (equal to) D. 16 E. 20
984. (12 + 4 - 8 ÷ 2 × 3) ? (7 - 5 + 14 ÷ 2 + 6) 998. √945 ÷ 45 + 12 𝑥 5 - ? = 156 ÷ 39
a) > (Greater than)
A. 3 B. 7 C. 9
b) < (Less than)
D. 2 E. 5
c) ≤ (either lesser than or equal to ) 2
999. 11 x ? x 23 = 57 + 2570
d) ≥ (either greater than or equal to) A. 33 B. 25 C. 17
e) = (equal to) D. 23 E. 27
985. (16 + 8 ÷ 4 − 2 × 3) ? (15 + 5 ÷ 5 × 2 ) 1000. 90% of 670 + ?% of 480 = 1131
a) > (Greater than) A. 140 B. 95 C. 110
b) < (Less than) D. 130 E. 155
c) ≤ (either lesser than or equal to ) 1001. 320.16 x 320.04 = ?
d) ≥ (either greater than or equal to) A. 1 B. 2 C. 4
e) = (equal to) D. 8 E. 16
𝟒 1002. (?) + 1 = 23 + 292
2 2
986. (2450 ÷ 7 + 112 × 2.5) ? 𝟕of 441 ÷ 18 + 14
A.-38 B. -53 C. -113
a) > (Greater than) D. 37 E. 142
b) < (Less than)
1003. 4 𝑜𝑓 √256 = 80? 𝑜𝑓 3√125
?

c) ≤ (either lesser than or equal to ) A. 34 B. 31 C. 19


d) ≥ (either greater than or equal to) D. 22 E. 10
e) = (equal to)
987. 126 ÷ 14 * 9√2 - √72 = ?√2
A. 75 B. 125 C. 50
D. 100 E. 150
988. 32% of 250 − 45% of 120 = 10 - (?/3)
A. 48 B. 36 C. 24
D. -36 E. -48

For Free Study Material & Quizzes visit : www.mathsbyarunsir.com


For any queries or suggestions email us @ mathsbyarunsir003@gmail.com or whatsapp @ 8881331466
For Free Study Material & Quizzes visit : www.mathsbyarunsir.com
For any queries or suggestions email us @ mathsbyarunsir003@gmail.com or whatsapp @ 8881331466
For Free Study Material & Quizzes visit : www.mathsbyarunsir.com
For any queries or suggestions email us @ mathsbyarunsir003@gmail.com or whatsapp @ 8881331466
For Free Study Material & Quizzes visit : www.mathsbyarunsir.com
For any queries or suggestions email us @ mathsbyarunsir003@gmail.com or whatsapp @ 8881331466
For Free Study Material & Quizzes visit : www.mathsbyarunsir.com
For any queries or suggestions email us @ mathsbyarunsir003@gmail.com or whatsapp @ 8881331466
Solution sections Sol. Sol.
1. Ans.(d) is correct √961 + √1369 − √1444 = √361 + √?
31 + 37 − 38 = 19 + √?
√? = 30 − 19
√? = 11
2. Ans.(b) is correct ? = (11)2
= 121
11. Ans.(a) is correct
Sol.
52500 ÷ 7 + 64680 ÷ 6 = 19500 − ?
3. Ans.(a) is correct 7500 + 10780 = 19500 − ?
18280 = 19500 − ?
? = 19500 − 18280
4. Ans.(c) is correct = 1220
12. Ans.(e) is correct
Sol.
28.5 × 34 + 2320 ÷ 8 = (36)2 − ?
969 + 290 = 1296 − ?
1259 = 1296 − ?
5. Ans.(b) is correct ? = 1296 − 1259 = 37
13. Ans.(d) is correct
Sol.
? = 47 × 27 + 15600 ÷ 8 + 181
1269 + 1950 + 181
6. Ans.(e) is correct
= 3400

14. Ans.(c) is correct


7. Ans.(e) is correct
8. Ans.(a) is correct ? = 112.5 × 5 + 4560 ÷ 6 − 175 × 7
= 562.5 + 7602 − 1225
= 1322.5 − 1225
= 97.5
15. Ans.(e) is correct

9. Ans.(b) is correct (16 + 18) ÷ 17 = (? )0.2


2 = (? )0.2
? = 32
16. Ans.(d) is correct
3 5 32 ?
× 27 × 45 × 2187 = 100 × 400
8
4
10. Ans.(d) is correct × 2187 = 4 ×?
81

For Free Study Material & Quizzes visit : www.mathsbyarunsir.com


For any queries or suggestions email us @ mathsbyarunsir003@gmail.com or whatsapp @ 8881331466
27 × 4 1143 4
?= + =?
4 33 11
? = 27 ? = 35
23. Ans.(c) is correct
17. Ans.(a) is correct 1 ?
(33 )2.6 × + (25 )0.8 = × 1036
Sol. (34 )0.7 100
7 + 10– 9 2 ?
(4 + 6 – 8) + = ? +1 37.8–2.8 + 24 = × 1036
12 3 100
2 2 ?
2 – 1 =? 35 + 24 = × 1036
3 3 100
?=1 ?
243 + 16 = × 1036
100
18. Ans.(c) is correct 259 × 100
?=
56 475 1036
× 4750 + × 12 – (? )2 = 217 ? = 25
100 100
2660 + 57 – (?)² = 217
24. Ans.(b) is correct
(?)² = 2717 – 217
Sol.
(?)² = 2500
166.8 + 272.3 – ? = 169 + 20
? = 50
? = 439.1 – 189
19. Ans.(e) is correct
? = 250.1
Sol.
(268.20 – 76.2) – ? = 144
? = 192 – 144
25. Ans.(d) is correct
? = 48
Sol.
20. Ans.(b) is correct
(4/9) × 486 + 35% of 120 +(3/8) × 208 = ?
275.1
+ 84.3 – ? = 169 4 × 54 + 0.35 × 120 + 3 × 26= ?
3
(91.7 + 84.3) – 169 = ? ? = 216 + 42 + 78
?=7 ? = 336
21. Ans.(d) is correct
37 26. Ans.(a) is correct
×? +121 – 171 + 12 = 961 Sol.
100
37 ×? (7800 ÷ 312) + 65% of 1120 = 9 × ? + 24
= 961 + 38
100 25 + 0.65 × 1120 = 9 × ? + 24
37×? = 99900 25 + 728 – 24 = 9 × ?
? = 2700 ? = 729 ÷ 9
22. Ans.(a) is correct ? = 81
26 1 4 Hence, option a.
( × 3800 + × 620) ÷ 33 + =?
100 4 11
4
(988 + 155) ÷ 33 + =? 27. Ans.(c) is correct
11
Sol.

For Free Study Material & Quizzes visit : www.mathsbyarunsir.com


For any queries or suggestions email us @ mathsbyarunsir003@gmail.com or whatsapp @ 8881331466
? × 15 + (3960 ÷ 12) = 262 + (350 ÷ 25) 187 ?
× (42)2 − 22 × 38 =
? × 15 + 330 = 676 + 14 357 4
11 ?
? = (690 – 330) ÷ 15 × 42 × 42 − 22 × 38 =
21 4
? = 24 22(42 − 38) × 4 = ?
? = 352

28. Ans.(e) is correct


34. Ans.(c) is correct
Sol.
28 9 11
45% of 1200 – 32% of 1500 = 15% of ? × × ×? = (10)2
252 22 3
(0.45 × 1200 – 0.32 × 1500) ÷ 0.15 = ? 100 × 3 × 22 × 252
?=
28 × 9 × 11
? = (540 – 480) ÷ 0.15 ? = 600
? = 400
Hence, option e. 35. Ans.(c) is correct
Sol.
29. Ans.(b) is correct 55
× 1400 + ?2 = 1728 − 282
Sol. 100
?2 = 676
(322 – 222) ÷ 32= ? ? = 26
(1024 – 484) ÷ 9 = ?
? = 540 ÷ 9 36. Ans.(b) is correct
? = 60
42 22.5
Hence, option b. × 500 + ×? = 426
100 100
22.5
×? = 216
100
? = 960
30. Ans.(d) is correct 37. Ans.(a) is correct
1 1 1 1 1 Sol.
5 +7 +4 =3 + ? +5
4 3 2 6 6 ? +2197=3481-681
1 1 1 1 1 ?=2800-2197
(5 + 7 + 4) + ( + + ) − 3 − 5 − − = ?
4 3 2 6 6 ? = 603
1 1 1 1 38. Ans.(d) is correct
8+ + + − = ? 540 15
4 2 3 3
+ 450 = × 3200
2+4 3 ? 100
?= 8+ =8 540
8 4 = 480 − 450
?
31. Ans.(c) is correct ? = 18
Sol.
5 7 1 39. Ans.(c) is correct
× × × 1600 = 260 + ? −499
2 8 28 63 10 15
? = 499 + 125 − 260 = 364 ? + − × =5
4 3 4
63 25
32. Ans.(a) is correct ?= 5 − +
2 2
Sol.
7
? = √5125 − 289 − 75 ?=
4
= √4761 = 69 ? = 1.75

33. Ans.(d) is correct


40. Ans.(b) is correct
For Free Study Material & Quizzes visit : www.mathsbyarunsir.com
For any queries or suggestions email us @ mathsbyarunsir003@gmail.com or whatsapp @ 8881331466
45
× 400 +?2 = 376 47. Ans.(a) is correct
100
?2 = 196 ? = √5125 − 289 − 75
? = 14 = √4761 = 69

41. Ans.(d) is correct 48. Ans.(b) is correct


12 1
×? + × 960 = 192 (? )2 = 16 × 7 + 361 + 11
100 8
12 = 484
×? = 192 − 120
100 ? = 22
12
×? = 72
100
? = 600 49. Ans.(b) is correct
252 + 26 + 420 = 121 + ?
42. Ans.(e) is correct ? = 577
20
15 ×? + × 450 = 360
100
50. Ans.(c) is correct
15 ×? = 360 − 90
15 ×? = 270
Sol.
? = 18 35% of 5000 – 322 + 3 = ?
0.35 × 5000 – 1024 + 3 = ?
43. Ans.(a) is correct 1750 – 1024 + 3 = ?
648 25 ? = 729
+ 199.5 = × 960
? 100
648
= 40.5 51. Ans.(a) is correct
?
? = 16 20% of 5000 – 25% of 3000 = ? × 52
0.20 × 5000 – 0.25 × 3000 = ? × 25
44. Ans.(c) is correct 1000 – 750 = ? × 25
79 41
? + + = 24 ? = 250/25 = 10
6 6
? = 24 − 20 = 4
52. Ans.(a) is correct
45. Ans.(d) is correct Sol.
Sol. 16 × 15 – 240 – 32 × 10 + 420 = ?2
48 ? 240 – 240 – 320 + 420 = ?2
× 525 + × 250 = 499
100 100 -320 + 420 = ?2
247 × 100
?= = 98.8 ? = 10
250

46. Ans.(c) is correct 53. Ans.(e) is correct


5 7 1 Sol.
× × × 1600 = 260+? −499
2 8 28 65% of 200 – 26% of 500 = ?
? = 499 + 125 − 260 0.65 × 200 – 0.26 × 500 = ?
= 364 ? = 130 – 130

For Free Study Material & Quizzes visit : www.mathsbyarunsir.com


For any queries or suggestions email us @ mathsbyarunsir003@gmail.com or whatsapp @ 8881331466
?=0
61. Ans.(c) is correct
54. Ans.(b) is correct 2 ? = (2)6 × (2 5 )2 ÷ 29
5 2? = 27
52 × 18– √2304 = (30)? − × 240
100 ?= 7
936 – 48 + 12 = (30)?
900 = (30)? 62. Ans.(b) is correct
?=2 Sol.
? = 572 ÷ 26 × 12– 200
55. Ans.(d) is correct ? = 64
Sol.
? = 9651 – 7506 63. Ans.(e) is correct
? = 2145 1 1
? = (5 + 4 − 4 ) + ( − )
2 3
56. Ans.(c) is correct 3−2 1
=5+( )=5
6 6
?2 = 135 + 15 − 141
?2 = 9 64. Ans.(e) is correct
?= 3 Sol.
49% of 180 - 70% of 120= 9 – ?
57. Ans.(a) is correct 88.2-84=9-?
? = 96 − 14 + 118 ?=4.8
? = 200
58. Ans.(a) is correct
65. Ans.(b) is correct
? × 9 = 34 + 47
Sol.
81
?= 280
9 3
√? = =8
35
?= 9
⇒? = 512

59. Ans.(e) is correct


66. Ans.(b) is correct
64 75
× 2500 + × 1600 = ? × 112 (? )2 = 1040 − 311 = 729
100 100
? = 27
? × 112 = 1600 + 1200
2800
?= 67. Ans.(b) is correct
112
? = 25 ? = 666 + 360 = 1026

60. Ans.(a) is correct 68. Ans.(d) is correct


? = 12.24
√360 − 421 + 350 + 272 = (? )2
69. Ans.(c) is correct
? = √289 = 17
? = 88 + 288 = 376

For Free Study Material & Quizzes visit : www.mathsbyarunsir.com


For any queries or suggestions email us @ mathsbyarunsir003@gmail.com or whatsapp @ 8881331466
4 3 19
8 +9 − ?= 6
70. Ans.(d) is correct 7 4 28
4 3 19 9
640 1 ? = (8 + 9 − 6) + ( + − ) = 11
= (3375 − 2700) × 7 4 28 14
? 33.75
640
= 20
? 78. Ans.(e) is correct
? = 32
8 − (1.11 + 2.111 + 3.1) = ?
71. Ans.(b) is correct
? = 1.679
85 79. Ans.(b) is correct
?2 = × 280 + 34 − 224 + 96
100 34 5
? = 108 × × = 34
?2 = 144 45 17
? = 12
80. Ans.(a) is correct
72. Ans.(b) is correct 0.9 × 10 = √?÷ 18 + 4
Sol. √? = 90
39+441-90=5× ? ? = 8100
5× ?=390
?=78 81. Ans.(b) is correct
73. Ans.(a) is correct 143
48 125 48 × 48 + = ?
(? )2 = × 800 + × 1200 − 120 √169
100 100 ? = 2304 + 11
(? )2 = 384 + 1500 − 120
? = 2315
(? )2 = 1764
? = 42 82. Ans.(c) is correct
?
11 × 75 =
74. Ans.(b) is correct 2
? = 1650
?
× 700 = 116 − 18
100
1148 83. Ans.(a) is correct
?=
7 7 36
13 − = ? +
? = 164 2 5
19 36
?= −
2 5
75. Ans.(b) is correct 3
?= 2
Sol. 10

15%of 256 + 35% of 280 = ?


84. Ans.(d) is correct
? = 38.4 + 98 = 136.4 ?
576 + 24 − = 225
5
76. Ans.(a) is correct ?
600 − 225 =
5
∴ (? )2 = 64 + 23 + 34 = 121 = (11)2 ? = 5 × 375
∴ ? = 11 ? = 1875

77. Ans.(a) is correct 85. Ans.(b) is correct


? = 3735 + 4565 + 6245 − 6432

For Free Study Material & Quizzes visit : www.mathsbyarunsir.com


For any queries or suggestions email us @ mathsbyarunsir003@gmail.com or whatsapp @ 8881331466
? = 8113 196 + ?3 × 7.5 = 256
60
?3 = 7.5
86. Ans.(e) is correct
?3 = 8
? 6 21
× × × 216 = 36 ?=2
3 7 36
?=1
93. Ans.(b) is correct
87. Ans.(d) is correct Sol.
Sol. ? = (7 + 2 −3)
3+2−6
24
? = 144 + 111 – 29 – 27 23
? = = 5 24
= 199

88. Ans.(d) is correct 94. Ans.(b) is correct

Sol. Sol.
40
? = 27 + 87 – 48 + 39 = 105 12.5 × ? + 1728 = 100 × 4445

12.5 × ? = 1778−1728
89. Ans.(e) is correct 50
? = 12.5
44 30
?= × 250 + × 270 + 169 ?=4
100 100
= 110 + 81 + 169
= 360 95. Ans.(d) is correct
Sol.
90. Ans.(d) is correct ?
× 144 + 192 = 324 − 24
100
Sol. ?
100
× 144 = 300 − 192
55
6.25 + 100 × 25 = ?2− 5 108
? = 144 × 100
6.25 + 13.75 = ?2− 5 ? = 75
2
? = 20 + 5
?=5 96. Ans.(b) is correct
Sol.
91. Ans.(b) is correct 13
156 + 376 − ? = 100 × 3000
Sol.
532 − ? = 390
121.75 + ? = 4 (216 −91)
121.75 + ? = 500 ? = 532 − 390
? = 142
? =500 −121.75
? = 378.25
97. Ans.(e) is correct
5612 – 1394
92. Ans.(b) is correct Sol. ? = × 100 = 570
740
Sol.
112 98. Ans.(a) is correct
× 175 + ?3 × 7.5 = 256
100
Sol. ? = 21 × 41 – 89 = 772

For Free Study Material & Quizzes visit : www.mathsbyarunsir.com


For any queries or suggestions email us @ mathsbyarunsir003@gmail.com or whatsapp @ 8881331466
8 + 8– 8– 9
𝑥 =6+( )
99. Ans.(c) is correct 36
1
Sol. 𝑥 = 6 + (– )
36
25 12 85 54 35
× +? = × 𝑥 = 536
3 5 6 17
5 × 4 + ? =9 × 5
? = 45 – 20 ? = 25 106. Ans.(b) is correct
100. Ans.(d) is correct Sol.
45 𝑥 = 10288 – 5220 – 1375 + 5364
× 80 + √841 + 𝑥 2 = 2121 ÷ 21
100
36 + 29 + x² = 101 𝑥 = 9057

𝑥 2 = 36
107. Ans.(e) is correct
𝑥=6
1350
?× = √5929 + √8281
112.5
101. Ans.(c) is correct ? ×12 = 77 + 91 = 168
36 + 3𝑥
+ 1 = 52 ⇒ ? = 14
23
36 + 3𝑥 + 23 = 52 × 23
3𝑥 + 59 = 1196 108. Ans.(a) is correct
3𝑥 = 1196 – 59 18750
= 36 × 11 + 59 × 6
3𝑥 = 1137 √?
18750
𝑥 = 379 ⇒ = 396 + 354
√?
18750
102. Ans.(e) is correct ⇒ √? = = 25
750
343 175 ? = 625
+ × 350 = 𝑥 2
2 100
𝑥 2 = 171.5 + 612.5 109. Ans.(a) is correct
2
𝑥 = 784 729 216 × 72
3? = × = 81
𝑥 = 28 243 576
3? = 34 ⇒? = 4
103. Ans.(d) is correct
23(24 + 47– 54) = 𝑥 110. Ans.(e) is correct
𝑥 = 23 × 17 ? = 114 × 8 + 24 − 144
𝑥 = 391 = 792

104. Ans.(c) is correct 111. Ans.(b) is correct


6 2 3 1 1
× 650 + 320 + 51 = 𝑥 ? = (3 + 4 + 2 − 5) + ( + + − )
5 3 5 2 6
780 + 320 + 51 = 𝑥 20 + 18 + 15 − 5 48 3
=4+( )= 4+ =5
𝑥 = 1151 30 30 5

105. Ans.(e) is correct 112. Ans.(e) is correct


2 2 2 1 Sol.
𝑥 = 11 + 12– 13– 4 ( + – – )
9 9 9 4

For Free Study Material & Quizzes visit : www.mathsbyarunsir.com


For any queries or suggestions email us @ mathsbyarunsir003@gmail.com or whatsapp @ 8881331466
? = (1728) ÷ (144) 120. Ans.(d) is correct
? = 12 2.5
?% of 125 + 225 = 256 – 100 × 640
?
× 125 = 256 – 16 – 225
113. Ans.(d) is correct 100
?
× 125 = 15
100
15×100
?= 125
? = 12
114. Ans.(e) is correct
121. Ans.(c) is correct
17 70 40
?= × × × 450 24
5 17 7 1728 + ×? = 1830
= 36000 100
24
×? = 1830 – 1728
100
115. Ans.(a) is correct ?=
102×100
24
Sol.
? = 425
? 3 4
5
× 125 = 5 × 7 × 3430 122. Ans.(a) is correct
? × 25 = 1176 50
? 3 4 7.8 + × 64.4 = 49 – ?2
5
× 125 = 5 × 7 × 3430 100
? × 25 = 1176 7.8 + 32.2 = 49 – ?²
⇒ ? = 47.04 ?² = 49 – 40
?² = 9
116. Ans.(b) is correct ?=3
Sol.
13 21 24 123. Ans.(b) is correct
? = 100 × 450 + 100 × 250 − 100 × 80
? = 6/5 of 8/7 of 420
= 58.5 + 52.5 − 19.2 = 91.8 ? = 6/5 × 8/7 × 420
? = 576
117. Ans.(e) is correct
28 9 11
× × ×? = (10)2 124. Ans.(d) is correct
252 22 3
100 × 3 × 22 × 252 1 50 ?
?= Sol. 1350 × 25 × 100 × 1700 = 100 × 2700
28 × 9 × 11
? = 600 ⇒ ? = 1700

118. Ans.(d) is correct 125. Ans.(e) is correct


Sol. 26
Sol. 1330 + 100 × 1800 – 1190 = ? × 50 – 1742
4 ×? = 231 + 424 ⇒ ? = 163.75 1330 + 468 – 1190 + 1742 = ? × 50
2350
=?
50
119. Ans.(a) is correct
? = 47
Sol.
? = 9 + 28 + 4 – 7 = 34
126. Ans.(a) is correct
8 80
Sol. 1900 – (2600 × 13
+ 1740)+ ? =
100
× 700

For Free Study Material & Quizzes visit : www.mathsbyarunsir.com


For any queries or suggestions email us @ mathsbyarunsir003@gmail.com or whatsapp @ 8881331466
1900 – 3340 + ? = 560
? = 2000 135. Ans.(a) is correct
8104.32 100
Sol. ? = × 120 = 67
100.8
127. Ans.(d) is correct
? = 104 70 50
× 150 + ×( × 180 + 20 ) – 119 136. Ans.(b) is correct
100 100 3×100
7
? = 156 + 10 (50) − 119 Sol. ?2 = 21025 − 3136 − 3869 − 8836 = 5184
? = 37 + 35 = 72 Or, ? = 72
128. Ans.(b) is correct 137. Ans.(c) is correct
90 21 1024×4
526 + 324 + × 2400 + 640 = (? )3 − 100 × 2000 − 26 Sol. ? = 256×0.5 = 32
100
850 + 2160 + 640 = (?)³ – 420 – 26
(?)³ = 4096 138. Ans.(e) is correct
? = 16 42875×12
Sol. ? = 70×25×58.8 = 5

129. Ans.(b) is correct


139. Ans.(b) is correct
Sol.
203 = 8000, 33 = 27, 73 = 343
35 ÷ 7 × 342 =? 3 3 3
? = √8000 + √27 − √343
Or, ? = 1710
= 20 + 3 - 7 = 16

130. Ans.(a) is correct


140. Ans.(d) is correct
805 + 187 + 2? = 1000
14400 ÷ 36 + 15600 ÷ 12 + 450 = 1800+?
?
Or, 2 = 8 400 + 13000 + 450 = 1800+?
Or, ? = 3 2150 = 1800 + ?
? = 2150 − 1800
131. Ans.(d) is correct = 350

18? = 187.9 × 180.1 ÷ 184


Or, ? = 7.9 + 0.1 - 4 = 4 141. Ans.(a) is correct
750 + 5880 − 6890 = 9000 − ?
132. Ans.(b) is correct ? = 9000 − 6440
35 22 13 36 38 18 245 = 2560
= + + − − =
? 7 5 5 7 35 35
35×35
Or, ? = =5 142. Ans.(c) is correct
245
Sol.
133. Ans.(e) is correct 32× 25 + 44 × 18 + 348 ÷ 6 = ?
Sol. ? = 8002 + 792 + 58
88.2 − 4 = 10−? = 1650
Or, ? = -74.2
143. Ans.(c) is correct
134. Ans.(b) is correct
√1225 × 28 + 203 × 7 = (? )2
33670
Sol. ? = = 65 (? )2 = 35 × 28 + 203 × 7
518

For Free Study Material & Quizzes visit : www.mathsbyarunsir.com


For any queries or suggestions email us @ mathsbyarunsir003@gmail.com or whatsapp @ 8881331466
= 980 + 1421 ? = 19
= 2401 151. Ans.(d) is correct
? = √2401 8 × 16 × 16 ×
= (2)?−2
8×8
= 49
⇒ (2)?−2 = (2)9 ⇒? − 2 = 9
⇒? = 11
144. Ans.(a) is correct
152. Ans.(e) is correct
Sol. 39 38
× × 100 = ? × 10
? = (28 × 16 - 28)/30 = (448 - 28)/30 = 420/30 = 14 13 19
? = 60
145. Ans.(a) is correct
153. Ans.(a) is correct
Sol.
3 7
? = (15 × 680/100) + (45 × 340/100) × 250 + × 750 − 75 = (? )2
20 20
= (10200/100) + (15300/100) 75
[1 + 7] − 75 = (? )2
2
= 102 + 153 = 255
? = 15
154. Ans.(b) is correct
146. Ans.(c) is correct
60 − 10
Sol. =?
2.5
(?)2 = 382 - 39 - 93 ? = 20
= 1444 - 39 - 729 = 676 155. Ans.(d) is correct
So, ? = 26 10000 = ? × 40
? = 250
147. Ans.(b) is correct 156. Ans.(a) is correct
Sol. Sol.
9× 3 + 1107+42 = ?
?2 = (3 × 60 × √144)/(∛3375) ? = 1176
= 3 × 60 × 12/15 157. Ans.(e) is correct
= 144 = 122 Sol.
So, ? = 12 299+999+302 = ?
? = 1600
148. Ans.(e) is correct 158. Ans.(b) is correct
Sol. Sol.
493+287-334+54 = ?× 5
? ≈ (40 × 400/100) + (15 × 620/100) ? =100
= (16000/100) + (9300/100) 159. Ans.(e) is correct
= 160 + 93 = 253 Sol.
√5776 − √1444 + √729 = 43 + ?
149. Ans.(c) is correct 76 − 38 + 27 = 43+?
3 2 1 23 ? = 65 − 43 = 22
2 +3 −1 −1 = ?
5 3 2 30 160. Ans.(a) is correct
13 11 3 53
?= + − − Sol.
5 3 2 30
?= 3 78 × 26 ÷ 6 + 1262 = 1311 + (? )2
150. Ans.(b) is correct 2028 ÷ 6 + 1262 = 1311 + (? )2
3×4−2+9= ? 338 + 1262 = 1311 + (? )2
For Free Study Material & Quizzes visit : www.mathsbyarunsir.com
For any queries or suggestions email us @ mathsbyarunsir003@gmail.com or whatsapp @ 8881331466
Sol.
(? )2 = 1600 − 1311 = 289
728 24
? = √289 = 17 × 15 + × 550– 38 = 484
? 100
728 × 15
161. Ans.(a) is correct = 484 + 38– 132
?
Sol. 728 × 15
?=
17 + 13 + 47 = ? 390
? = 77 ? = 28
162. Ans.(b) is correct 169. Ans.(a) is correct
Sol. Sol.
?3 = 8 + 4 × 14 = 43 36 ×? +33 × 4 + √441 = (21)2
?= 4 36 ×? = 441– 21– 132
163. Ans.(d) is correct 288
?=
36
? = 270 − 89 = 181 ?=8
164. Ans.(d) is correct 170. Ans.(b) is correct
19 29 27 Sol.
? =( + − )
12 12 16
8
37 568 + 330 – (12)2 = ×?
?= 100
16 8
165. Ans.(e) is correct ×? = 898– 144
100
Sol. 754×100
?= 8
? = 132 + 264 = 396 ? = 9425
166. Ans.(b) is correct
171. Ans.(b) is correct
Sol.
26 ?
× 250 + × 640 = 225
42 100 100
× 650 + 243– √? = 2 × (4)4
100 65 + 6.4×? = 225
273 + 243 – √? = 2 × 256 6.4×? = 160
√? = 516 – 512 160
?=
6.4
√? = 4
? = 25
? = 16
172. Ans.(c) is correct
167. Ans.(d) is correct
Sol.
Sol.
299 + 121 - ? = 361
?
× 750 + (15)2 = (8)2+356 ? = 420 – 361
100
? ? = 59
× 750 + 225 = 64 + 356
100 173. Ans.(d) is correct
?
× 750 = 195 Sol.
100
? 65
195 × 100 + × 260 = 199
?= 24 100
750 ?
? = 26 = 199 − 169
24
168. Ans.(e) is correct ? = 30 × 24
? = 720
For Free Study Material & Quizzes visit : www.mathsbyarunsir.com
For any queries or suggestions email us @ mathsbyarunsir003@gmail.com or whatsapp @ 8881331466
174. Ans.(e) is correct Sol.
Sol. 1484 ÷ 28 + 1462 ÷ 34 − 12 × 7 = ?
25
2.46 × 15 + × 92.4 = 15 × ? ? = 53 + 43 − 84 = 12
100
36.9 + 23.1 = 15 × ? 182. Ans.(c) is correct
60 42.5 × 15 + 37.5 × 25 = 1420 + ?
?=
15
637.5 + 937.5 = 1420 + ?
?=4
? = 1575 − 1420 = 155
175. Ans.(d) is correct
183. Ans.(a) is correct
Sol.
Sol.
1387 – 433 + ? = 961
2450 + 3760 − 3830 = 6000 − ?
? = 961 – 954 2380 = 6000 − ?
?=7 ? = 6000 − 2380 = 3620
176. Ans.(b) is correct
Sol. 184. Ans.(b) is correct
128 4 Sol.
+ × 4 = ? + 10
2 2 203 = 8000, 33 = 27, 73 = 343
64 + 8 = ? + 10
? = 3√8000 + 3√27 − 3√343
? = 62 = 20 + 3 - 7 = 16
177. Ans.(c) is correct
Sol.
185. Ans.(b) is correct
11
+ 9+? = 27 Sol.
11
We note that 33.33% = 3/9, 66.66% = 6/9
1 + 9+? = 27
So, ? = 189 x (3/9) + 99 x (6/9)
? = 17
178. Ans.(b) is correct = (9 x 21) x (3/9) + (9 x 11) x (6/9)

Sol. = (3 x 21) + (6 x 11) = 129

(3)2 × (3)6 × ((3)2 )2 ÷ (33 )2


186. Ans.(c) is correct
32+6+4 312
⇒ ⇒ 6 = 36 (?)2 = 152 + 122 - 8
36 3
179. Ans.(b) is correct = 225 + 144 - 8
Sol. = 361
123 + 447 − 170 + 500 = ? − 200 ? = 19

570 − 170 + 500 + 200 = ?


187. Ans.(b) is correct
? = 1100
180. Ans.(b) is correct ? = √961 + √1225 + √576
Sol. = 31 + 35 + 24
196 + 179 + 25 = (? )2 = 90
(? )2 = 400
? = 20 188. Ans.(d) is correct
? = (5 x 340/100) + (25 x 780/100)
181. Ans.(a) is correct
For Free Study Material & Quizzes visit : www.mathsbyarunsir.com
For any queries or suggestions email us @ mathsbyarunsir003@gmail.com or whatsapp @ 8881331466
= (1700/100) + (19500/100)
= 17 + 195 = 212 196. Ans.(c) is correct
Sol.
189. Ans.(d) is correct 0.075×88-1.9=?
?=6.6-1.9
512 ÷?× 6 = 192
?=4.7
512 × 6
?= = 16
192
190. Ans.(a) is correct 197. Ans.(d) is correct
Sol. Sol.
432 ÷ 48 × 602 ÷ 86 = ? 4
9×7=? × 1080 ÷ 5 + 28 = ?
9
?=63 ? = 96 + 28 = 124

191. Ans.(b) is correct 198. Ans.(c) is correct


3
×? − 28 = 89 ≈ 65% × 480 − ? + 175 = 350
8
? = 312 ≈ ? = 137

192. Ans.(a) is correct 199. Ans.(c) is correct

Sol. ? = 3045 ÷ 87 - 54
3 5 5 3 = 35 - 54
1 5 × 6 × 8of2 ×? = 150
= -19
8 5 15
× × ×? = 150
5 6 16
200. Ans.(a) is correct
? = 120
Sol.

193. Ans.(b) is correct ? = (12 × 201 × 264)/(67 × 66)


= 12 × (201/67) × (264/66) = 12 × 3 × 4 = 144
2.16 × 31.25
27
? = 2.5 201. Ans.(c) is correct
Sol.
194. Ans.(d) is correct 712 = 5041, 242 = 576, 642 = 4096
Sol. ? = √5041 + √576 - √4096
6080 ÷ 19 = ? %of 256 = 71 + 24 - 64 = 31
256
320 = ? × 202. Ans.(c) is correct
100
? = 125 ? = 25(0.26 + 0.24) = 250.50 = 251/2
Since, 251/2 = 5, ? = 51 = 5
195. Ans.(e) is correct 203. Ans.(b) is correct
Sol. Sol.
48 4 ? = 342 - 382 + 312
× 625 × = ?
100 3 = 1156 - 1444 + 961
? = 400

For Free Study Material & Quizzes visit : www.mathsbyarunsir.com


For any queries or suggestions email us @ mathsbyarunsir003@gmail.com or whatsapp @ 8881331466
= 673 ∴ ? = 13

204. Ans.(d) is correct 211. Ans.(c) is correct


Sol. 180 × 170
= 918 + 612
183 = 5832, 143 = 2744, 63 = 216 16 ×?
180 × 170
3 3
? = √5832 − √2744 − √216
3 ?=
16 × 1530
= 18 - 14 - 6 = -2 ? = 1.25
205. Ans.(e) is correct 212. Ans.(e) is correct
Sol. Sol.
? = (29 × 31 - 24)/35 (?)² = 621 + 144 + 100 – 576
= (899 - 24)/35 (?)² = 289 = (17)²
= 875/35 = 25 ? = 17
206. Ans.(c) is correct
? = 362 - 282 - 392 213. Ans.(e) is correct
= 1296 - 784 - 1521 = -1009 ? = (780) ÷ (2820 – 2780)
? = 780 ÷ 40 = 19.5
207. Ans.(d) is correct
Sol,. 214. Ans.(c) is correct
(?)2 = 332 - 36 - 93 25
× 360 + 50 = ?2 + 40
100
= 1089 - 36 - 729 = 324
?2 = 140 − 40
So, ? = 18
? = 10
215. Ans.(d) is correct
208. Ans.(e) is correct 40
324 +?2 = × 1450
Sol. 100
324 +?2 = 580
We note that 55.55% = 5/9, 57.14% = 4/7
So, ? = 45 × (5/9) + 91 × (4/7) ?2 = 256

= (9 × 5) × (5/9) + (7 × 13) × (4/7) ? = 16

= (5 × 5) + (4 × 13) = 77
216. Ans.(a) is correct
Sol.
209. Ans.(b) is correct
1 1 1 1 1 56 13
? +2 + 1 = 1 + 4 + 7 + 512 = × 4000
2 4 8 2 4 ? 100
1 1 1 1 1 56
? = (1 + 4 + 7 – 2 – 1) + [ + + – – ] = 520 − 512
8 2 4 2 4 ?
1 73
?= 9 = ?= 7
8 8
210. Ans.(d) is correct
Sol. 217. Ans.(b) is correct

69 + (5)² × (2)² = (?)² √841 + √1296 − √1024 = √?

169 = (?)² 29 + 36 − 32 = √?
(?)² = (13) ² √? = 33

For Free Study Material & Quizzes visit : www.mathsbyarunsir.com


For any queries or suggestions email us @ mathsbyarunsir003@gmail.com or whatsapp @ 8881331466
225. Ans.(b) is correct
? = (33)2
= 1089
Sol.
36 15
?= ( × 5000) ÷ ( × 1800 + 90)
100 100
218. Ans.(b) is correct = (1800) ÷ (270 + 90)
?3 × 6 + 8 = 170 =
1800
=5
360
?3 × 6 = 162
?3 = 27 226. Ans.(c) is correct
?= 3 Sol.
219. Ans.(e) is correct
525
+ 2300 + 40 = 1810 + 1580
51 7 ?
+ − ?= 4 525
12 4 = 3390 – 2340
?
51 + 21 525
?= −4 ?= = 0.5
12 1050
?= 6 − 4 = 2
227. Ans.(a) is correct
220. Ans.(a) is correct Sol.
1 256 + 441 – 169 + (?)² = 852
Sol. 34 × 2 × 31 × 31 = ? − 30.
(?)² = 852 – 528 = 324
? = 68 + 30
? = ±18
? = 98

228. Ans.(a) is correct


221. Ans.(c) is correct
Sol.
Sol. 101 + 154 + 26 = 281
1 1 1 2 1
? = (12 – 7) + ( + + – – )
5 8 2 5 4
222. Ans.(b) is correct 8 + 5 + 20 – 16 – 10 7
=5+( ) =5
1 1 1 40 40
Sol. 15.5 × 28 × 112 + 1230 × 240 = ? × 5
31 41 1
+ =?× 229. Ans.(b) is correct
8 8 5
72 Sol.
?= ×5
8 14 × 21 14 × 21
= 4 ?= = 4.9
? = 45 15 ×? 15 × 4

223. Ans.(d) is correct 230. Ans.(a) is correct


1
13 Sol. 2652 − 441 + 928 – 15 × 3375 = ? + 961
Sol. 9 × 17 + × 378 – 241 = ?
9
2652 − 441 + 928 – 225 – 961 = ?
? = 153 + 546 – 241
? = 1953
? = 458
231. Ans.(c) is correct
224. Ans.(c) is correct
Sol. 9240 × 1? = 330
Sol. 18 × 928– 17 × 980 + 144 × 14 = ?
? = 28
116 – 140 + 36 = ?
? = 12
232. Ans.(e) is correct

For Free Study Material & Quizzes visit : www.mathsbyarunsir.com


For any queries or suggestions email us @ mathsbyarunsir003@gmail.com or whatsapp @ 8881331466
1 4845 + (?)² = 4989
Sol. 17 × 4913 − 225 = (? )2
(? )2 = 64
(?)² = 144
?=8 ? = 12
241. Ans.(c) is correct
233. Ans.(b) is correct Sol.
Sol. ? × 1211 = 24220 1
5760 × × 15 =?
45
? = 20
? = 1920
242. Ans.(b) is correct
234. Ans.(d) is correct
Sol.
1
Sol. 240 × 300 × 240 = ? + 240
1863 – 187 = ?
? = 300 – 240 ? = 1676
? = 60 243. Ans.(c) is correct
Sol.
235. Ans.(b) is correct
25639 – 8484 = ?
5
Sol. 4 × 92– 16+? = 101 ? = 17155
99+? = 101 244. Ans.(b) is correct
?=2 Sol.
1 3 2 3
? = (2 + 3 – 1 + 3) + ( + – + )
236. Ans.(d) is correct 4 5 3 4

Sol. (3 + 4 + 9 + 6) + (12 + 34 + 34 + 78) = ? + 23 + 19


15 + 36 – 40 + 45 56
8 ?= 7 + ( )=7+
60 60
23 23
22 + = ? + + 19 14 14
8 8
=7+ =7
?=3 15 15
245. Ans.(a) is correct
237. Ans.(e) is correct 27
Sol. 100 × 450 – 76.5 = (? )% of 375
Sol. 34 − 12 + 178 − (? )2 = 45 + 55
121.5 – 76.5 = ? × 3.75
200 − 100 = (? )2
45
(? )2 = 100 =?
3.75
? = 10
? = 12
246. Ans.(d) is correct
238. Ans.(a) is correct Sol.
Sol. 3996 − 3721 − (? )2 = 3150 − 100 − 3000
5287 – 2482 = ?
275 − (? )2 = 50
? = 2805
(? )2 = 225
247. Ans.(a) is correct
? = 15
3 21 11
239. Ans.(e) is correct ? = 12 + ×
5 5 3
Sol. 14 + 289 − 3 = ? − 632 63 77
?= +
? = 932 5 5
140
240. Ans.(a) is correct ?= = 28
5
Sol. 248. Ans.(b) is correct
For Free Study Material & Quizzes visit : www.mathsbyarunsir.com
For any queries or suggestions email us @ mathsbyarunsir003@gmail.com or whatsapp @ 8881331466
257. Ans.(c) is correct
24 × √? = 864
Sol.
√? = 36
32.12 + 34.18 + 27.7 = 47 × ?
? = 1296
47 × ? = 94
249. Ans.(c) is correct
?=2
1
? = 1615 × 258. Ans.(e) is correct
50 × 0.85
? = 38 Sol.
250. Ans.(d) is correct 1 1
384 × × = (2)?
4 3
195 (2)? = 32
⇒ × 78 = ?
1000 (2)? = (2)5
39 × 39
⇒ ?= ? = 15.21 ?=5
100
251. Ans.(b) is correct 259. Ans.(a) is correct
Sol.
168 625 1
× = ×? 1 3 2 1 3 3
10 100 5 ? = (1 + 2 – 5 + 4) + (2 + 4 – 3 + 6)?= 2 + 4 ? = 2 4
? = 21 × 5 × 5 260. Ans.(b) is correct
? = 105 × 5 Sol.
? = 525
? = 6526 – 596 – 4683 – 1200
252. Ans.(a) is correct
? = 47
441 × 49 × 9 = (?)² 261. Ans.(a) is correct
? = 21 × 7 × 3 Sol.
? = 441
2 3 4 125
253. Ans.(c) is correct × × × ×? = 1112
3 5 5 100
Sol. ? = 556 × 5
4 × 3 × 121 = ? ? = 2780
? = 1452 262. Ans.(d) is correct
254. Ans.(e) is correct 1
1
780 × + (? )2 = 1326
38 × + 3.5 × 4 = (? )2
26
19 (? )2 = 1326 − 30
2 + 14 = (?)²
(? )2 = 1296
? =4
? = 36
255. Ans.(a) is correct
263. Ans.(c) is correct
3375 + 11 = (?)² – 95 Sol.
(?)² = 3481
8153.14 - ? = 7592.14
? = 59
? = 561
256. Ans.(b) is correct
264. Ans.(b) is correct
Sol.
60
× 300 + ? = 256
100
180 + ? = 256
? = 76

For Free Study Material & Quizzes visit : www.mathsbyarunsir.com


For any queries or suggestions email us @ mathsbyarunsir003@gmail.com or whatsapp @ 8881331466
Sol. Or, ? = 5
3 5 1 19
(1 + 2 − 3) + + − = ? ×
5 6 6 15
18 + 25 − 5 19 272. Ans.(b) is correct
0+ =?×
30 15
38 15 Sol. 2? = 572 ÷ 26 = 22 × 12 = 264 – 200 = 64 = 26
?= ×
30 19 Or, ? = 6
?=1
265. Ans.(b) is correct 273. Ans.(a) is correct
1 7 5
Sol. Sol. ? = (4 + 1 – 2) + (2 + 12 − 6)
136 − 96 + 61 101 6 + 7 − 10 1
?= = = 0.5 =3+( )=3
36 + 45 + 121 202 12 4
274. Ans.(e) is correct
Sol. 36% of 245 = (40 – 4)% of 245
266. Ans.(d) is correct 2 4 × 245
= × 245 − = 98 − 9.8 = 88.2
1 5 100
464 × + 4.5 = ? 2
16 × 2.32 40% of 210 = 5 × 210 = 84
100
+ 4.5 = ? Difference = 88.2 – 84 = 4.2
8
? = 12.5 + 4.5 ? = 10 – 4.2 = 5.8
? = 17 275. Ans.(a) is correct
267. Ans.(a) is correct 2
Sol. (40% = 5) of 265 + (35% of 180
(5863 − 52) × 0.5 = ? + 2601 = 180% of 35) = 50% of ?
5811 × 0.5 – 2601 = ? 9
⇒ 106 + (180% = 5) of 35 = 50% of ?
? = 2905.5 – 2601
? = 304.5 ⇒ 106 + 63 = 50% of ?
268. Ans.(c) is correct ⇒ ? = 169 × 2 = 338
? 145 276. Ans.(e) is correct
1231 − × 500 = × 780 1
100 100 Sol. Hint: 460 × 15 = 460 × 10 + 2 × 4600 – 100 = 4600 + 2300 – 100
1231 − 5 ×? = 1131
= 6900 – 100 = 6800
1
? = 100 × 5 277. Ans.(d) is correct
? = 20 Sol. 5163 + 3209 = 8372 – 4018 = 4354
269. Ans.(e) is correct 278. Ans.(b) is correct
Sol. 21 10
Sol. × + ? = 24
5 3
41
× 208 + 786 = 2000 − ? ? = 24 – 14 = 10
8
279. Ans.(c) is correct
41 × 26 + 786 = 2000 - ?
Sol. 4848 ÷ 24 = 202,
? = 2000 – 1066 – 786
? = 148 202 × 11 = 2222, 2222 – 222 = 2000
270. Ans.(a) is correct 280. Ans.(a) is correct
1750
Sol. √360 − 450 + 379 = √289 = 17 × 50 + 111 ≈ (? )2
350
271. Ans.(c) is correct
? = 19
Sol. 3? = (32 )3 × (34 )3 = 36 × 38 ÷ 39 = 35 281. Ans.(a) is correct

For Free Study Material & Quizzes visit : www.mathsbyarunsir.com


For any queries or suggestions email us @ mathsbyarunsir003@gmail.com or whatsapp @ 8881331466
290. Ans.(b) is correct
2 15 9
?= × × + 88 × 26 + 562
15 18 7 3840 1440 1330
1 √ + −
= + 2288 + 562 60 40 70
7
1 = √64 + 36 − 19
= 2850
7 = √81
282. Ans.(e) is correct
=9
3 1 100
× × 920 × = ? 291. Ans.(c) is correct
4 5 650
Sol.
? = 21.23
4200 525
𝟔𝟑𝟎𝟎𝟎 25 × 18 + − = 740−?
𝟐𝟖𝟓 × 𝟓𝟐𝟓 40 105
= ? −𝟖𝟔
𝟔𝟎𝟎 450 + 105 − 5 = 740 − ?
283. Ans.(c) is correct
? = 740 − 550
285 × 63000
? = 86 + = 190
525 × 600
292. Ans.(d) is correct
= 143
Sol.
284. Ans.(a) is correct
3845 + 43820 + 2640 − 5965 = (? )2
112 100
? = 4 × 8888 × × (? )2 = 10865 − 5965
100 110
= 36198.4 = 4900
? = √4900
285. Ans.(b) is correct = 70
Sol. 293. Ans.(b) is correct
5 × 18 × 500 1 Sol.
?= =
45 × 3276000 3276 400 ÷ 20 × 35 + 6666 ÷ 33+? = 1100
286. Ans.(a) is correct
20 × 35 + 202 + ? = 1100
Sol.
13 × 5700 26 ? = 1100 − ( 700 + 202)
?= =
15 × 8550 45 = 1100 − 902
287. Ans.(d) is correct = 198
600 294. Ans.(b) is correct
?= [1716 − 875]
50 Sol.
600
= 841 × = 10092 28 × 14.5 + 1680 ÷ 15 + 445 = 1000−?
50
288. Ans.(a) is correct 406 + 112 + 445 = 1000 − ?
Sol. 963 = 1000 − ?
5 ? = 1000 − 963 = 37
150+ 270 – 105 × 6 × 3 = ? − 105 − 232 × 7
295. Ans.(c) is correct
⇒? = 420 − 1050 + 105 + 1624
Sol.
= 1099
289. Ans.(c) is correct √(256) × √169 + 3600 ÷ 12 = 800 − ?

1396 + 412 − 2704 ≈? − (31)2 16 × 13 + 300 = 800 − ?

? ≈ 961 − 896 ≈ 65 208 + 300 = 800 − ?

For Free Study Material & Quizzes visit : www.mathsbyarunsir.com


For any queries or suggestions email us @ mathsbyarunsir003@gmail.com or whatsapp @ 8881331466
Sol.
? = 800 − 508
? = (1728) ÷ (144)
= 292
296. Ans.(a) is correct ? = 12
Sol. 303. Ans.(a) is correct
Sol.
? = 37.5 × 14 + 800 − (26)2 + 136
? = 525 + 800 − 676 + 136 ? = √1714 + 260 – 950
? = 1325 − 540
? = √1024 = 32
= 785
304. Ans.(c) is correct
(52 )2.7 × 54.2 ÷ 56.4 = (52 )?
297. Ans.(d) is correct 55.4 × 54.2
⇒ = 52×?
56.4
5430+3780-6430=2260+?
9210-6430=2260+? ⇒ 55.4+4.2 – 6.4 = 52×?
2780=2260+? ⇒ 53.2 = 52×?
?=2780-2260
=520 ⇒ 2 ×? = 3.2
298. Ans.(e) is correct 3.2
⇒ ?= = 1.6
Sol. 2
2160÷12+5740÷14-3150÷15+?=400 305. Ans.(d) is correct
180+410-210+?=400 √400 × 42
590-210+?=400 ?=
7
?=400-380
= 20 × 6 = 120
=20
306. Ans.(d) is correct
299. Ans.(a) is correct
Sol.
Sol.
? 60 – 40
√3481 × 7 + √5625 × 4 = 500 + ? =
5 2
59×7+75×4=500+?
⇒ ? = 5 × 10
413+300=500+?
?=713-500 = 50
=213 307. Ans.(c) is correct
300. Ans.(a) is correct 621
Sol. √? = × 2 – 37
27
1 = 46 – 37 = 9
220 + 340 + ? = 18 × 2700 + 400
⇒ ? = 9 × 9 = 81
? = 150 + 400 – 220 – 340
308. Ans.(d) is correct
? = –10
Sol.
301. Ans.(c) is correct
250 × 36 50 × 18
28 9 11 × =? +10
× × ×? = (10)2 100 100
252 22 3
100 × 3 × 22 × 252 ⇒ 810 = ? + 10
?=
28 × 9 × 11 ∴ ? = 810 – 10 = 800
? = 600 309. Ans.(b) is correct
302. Ans.(e) is correct

For Free Study Material & Quizzes visit : www.mathsbyarunsir.com


For any queries or suggestions email us @ mathsbyarunsir003@gmail.com or whatsapp @ 8881331466
49– 36 18 315. Ans.(d) is correct
= 48 ?
26 ? × 525 + × 250 = 499
100 100
13 18
⇒ = 247 × 100
26 ? ?= = 98.8
250
1 18 316. Ans.(c) is correct
⇒ =
2 ? Sol.
⇒ ? = 2 × 18
5 7 1
= 36 × × × 1600 = 260 + ? – 499
2 8 28
310. Ans.(c) is correct ? = 499 + 125 – 260
Sol. = 364
√(256) × √169 + 3600 ÷ 12 = 800 − ? 317. Ans.(a) is correct
16 × 13 + 300 = 800 − ?
? = √5125 – 289 – 75
208 + 300 = 800 − ?
= √4761 = 69
? = 800 − 508
318. Ans.(b) is correct
= 292
Sol.
311. Ans.(a) is correct
Sol. (?)² = 16 × 7 + 361 + 11
= 484
? = 37.5 × 14 + 800 − (26)2 + 136
? = 525 + 800 − 676 + 136 ? = ±22.
? = 1325 − 540 319. Ans.(a) is correct
= 785 4750
250 + 283 – 157 + 24 + 100 = ?
4750
312. Ans.(d) is correct ?= = 9.5
500
Sol. 320. Ans.(b) is correct
5430+3780 - 6430 = 2260 + ? 30
9210 - 6430=2260 + ? Sol. 130% × 850 – 175% × 460 = × ?
100
2780 = 2260 +? 30
?=2780 -2260 ? × = 1105 − 805 = 300
=520 100
? = 1000
313. Ans.(e) is correct
321. Ans.(d) is correct
Sol.
40 72 65 121
2160÷12+5740÷14-3150÷15+ ? = 400 Sol. ? × = 11 × ×
3 9 13
180+410-210+ ? =400
3
590-210+? =400 ?= × 8 × 5 × 11 = 33
?=400- 380 40
=20 322. Ans.(c) is correct
314. Ans.(a) is correct 1872
Sol. √? = 12×6.5 = 24
Sol.
? = 242 = 576
√3481 × 7 + √5625 × 4 = 500 + ? 323. Ans.(b) is correct
59×7+75×4=500+ ?
Sol. 3? = √1800 − 1300 + 229
413+300 =500+ ?
?=713-500 = √729 = 27 = 33
=213
⇒ ?= 3

For Free Study Material & Quizzes visit : www.mathsbyarunsir.com


For any queries or suggestions email us @ mathsbyarunsir003@gmail.com or whatsapp @ 8881331466
324. Ans.(e) is correct 312 × 100
2 ?= = 800
Sol. ? = 8973 − 2378 + 1577 − 4572 39
332. Ans.(e) is correct
?2 = 3600
5
⇒ ? = 60 ?2 = 40% of × 352
11
325. Ans.(b) is correct
2 5
2106×100×100 ?2 = × × 352 = 64
Sol. ? = 5 11
52×180×45
= 50 ⇒ ?= 8
333. Ans.(a) is correct
Sol.
11 195 132
? × = ×
326. Ans.(e) is correct 15 108 65
1 2 1
?=5
Sol. (7 + 6 − 4) + (9 + 3 − 2) = ? − 5 + 4 − 4 − 334. Ans.(c) is correct
2 1 3
+4−4 Sol.
9
1 2 2 1 1 3
?= 9+5+ + + − − + ? = √60% 𝑜𝑓 850 + 80% 𝑜𝑓 550 − 83 × 2
9 3 9 2 4 4
= 15
= √510 + 440 − 166
327. Ans.(e) is correct
252 276 55
= √784 = 28
Sol. ? = × 630 × 12 × 18 335. Ans.(e) is correct
23
= 396 Sol.
328. Ans.(b) is correct
(√1444 + √676) 38 + 26 64
Sol. ? = 33 + 93 = 126 ?2 = = = = 16
4 4 4
329. Ans.(c) is correct ?=4
512 64 336. Ans.(d) is correct
(2)?+2 = × ×8
32 128 Sol.
29 × 26 × 23 64 × 288 × 16
= 5 7
= 29+6+3−5−7 ?2 = = 32 = 25
2 ×2 256 × 36
6
=2 ?=5
? +2 = 6 ⇒ ? = 4 337. Ans.(b) is correct
330. Ans.(d) is correct Sol.
Sol. 3
90% 𝑜𝑓 ? = √576 ÷ √64 × √729
? = (6² - 3²)× (8² - 4²) ÷ (10.5² - 1.5²) 24 27
?= × × 10
(6 + 3) × (6 − 3) × (8 + 4) × (8 − 4) 4 9
?= ? = 180
(10.5 + 1.5) × (10.5 − 1.5)
9 × 3 × 12 × 4 338. Ans.(a) is correct
?= = 12 Sol.
12 × 9
331. Ans.(b) is correct 43 108
? = 40% of 320 + ×
Sol. 18 8
= 128 + 48 = 176
35% 𝑜𝑓 540 + 410% 𝑜𝑓 30 = 39% 𝑜𝑓 ?
339. Ans.(d) is correct
39
189 + 123 = 100 × ?

For Free Study Material & Quizzes visit : www.mathsbyarunsir.com


For any queries or suggestions email us @ mathsbyarunsir003@gmail.com or whatsapp @ 8881331466
Sol. 38 85
60 152 × 250 – × 560 + 13 × ? = 61
× ×? = 33 × 42 100 100
100 19
27 × 16 × 100 × 19 ⇒ 95 – 476 + 13 × ? = 61
⇒? = = 90
60 × 152 ⇒13 ×? = 61 + 381
340. Ans.(c) is correct
⇒ ? = 34
Sol.
3.5 × 18 – (?)² = 36 + 2
63 – 38 = (?)²
347. Ans.(e) is correct
25 = (?)²
19 21 3 1 3
?=5 × × – =? –
9 19 7 2 2
341. Ans.(b) is correct 1 3
Sol. 1– =? –
2 2
2975 ?=2
?=
1190 348. Ans.(b) is correct
? = 2.5 Sol.
342. Ans.(b) is correct 26
Sol. 12√? – × 1650 + 19 = 13 × 34
100
25 ÷ 4 × 6 12√? – 429 + 19 = 442
= ?
3
12√? = 871 – 19
? = 12.5
852
343. Ans.(c) is correct √? = = 71
12
Sol. ? = 5041
(390 + 310 – 225) 4 = ? 349. Ans.(d) is correct
(700 – 225) 4 = ? Sol.
475 × 4 = ? 535 26 63 5
× 720 × [ × × ] =?
? = 1900 1000 28 39 9
344. Ans.(e) is correct ? = 321
Sol. 350. Ans.(a) is correct
9 × 25 + 1225 + 150 = (?)² Sol.
225 + 1225 + 150 = (?)² 215 142 74
+ − ?=
9 9 9
? = √1600 283 4
?= = 31
? = 40 9 9
351. Ans.(c) is correct
345. Ans.(c) is correct
Sol.
Sol.
1 1
4900 × × 444 × – 6450 = (? )2 165 1
28 12 × 18 × × 11 + 48 = (? )2
10 27
⇒ 6475 – 6450 = (?)² (?)² = 11 × 11 + 48
⇒√25 = ? (?)² = 121 + 48
⇒?=5 (?)² = 169
346. Ans.(a) is correct ?= 13

For Free Study Material & Quizzes visit : www.mathsbyarunsir.com


For any queries or suggestions email us @ mathsbyarunsir003@gmail.com or whatsapp @ 8881331466
? = 24 × 24
352. Ans.(b) is correct ? = 576
Sol. 360. Ans.(b) is correct
101 + 1001 = ? + 841 Sol.
? = 261 59.98 = ? – 15.81
353. Ans.(a) is correct ? = 75.79
Sol. 361. Ans.(d) is correct
1 Sol.
9240 × = 440
?
1863 - ? = 202.64
9240
?= = 21 ? = 1660.36
440
354. Ans.(e) is correct 362. Ans.(c) is correct
Sol. Sol.
? × 1211 × 5 = 24220 44.5 − 5.5 = √?
24220
?= √? = 39
1211 × 5
?= 4 ? = (39)² = 1521
355. Ans.(b) is correct 363. Ans.(a) is correct
1 1 Sol.
? = (30 × 8) × (12 × 4) × × × 289
(17 × 10) 96 22 + 93 + 361 = 441 + 33 + ?
? = 204 476 – 474 = ?
356. Ans.(e) is correct ?=2
Sol. 364. Ans.(e) is correct
1 Sol.
√31 + 29 − 25 × × √35 = ?
5
1 24 × √? = 864
? = √35 × × √35 1
5
√? = 864 ×
?=7 24

357. Ans.(d) is correct √? = 36


? = 1296
Sol.
365. Ans.(a) is correct
1 40 5
(? )2 × = × × 400 Sol.
352 100 11
40 × 5 × 4 × 352 6526 – 596 – ? = 4683
(? )2 = = 25600
11
(? )2 = (160)2
? = 1247
? = 160 366. Ans.(b) is correct
358. Ans.(c) is correct Sol
3 14 34
60 × × = (? )2
× 32 ×? = 27 × 16 25 17 21
100 4
27 × 16 × 10 (? )2 =
25
?= = 22.5
6 × 32 2
? = = 0.4
359. Ans.(b) is correct 5
Sol. 367. Ans.(b) is correct
1 1 Sol.
× ? = 33 × × 24
8 11

For Free Study Material & Quizzes visit : www.mathsbyarunsir.com


For any queries or suggestions email us @ mathsbyarunsir003@gmail.com or whatsapp @ 8881331466
34 45 624
× 250 − × 340 + 150 = ? ?= =3
100 100 208
85 – 153 + 150 = ? 374. Ans.(a) is correct
? = 82 Sol.
368. Ans.(a) is correct ? 1 1
× 625 + × 750 + × 5500 – 25 = 1000
100 3 11
Sol.
? × 6.25 + 250 +500 – 25 = 1000
17 53 91 252
× × = ? × ? × 6.25 = 275
19 221 106 4
7 252 ? = 44
= ? ×
19 × 2 4 375. Ans.(d) is correct
252 19 × 2
?= × Sol.
4 7
? = 18 × 19 ? = 342 1645 + 1500 + ? = 3600
369. Ans.(c) is correct ? = 455
Sol. 376. Ans.(e) is correct
4 2 2 1 1 1 Sol.
(1 + 3 + 2 + 1 − 1 − 1) + ( + + + − − ) =?
5 5 3 6 9 6 ?
72 + 36 + 60 + 15 − 10 − 15 2450 – 1540 + 1700 – 710 = 100 × 1900
?= 5 +
90 ? × 19 = 1900
34
?= 5 + 1 ? = 100
45
34 377. Ans.(c) is correct
?= 6 45
Sol.
370. Ans.(a) is correct
2 1
Sol. × 1325 + 299 + 271 = 3 × 3300 + ?
5
3 530 + 299 + 271 = 1100 + ?
× 4200 + 625 + 1000 – 700 = (? )2
8 ?=0
(?)² = 1575 + 625 + 1000 – 700 = 2500
378. Ans.(a) is correct
? = 50
Sol.
371. Ans.(c) is correct 4750
250 + 283 – 157 + 24 + 100 =
? +62.5 ?
+ 360 – 144 – 20 = 169 + 87 4750
3 ?= = 9.5
500
? + 62.5 = (169 + 87 + 144 + 20 – 360) × 3
379. Ans.(d) is correct
? = 180 – 62.5 = 117.5
2250 + 270 – 20 + 125 + ? = 3600
372. Ans.(d) is correct
? = 975
Sol.
380. Ans.(c) is correct
270 + 121 – √? = 361 Sol.
√? = 30 (2/5){(3/5) × 750 + 40% of 375} = ? × 16
? = 900 (2/5){3 × 150 + 0.40 × 375} ÷ 16 = ?
373. Ans.(b) is correct (450 + 150) ÷ 40 = ?
Sol. ? = 600 ÷ 40
624 ? = 15
+ 21 + 27 = 36 + 220
?
624 381. Ans.(a) is correct
= 208
? Sol.

For Free Study Material & Quizzes visit : www.mathsbyarunsir.com


For any queries or suggestions email us @ mathsbyarunsir003@gmail.com or whatsapp @ 8881331466
(42% of 1500 – 230) ÷ 25 = ? ÷ 4 (5/6){(122 + 221) – (212 + 112)} = ? ÷ 24
(0.42 × 1500 – 230) ÷ 25 = ? ÷ 4 (5/6) × 19 × 24 = ?
(630 – 230) ÷ 25 = ? ÷ 4 ? = 5 × 19 × 4
(400 ÷ 25) × 4 = ? ? = 380
16 × 4 = ? 388. Ans.(a) is correct
? = 64 Sol.
382. Ans.(d) is correct (1600 – 160 – 16) ÷ 16 = ?
Sol. 1424 ÷ 16 = ?
{(420 ÷ 28)% of 1400} ÷ 7 = ? ? = 89
(15% of 1400) ÷ 7 = ? 389. Ans.(e) is correct
(0.15 × 1400) ÷ 7 = ? Sol.
210 ÷ 7 = ? ? = 30 (16 × 15) + (45 × 12) + (30 × 24) = ?
383. Ans.(c) is correct 15 × (16 + 36 + 48) = ?
(15 × 16) + 242= ? × 16 15 × 100 = ?
(240 + 576) ÷ 16 = ? ? = 1500
816 ÷ 16 = ? 390. Ans.(a) is correct
? = 51 Sol.
384. Ans.(a) is correct (22% of 1500 + 15% of 2200) = ? × 11
Sol. (0.22 × 1500 + 0.15 × 2200) ÷ 11 = ?
(2/5)(32% of 4500 – 440) = ? × 8 (330 + 330) ÷ 11 = ?
(0.32 × 4500 – 440) ÷ 20 = ? ? = 660 ÷ 11
? = (1440 – 440) ÷ 20 ? = 60
? = 1000 ÷ 20 391. Ans.(c) is correct
? = 50 Sol.
385. Ans.(b) is correct (342 – 20% of 5280) = ? ÷ 3
Sol. (1156 – 0.20 × 5280) × 3 = ?
(6/7) × 1050 – (3/8) × 1200 = ?% of 750 (1156 – 1056) × 3 = ?
900 – 450 = ?% of 750 ? = 100 × 3
?% = 450/750 ? = 300
?% = 0.6 392. Ans.(d) is correct
? = 60 Sol.
386. Ans.(d) is correct 321 + 52 × 5 – 125 = ? × 12
Sol. 321 + 260 – 125 = ?× 12
35% of 1600 – 142 = ? × 14 ? × 12 = 456
(0.35 × 1600 – 196) ÷ 14 = ? ? = 38
(560 – 196) ÷ 14 = ? 393. Ans.(e) is correct
364 ÷ 14 = ? Sol.
? = 26 (78 × 15) ÷ 52 × 8 = ? + 55
387. Ans.(c) is correct 22.5 × 8 – 55 = ?
Sol. ? = 125

For Free Study Material & Quizzes visit : www.mathsbyarunsir.com


For any queries or suggestions email us @ mathsbyarunsir003@gmail.com or whatsapp @ 8881331466
394. Ans.(e) is correct ? = 0.24 × 0.35 × 1.5 × 2500
Sol. ? = 315
(48 × 12) ÷ 18 × 8 = 4 ? 401. Ans.(c) is correct
4? = 8 × 32 Sol.
4? = 256 32% of 3500 + 28% of 2500 = ?% of 7280
? 4
4 =4 32 × 35 + 28 × 25 = ?% of 7280
?=4 1120 + 700 = ?% of 7280
395. Ans.(d) is correct 1820 = ?% of 7280
Sol. ? = 25
645 + 255 + 365 – 575 = ? 402. Ans.(e) is correct
? = 1265 – 575 Sol.
? = 690 37.5% of 2016 + 25% of 3648 = ? × 12
396. Ans.(c) is correct 0.375 × 2016 + 0.25 × 3648 = ? × 12
Sol. ? × 12 = 756 + 912
2
62.5% of 640 + 45% of ? = 25 ? = 1668/12 = 139
400 + 0.45 × ? = 625 403. Ans.(d) is correct
0.45 × ? = 225 Sol.
? = 500 According to the question,
397. Ans.(b) is correct x × 20% × 5/3 × 35% + 46 = 74
Sol. x × 1/5 × 5/3 × 7/20 = 28
2
(7/8 + 7/16) × 256 – 12 = ? x = 240
21/16 × 256 – 12 = ?2 404. Ans.(c) is correct
?2 = 324 Sol.
? = ± 18 22.5% of 400 + 32% of 625 = ? + 100
398. Ans.(a) is correct 9/40 × 400 + 8/25 × 625 = ? + 100
Sol. 90 + 200 = ? + 100
16/15 of 21/28 of 321.5 = ? ? = 190
16/15 × 3/4 × 321.5 = ? 405. Ans.(d) is correct
4/5 × 321.5 = ? Sol.
? = 257.2 (315 × 81) ÷ 45 – 147 = ? × 3
399. Ans.(e) is correct 7 × 81 – 147 = ? × 3
Sol. 567 – 147 = ? × 3
√4096 + √3249= (?)2 420/3 = ?
64 + 57 = ?2 ? = 140
? = √121 406. Ans.(a) is correct
? = 11 Sol.
400. Ans.(c) is correct √1521 + √1681= ?2 - 41
Sol. 39 + 41 = ?2 – 41
24% of 35% of 150% of 2500 = ? 80 + 41 = ?2
?2 = 121
For Free Study Material & Quizzes visit : www.mathsbyarunsir.com
For any queries or suggestions email us @ mathsbyarunsir003@gmail.com or whatsapp @ 8881331466
? = ± 11 Sol.
407. Ans.(e) is correct (7800 ÷ 312) + 65% of 1120 = 9 × ? + 24
Sol. 25 + 0.65 × 1120 = 9 × ? + 24
964 + 256 - ? = 465 + ? + 121 25 + 728 – 24 = 9 × ?
964 + 256 – 465 – 121 = 2 × ? ? = 729 ÷ 9
? = 634/2 ? = 81
? = 317 414. Ans.(c) is correct
408. Ans.(b) is correct Sol.
Sol. ? × 15 + (3960 ÷ 12) = 262 + (350 ÷ 25)
(1/2 × 4/5 + 3/5 – 1/5) × 245 = ? ? × 15 + 330 = 676 + 14
(2/5 + 3/5 – 1/5) × 245 = ? ? = (690 – 330) ÷ 15
? = 4/5 × 245 ? = 196 ? = 24
409. Ans.(a) is correct Hence, option c.
Sol. 415. Ans.(e) is correct
? = 45% of 2400 – 152 – 282 Sol.
? = 1080 – 225 – 784 45% of 1200 – 32% of 1500 = 15% of ?
? = 1080 – 1009 (0.45 × 1200 – 0.32 × 1500) ÷ 0.15 = ?
? = 71 ? = (540 – 480) ÷ 0.15
410. Ans.(c) is correct ? = 400
Sol. Hence, option e.
23% of ? = 25% of 448 + 12 ×√(841) – 20% of 460 416. Ans.(b) is correct
23% of ? = 112 + 12 × 29 – 92 Sol.
23% of ? = 112 + 348 – 92 (322 – 222) ÷ 32= ?
23% of ? = 368 (1024 – 484) ÷ 9 = ?
? = 1600 ? = 540 ÷ 9
411. Ans.(a) is correct ? = 60
Sol. Hence, option b.
417. Ans.(a) is correct
√3481 + √4356= ?% of 500
Sol.
59 + 66 = ?% of 500
{(45 × 220) ÷ 33} + 325 = ?
?% of 500 = 125
{9900 ÷ 33} + 325 = ?2
? = 125 × 100/500 = 25
412. Ans.(d) is correct ?2 = 300 + 325

Sol. ?2 = 625
? = 25
(4/9) × 486 + 35% of 120 +(3/8) × 208 = ?
Hence, option a.
4 × 54 + 0.35 × 120 + 3 × 26= ?
418. Ans.(b) is correct
? = 216 + 42 + 78
Sol.
? = 336
(111 + 121 + 131 + 141) ÷ 21 = ?
Hence, option d.
413. Ans.(a) is correct 504 ÷ 21 = ?
? = 24

For Free Study Material & Quizzes visit : www.mathsbyarunsir.com


For any queries or suggestions email us @ mathsbyarunsir003@gmail.com or whatsapp @ 8881331466
Hence, option b. 93 × (8910 ÷ 110) = (14580 ÷ 20) × ?
419. Ans.(e) is correct 729 × 81 = 729 × ?
Sol. ? = 81
2
22% of 3500 – 15 = 20% of 725 + ? Hence, option c.
0.22 × 3500 – 225 – 0.20 × 725 = ? 425. Ans.(b) is correct
? = 770 – 225 – 145 Sol.
? = 400 262 + 113 - √49 = 2 × ?3
Hence, option e. 676 + 1331 – 7 = 2 × ?3
420. Ans.(d) is correct 2000 = 2 × ?3
Sol. ? = 10
?% of 4200 + 1380 × 5/46 = 25% of 3120 Hence, option b.
?% of 4200 + 150 = 0.25 × 3120 426. Ans.(d) is correct
? = {(780 – 150) × 100}/4200 Sol.
? = 15 65% of 1200 – 15% of 400 = ?3 × 45% of 200
Hence, option d. 0.65 × 1200 – 0.15 × 400 = ?3 × 0.45 × 200
421. Ans.(e) is correct (780 – 60)/90 = ?3
Sol. ?3 = 8
13 × 8 + 13 × 12 + 13 × 16 + 13 × 20 = ?× 8 ?=2
13 × (8 + 12 + 16 + 20) = ? × 8 Hence, option d.
? = (13 × 56) ÷ 8 427. Ans.(c) is correct
? = 91 Sol.
Hence, option e. 35% of 5000 – 322 + 3 = ?
422. Ans.(b) is correct 0.35 × 5000 – 1024 + 3 = ?
Sol. 1750 – 1024 + 3 = ?
737 × 3 + 363 × 2 + 11 × 33 = ? × 11 ? = 729
Dividing by 11 Hence, option c.
67 × 3 + 33 × 2 + 33 = ? 428. Ans.(a) is correct
3 × (67 + 22 + 11) = ? Sol.
? = 3 × 100 20% of 5000 – 25% of 3000 = ? × 52
? = 300 0.20 × 5000 – 0.25 × 3000 = ? × 25
Hence, option b. 1000 – 750 = ? × 25
423. Ans.(d) is correct ? = 250/25 = 10
Sol. Hence, option a.
? × 32 – 25% of (480 × 128) = 0 429. Ans.(a) is correct
? × 32 – 0.25 × 480 × 128 = 0 Sol.
? × 32 – 120 × 128 = 0 16 × 15 – 240 – 32 × 10 + 420 = ?2
? = 480 240 – 240 – 320 + 420 = ?2
Hence, option d. -320 + 420 = ?2
424. Ans.(c) is correct ? = 10
Sol.

For Free Study Material & Quizzes visit : www.mathsbyarunsir.com


For any queries or suggestions email us @ mathsbyarunsir003@gmail.com or whatsapp @ 8881331466
Hence, option a. ? = (2000 – 1000) ÷ 25
430. Ans.(e) is correct ? = 40
Sol. Hence, option d.
65% of 200 – 26% of 500 = ? 436. Ans.(a) is correct
0.65 × 200 – 0.26 × 500 = ? Sol.
? = 130 – 130 ? = (28 x 16 - 28)/30
?=0 = (448 - 28)/30
Hence, option e. = 420/30 = 14
431. Ans.(a) is correct 437. Ans.(a) is correct
Sol. Sol.
45% of 1500 – 2 × 52= ?2 ? = (15 x 680/100) + (45 x 340/100)
2
0.45 × 1500 – 2 × 25 = ? = (10200/100) + (15300/100)
675 – 50= ?2 = 102 + 153 = 255
? = 25 438. Ans.(c) is correct
Hence, option a. Sol.
432. Ans.(c) is correct (?)2 = 382 - 39 - 93
Sol. = 1444 - 39 - 729 = 676
2
(32 × 25) – 20 = 20% of ? So, ? = 26
(800 – 400) × 5 = ? 439. Ans.(a) is correct
? = 2000 Sol.
Hence, option c. Equating the powers of 2, 11 we have:
433. Ans.(e) is correct 2(2/3) = 22 x 4/5 x 2a/6
Sol. 2(2/3) = 28/5 x 2a/6
22% of 5000 + 14% of 400 = ?2 So, 2/3 = 8/5 + a/6
0.22 × 5000 + 0.14 × 400 = ?2 a = 6 x (2/3 - 8/5)
2
1100 + 56 = ? a = -28/5
? = 34 11(3/4) = 114+b
Hence, option e. b = 3/4 - 4
434. Ans.(b) is correct b = -13/4
440. Ans.(b) is correct
Sol.73 – 132– ? = 25% of 120 Sol.
343 – 169 – 0.25 × 120= ? ?2 = (3 x 60 x √144)/(∛3375)
174 – 30 = ? = 3 x 60 x 12/15
? = 144 = 144 = 122
Hence, option b. So, ? = 12
441. Ans.(c) is correct
435. Ans.(d) is correct Sol.
Sol.
? = 132 x (21/6) - √625
80% of 2500 - ?% of 2500 = 103
= 22 x 21 - 25
(0.80 × 2500 – 1000) ÷ 25 = ?

For Free Study Material & Quizzes visit : www.mathsbyarunsir.com


For any queries or suggestions email us @ mathsbyarunsir003@gmail.com or whatsapp @ 8881331466
= 462 - 25 = 437 5(3/4) = 56+b
442. Ans.(c) is correct b = 3/4 - 6
Sol. b = -21/4
2
? = 47 + 60 - 57 2 2 448. Ans.(c) is correct
= 2209 + 3600 - 3249 = 2560 Sol.
443. Ans.(d) is correct ? = 150 x 11 + 230 x 9
Sol. = 1650 + 2070 = 3720
(?)2 = 192 - 11 - 53 449. Ans.(b) is correct
= 361 - 11 - 125 = 225 Sol.
So, ? = 15 ? = (900/6) - 14 x 11 + √225
444. Ans.(b) is correct = 150 - 154 + 15
Sol. =11
We note that 12.5% = 1/8, 20% = 1/5, 55.55% = 5/9 450. Ans.(e) is correct
So, ? = 200 x (1/8) + 100 x (1/5) + 45 x (5/9) Sol.
= (8 x 25) x (1/8) + (5 x 20) x (1/5) + (9 x 5) x (5/9) We note that 66.66% = 2/3, 77.77% = 7/9
= (1 x 25) + (1 x 20) + (5 x 5) = 70 So, ? = 33 x (2/3) + 261 x (7/9)
445. Ans.(a) is correct = (3 x 11) x (2/3) + (9 x 29) x (7/9)
Sol. = (2 x 11) + (7 x 29) = 225
Equating the powers of 12, 13 we have: 451. Ans.(a) is correct
12(3/4) = 122 x 2/4 x 12a/4 Sol.
12(3/4) = 124/4 x 12a/4 (?)2 = 162 + 322 - 55
So, 3/4 = 4/4 + a/4 = 256 + 1024 - 55
a = 4 x (3/4 - 4/4) = 1225
a = -1 ? = 35
(3/4) 4+b
13 = 13 452. Ans.(b) is correct
b = 3/4 - 4 Sol.
b = -13/4 (0.33)107 ÷ (0.32)59 = (0.31)?
446. Ans.(d) is correct (0.3)(3x107 - 2x59) = (0.31)?
Sol. So, ? = (3x107 - 2x59 )/1 = 203
(0.43)113 ÷ (0.42)54 = (0.43)? 453. Ans.(c) is correct
(0.4)(3x113 - 2x54) = (0.43)? Sol.
So, ? = (3x113 - 2x54 )/3 = 77 3
?2 = (78 x 120 x √2197)/(√400)
447. Ans.(c) is correct
= 78 x 120 x 13/20
Sol.
= 6084 = 782
Equating the powers of 10, 5 we have:
So, ? = 78
10(3/4) = 102 x 2/6 x 10a/5
454. Ans.(b) is correct
10(3/4) = 104/6 x 10a/5
Sol.
So, 3/4 = 4/6 + a/5
? = 195 x 16 + 280 x 4
a = 5 x (3/4 - 4/6)
= 3120 + 1120 = 4240
a = 5/12

For Free Study Material & Quizzes visit : www.mathsbyarunsir.com


For any queries or suggestions email us @ mathsbyarunsir003@gmail.com or whatsapp @ 8881331466
455. Ans.(c) is correct 463. Ans.(c) is correct
Sol. Sol.
We note that 71.42% = 5/7, 77.77% = 7/9 ? = 36 x (28/6) - √961
So, ? = 189 x (5/7) + 306 x (7/9) = 6 x 28 - 31
= (7 x 27) x (5/7) + (9 x 34) x (7/9) = 168 - 31 = 137
= (5 x 27) + (7 x 34) = 373 464. Ans.(b) is correct
456. Ans.(e) is correct Sol.
Sol. ? = (35 x 34 - 30)/40
? = (660/6) - 15 x 12 + √256 = (1190 - 30)/40
= 110 - 180 + 16 = 1160/40 = 29
=-54 465. Ans.(d) is correct
457. Ans.(b) is correct Sol.
Sol. ? = 36 x 16 - 48 x 50
? = 115 x 15 + 150 x 11 = 576 - 2400
= 1725 + 1650 = 3375 = -1824
458. Ans.(d) is correct 466. Ans.(e) is correct
Sol. Sol.
3 3 3
7 = 343, 13 = 2197, 14 = 2744 ? = (45 x 580/100) - (30 x 530/100)
3 3
? = √343 − √2197 + √2744
3 = (26100/100) - (15900/100)
= 7 - 13 + 14 = 8 = 261 - 159 = 102
459. Ans.(d) is correct 467. Ans.(e) is correct
Sol. Sol.

? = 362 - 452 - 242 422 = 1764, 952 = 9025, 622 = 3844


= 1296 - 2025 - 576 = -1305 ? = √1764 + √9025 + √38444
460. Ans.(e) is correct = 42 + 95 + 62 = 199
Sol. 468. Ans.(d) is correct
(?)2 = 202 + 162 - 31 ? = 302 - 332 - 142
= 400 + 256 - 31 = 900 - 1089 - 196
= 625 = -385
? = 25 469. Ans.(a) is correct
461. Ans.(d) is correct Sol.
Sol. 193 = 6859, 123 = 1728, 133 = 2197
? = 1024(0.18 + 0.22) = 10240.40 = 10242/5 3 3
? = √6859 + √1728 + √2197
3

Since, 10241/5 = 4, ? = 42 = 16 = 19 + 12 + 13 = 44
462. Ans.(e) is correct 470. Ans.(a) is correct
Sol. Sol.
2 2 2
? = 19 + 36 + 37
? = 168 x (27/7) - √256
= 361 + 1296 + 1369
= 24 x 27 - 16
= 3026
= 648 - 16 = 632

For Free Study Material & Quizzes visit : www.mathsbyarunsir.com


For any queries or suggestions email us @ mathsbyarunsir003@gmail.com or whatsapp @ 8881331466
471. Ans.(c) is correct ? = 32(0.03 + 0.57) = 320.60 = 323/5
Sol. Since, 321/5 = 2, ? = 23 = 8
? = 142 + 152 - 342 480. Ans.(d) is correct
= 196 + 225 - 1156 = -735 Sol.
472. Ans.(a) is correct ? = 72 x (9/6) - √1681
(?)2 = 322 - 6 - 93 = 12 x 9 - 41
= 1024 - 6 - 729 = 289 = 108 - 41 = 67
So, ? = 17 481. Ans.(b) is correct
473. Ans.(e) is correct Sol.
Sol. ? = (33 x 20 - 20)/40
2 2 2
(?) = 14 + 34 - 56 = (660 - 20)/40
= 196 + 1156 - 56 = 640/40 = 16
= 1296 482. Ans.(d) is correct
? = 36 Sol.
474. Ans.(a) is correct ? = 142 + 152 - 592
Sol. = 196 + 225 - 3481 = -3060
3
?2 = (10 x 16 x √100)/(√64) 483. Ans.(a) is correct
= 10 x 16 x 10/4 Sol.
= 400 = 202 ? = 19 x 44 - 14 x 40
So, ? = 20 = 836 - 560
475. Ans.(e) is correct = 276
SOL. 484. Ans.(e) is correct
? = 135 x 15 + 190 x 3 Sol.
= 2025 + 570 = 2595 ? = (35 x 460/100) + (80 x 685/100)
476. Ans.(e) is correct = (16100/100) + (54800/100)
Sol. = 161 + 548 = 709
2 3 4
25 = 625, 4 = 64, 9 = 6561 485. Ans.(c) is correct
? = 625 - 4 + 9 Sol.
= 630 (?)2 = 132 + 262 - 4
477. Ans.(c) is correct = 169 + 676 - 4
Sol. = 841
? = (15 x 95 x 39)/(19 x 13) ? = 29
= 15 x (95/19) x (39/13) = 15 x 5 x 3 = 225 486. Ans.(e) is correct
478. Ans.(d) is correct Sol.
862 = 7396, 202 = 400, 802 = 6400 ?2 = (2 x 24 x √4)/(√216)
3

? = √7396 + √400 + √6400 = 2 x 24 x 2/6


= 86 + 20 + 80 = 186 = 16 = 42
479. Ans.(e) is correct So, ? = 4
Sol. 487. Ans.(d) is correct

For Free Study Material & Quizzes visit : www.mathsbyarunsir.com


For any queries or suggestions email us @ mathsbyarunsir003@gmail.com or whatsapp @ 8881331466
Sol. ? = 513
? = 175 x 8 + 310 x 5 Hence, option a.
= 1400 + 1550 = 2950 494. Ans.(e) is correct
488. Ans.(b) is correct
Sol.
252 = 625, 103 = 1000, 64 = 1296
? = 625 - 10 + 6
495. Ans.(b) is correct
= 621
489. Ans.(a) is correct
Sol.
(80% of 1400 + 70% of 2800) = ?
1120 + 1960 = ?
? = 3080
496. Ans.(a) is correct
Hence, option a.
490. Ans.(b) is correct
Sol.
(122 × 2 – 162) = ?
(144 × 2 – 256) = ?
288 – 256 = ?
? = 32
497. Ans.(d) is correct
Hence, option b.
491. Ans.(b) is correct
Sol.
232 + (242 ÷ 16) + 40% of 600= ?
529 + (576 ÷ 16) + 240= ?
529 + 36 + 240 = ?
? = 805 498. Ans.(e) is correct
Hence, option b.
492. Ans.(b) is correct
Sol.
(20% of 352) + 186 ÷ 6 – 27 × 6 = ?
(20% of 1225) + 31 – 162 = ?
499. Ans.(b) is correct
245 + 31 – 162 = ?
? = 114
Hence, option b.
493. Ans.(a) is correct
Sol.
500. Ans.(a) is correct
(16 × 9 of 2 + 152) = ?
(16 × 18 + 225) = ?
288 + 225 = ?

For Free Study Material & Quizzes visit : www.mathsbyarunsir.com


For any queries or suggestions email us @ mathsbyarunsir003@gmail.com or whatsapp @ 8881331466
501. Ans.(d) is correct
511. Ans.(b) is correct

502. Ans.(d) is correct


512. Ans.(a) is correct
Sol.9801+1137-1898+2346 = ? + 2446
⇒ 11386-2446= ?
⇒ ?=8940
503. Ans.(c) is correct 513. Ans.(e) is correct

514. Ans.(e) is correct


504. Ans.(b) is correct Sol.
? = 540 – 392
= 148
515. Ans.(d) is correct
505. Ans.(c) is correct Sol.
Sol.340+1428-1312+4 =? ? = 81.701
? = 460 516. Ans.(d) is correct
506. Ans.(a) is correct Sol.
Sol.158×11-1750+1131+221= ? ? = 12.52
? =1340
517. Ans.(a) is correct
507. Ans.(b) is correct
Sol.
Sol.490- 2160+3160+121 = ?
? = 1611 ? = 475 + 608 – 900 = 183
508. Ans.(c) is correct 518. Ans.(b) is correct
Sol.
64 47 4 ? (256)2
× 7 =( ) ×
1000 10 10 108
4 ? 47+3–8
509. Ans.(a) is correct ⇒( ) =
10 102
Sol. 4 2
=( )
29 + 17 – 18 = ? – 124 + 12 10
28 + 124 – 12 = ? ⇒ ?=2
? = 140
519. Ans.(d) is correct
510. Ans.(d) is correct
Sol.
⇒ 249 + 1149 + 2 =? +1335

For Free Study Material & Quizzes visit : www.mathsbyarunsir.com


For any queries or suggestions email us @ mathsbyarunsir003@gmail.com or whatsapp @ 8881331466
⇒ ? = 1400 − 1335 18? = 187.9 × 180.1 ÷ 184
= 65 Or, ? = 7.9 + 0.1 - 4 = 4
520. Ans.(a) is correct 527. Ans.(b) is correct
Sol. Sol.
33 × 27 3 35 22 13 36 38 18 245
⇒ × 11.25 + 45 × = + + − − =
3 7 4 ? 7 5 5 7 35 35
3 35×35
3 Or, ? = =5
⇒ 4 × 11.25 + 33.75 245
3 528. Ans.(e) is correct
⇒ 37.5 Sol.
88.2 − 4 = 10−?
521. Ans.(c) is correct
Or, ? = -74.2
Sol.
529. Ans.(b) is correct
140 × 36.5 100
⇒ × Sol. ? = 31.3
100 80 × 12.5
530. Ans.(c) is correct
∴ ? = 5.11
522. Ans.(d) is correct Sol. ? = 12.4 – 6.5

Sol. = 5.9
531. Ans.(d) is correct
385 × 12 4
⇒ +? = 344 + ×? 3545.64
8 3 Sol. ? = 12.25×21.6
4
⇒ 577.5 − 344 = ( − 1) ×? = 13.4
3
532. Ans.(b) is correct
⇒ ? = 3 × 233.5
15.5 24.5
? = 700.5 Sol. ? = 100 × 646 + 100 × 298
523. Ans.(e) is correct = 173.14
Sol. 533. Ans.(c) is correct
40 × 3.5 80 × 3.5 (? ) 1 1 1
⇒ + = Sol. ? = (2 + 1 + 2) + (3 + 5 + 4)
100 100 10
20 + 12 + 15
140 + 280 ? =5+
⇒ = 60
100 10
47
⇒ ? = 42 =5
60
524. Ans.(b) is correct 534. Ans.(b) is correct
Sol. Sol. ? = 72.089
35 ÷ 7 × 342 =? 535. Ans.(c) is correct
Or, ? = 1710 Sol. ? = 192 – 32 + 211
525. Ans.(a) is correct
= 371
Sol. 536. Ans.(e) is correct
805 + 187 + 2? = 1000 Sol. ? = 18.42 – 15
?
Or, 2 = 8 = 3.42
Or, ? = 3 537. Ans.(a) is correct
526. Ans.(d) is correct 56 67
Sol. × 958 + 100 × 1008 =? × 20
100
Sol.

For Free Study Material & Quizzes visit : www.mathsbyarunsir.com


For any queries or suggestions email us @ mathsbyarunsir003@gmail.com or whatsapp @ 8881331466
2 1 1
Sol. ? = (1+2–2) +   – 
? = 60.592 5 7 2
538. Ans.(a) is correct (28  10 – 35) 3
=1+ =1
Sol. ? = 491.91 70 70
539. Ans.(d) is correct 550. Ans.(c) is correct
Sol. Sol. ?2 = 56 + 25
 ?   21
1
3
 ?2 = 81
⇒ ? = 9,261  ?= 9
540. Ans.(c) is correct 551. Ans.(b) is correct
Sol.?=12,303-3,219 Ans. (b)
= 9,084 Sol. ? = 23, 350
541. Ans.(a) is correct 552. Ans.(d) is correct
Sol. Ans. (d)
? = 880
Sol. ? = 66.52 – 50.79
542. Ans.(d) is correct
=15.73
Sol.
553. Ans.(a) is correct
? = 110.8 – 37.9
? = 72.9  11 
5

543. Ans.(b) is correct Sol. (1.1) =    ? = 5


?

 10 
Sol. 554. Ans.(b) is correct
? = 581 + 207 = 788 Sol. ? =100 + 150
544. Ans.(a) is correct = 250
2 555. Ans.(c) is correct
Sol.(40% = 5) of 265 + (35% of 180) = 50% of ?
⇒ 106 + 63 = 50% of ? 13?
Sol. =26
4
⇒ ? = 169 × 2 = 338
545. Ans.(e) is correct 676 4
 ?=
13
Sol.
 ? = 208
460× 15 − 5 × 20 =?
556. Ans.(a) is correct
? = 6900 – 100 = 6800
Sol. ? = 0.333 + 2.02
546. Ans.(d) is correct
= 2.353
Sol. 5163 + 3209 = 8372 – 4018 = 4354
557. Ans.(b) is correct
547. Ans.(b) is correct
Sol. ? = 4.6 × 2.5 – 3.2
21 10
Sol. 5 × + ? = 24 ? = 8.3
3
? = 24 – 14 = 10 558. Ans.(e) is correct
548. Ans.(c) is correct 7 7

Sol. 4848 ÷ 24 = 202, Sol. ? 2 = (11)2


202 × 11 = 2222, 2222 – 222 = 2000  ? = 11
549. Ans.(e) is correct 559. Ans.(e) is correct
Sol. ? = 69.9 + 107.1 – 174
?=3
For Free Study Material & Quizzes visit : www.mathsbyarunsir.com
For any queries or suggestions email us @ mathsbyarunsir003@gmail.com or whatsapp @ 8881331466
560. Ans.(d) is correct 1
Sol. Hint: 460 × 15 = 460 × 10 + 2× 4600 – 100 = 4600 + 2300 – 100
Sol. ? = 21.35 – 3.75 = 17.6
= 6900 – 100 = 6800
561. Ans.(a) is correct
573. Ans.(d) is correct
Sol. ? = 5612 – 1394 = 4218
Sol. 5163 + 3209 = 8372 – 4018 = 4354
562. Ans.(e) is correct
574. Ans.(b) is correct
Sol. ? = 4207 – 3007 = 1200 21 10
Sol. × + ? = 24
563. Ans.(d) is correct 5 3

Sol. ? = 21 × 84 – 89 = 1675 ? = 24 – 14 = 10
575. Ans.(c) is correct
564. Ans.(c) is correct
Sol. ? = 55.8 + 7.2 – 38.2 = 24.8 Sol. 4848 ÷ 24 = 202,

565. Ans.(b) is correct 202 × 11 = 2222, 2222 – 222 = 2000


576. Ans.(b) is correct
Sol. ? = 589.57
566. Ans.(a) is correct Sol.
700 200 50
Sol. √360 − 450 + 379 = √289 = 17 ? = 900 × 3456 + 300 × 1881 – 300 × 12354
567. Ans.(c) is correct = 7 × 384 + 2 × 627 – 2059

Sol. 3? = (32 )3 × (34 )3 = 36 × 38 ÷ 39 = 35 = 2688 + 1254 – 2059


= 1883
Or, ? = 5
577. Ans.(c) is correct
568. Ans.(b) is correct
Sol.
Sol. 2? = 572 ÷ 26 = 22 × 12 = 264 – 200 = 64 = 26
5 3 45
Or, ? = 6 ? = √ 4 × 108 + 160 + 2 + 100 × 460
569. Ans.(a) is correct 5 45
= √243 + 100 × 460
1 7 5
Sol. ? = (4 + 1 – 2) + (2 + 12 − 6)
= 3 + 207
6 + 7 − 10 1 = 210
= 3+( )=3
12 4 578. Ans.(d) is correct
570. Ans.(e) is correct
Sol.
Sol. 36% of 245 = (40 – 4)% of 245
23 20 32
2 4 × 245 ?= × × 360 – 100 × 950
5 3
= × 245 − = 98 − 9.8 = 88.2
5 100 = 11040 – 304
2
40% of 210 = 5× 210 = 84 = 10736
Difference = 88.2 – 84 = 4.2 579. Ans.(a) is correct
? = 10 – 4.2 = 5.8 Sol.
54
571. Ans.(a) is correct ? = 327 × 5 + 105 – 100 × 1645
2
Sol. (40% = 5) of 265 + (35% of 180 = 851.7
580. Ans.(c) is correct
= 180% of 35) = 50% of ?
9 Sol.
⇒ 106 + (180% = 5) of 35 = 50% of ?
3 2 48 36
( + ) 𝑜𝑓 ? = × 550 + × 750
⇒ 106 + 63 = 50% of ? 5 5 100 100
⇒ ? = 534
⇒ ? = 169 × 2 = 338
581. Ans.(a) is correct
572. Ans.(e) is correct

For Free Study Material & Quizzes visit : www.mathsbyarunsir.com


For any queries or suggestions email us @ mathsbyarunsir003@gmail.com or whatsapp @ 8881331466
Sol. (6)? = (6)3 ÷ 64 × 66 590. Ans.(c) is correct
Sol.
⇒ (6)? = 63−4+6
? = 755 + 8291 + 1377
⇒?=5
582. Ans.(b) is correct = 10,423
591. Ans.(b) is correct
Ans.(b)
Sol.
Sol. ? = 576.03 – 472.31
? = 132.3 + 145.7 – 52.5
= 103.72
583. Ans.(e) is correct = 225.5
592. Ans.(c) is correct
Sol. ? = 24 ÷ 16 × 7.4 + 343 − 231
Sol.
= 11.1 + 112 (315+105)
= 123.1 ?= ×7
3
584. Ans.(c) is correct ? = 980
Sol. 7 ×? = 84×84 × 12 ×
1 593. Ans.(d) is correct
28 24
Sol.
? = 18
7 102
585. Ans.(b) is correct ? = 17 × 133 × 304 = 96
7.9 3.4 594. Ans.(b) is correct
Sol. ? = 100 × 134 − 100 × 79
Sol.
= 7.9
586. Ans.(b) is correct ? = 1198 – 881
Sol. = 317
17 90 69 52 595. Ans.(c) is correct
?= × + ×
5 17 13 23 Sol.
= 18 + 12
√?− 611 + 550 = 19
= 30
⇒√? = 80
587. Ans.(c) is correct
Sol. ⇒ ? = 6400
596. Ans.(a) is correct
? = √108 + 117
Sol.
= √225
7 49 343 1
? = 15 (0.7)? = × × ×
10 10 1000 10
588. Ans.(a) is correct
7 6
Sol. = ( ) ⇒ ?= 6
10
? × 3 + 198 = ? × 9 – 102 597. Ans.(e) is correct
300
⇒? = Sol.
6

⇒ ? = 50 79 87 101
?= + −
589. Ans.(b) is correct 8 16 24
474 + 261 − 202
Sol. =
48
?² = 49 + 32 533
=
⇒ ?² = 81 48
5
⇒ ?= ± 9 = 11
48

For Free Study Material & Quizzes visit : www.mathsbyarunsir.com


For any queries or suggestions email us @ mathsbyarunsir003@gmail.com or whatsapp @ 8881331466
598. Ans.(b) is correct ? = 4200 + 180
Sol. ? = 4380
1 2
?= × 243 + × 345
3 3 Ans 607. (c)
= 81 + 230 ? = 580 + 6 – 3
= 311 ?= 583
599. Ans.(c) is correct

Ans608. (b)
Sol. 75 45
? 1228   400
0.1 + 0.011 + 0.00011 100 100
?= 3
 1228  45  4
0.1 + 0.0111 + 1 4
0.11111 1  3  307  180
= =
1.1111 10  921  180
= 0.1  1101
Ans609. (c)
600. Ans.(e) is correct
? = 1520 + 18420 + 21
= 19961

601. Ans.(a) is correct Ans610. (c)


Sol. 44
63?  225   225
?² = 395 + 791 – 990 100
56
?² = 196 63?  225 
100
⇒ ? =± 14 ?2
602. Ans. (d)
65 150 Ans611. (b)
Sol. 100 × 240 + 100 ? = 210
18 190
1.5? = 210 – 156 = 54 ?  18 19
100
? = 36 1800 19
?
603. Ans. (a) 18 190
 ?  10
2 7 75
? = 3 × 5 × 100 × 540
? = 378 Ans612. (a)
604. Ans. (a) 1 1
?  444  202
1
? = (4 – 2 + 1) + ( − +
5 7
) 2 4
2 6 12 ?
3 1  202  111
? = 3 + 12 = 3 + 4 2
1  91  182
?=34
605. Ans. (b) Ans613. (a)
? = 1218.06

606. Ans. (c)


? = 4200 + 38 × 480

For Free Study Material & Quizzes visit : www.mathsbyarunsir.com


For any queries or suggestions email us @ mathsbyarunsir003@gmail.com or whatsapp @ 8881331466
64 16 3 256 4
( )( ) ( )  ( )? 1
1000 100 10000 10 Ans622. (a)
64 1000 162 4
   ( )? 1
16 16 16 104 10 ?  55  2695
4 4
 ( )  ( )? 1 ?  49
10 10
 ? 1  1 ?  2401
?0

Ans623. (b)
Ans 614. (e)
?=22500-6400-737-9604
?= 5759
?  1425  8560  8.4
?  9993.4 Ans624. (c)
48 + 6 -5 =?
Ans615. (a)
? = 49
75 35 Ans625. (d)
? 1240  1560
100 100
 930  546 ? = 18.657 – 13.290
 384 ? = 5.367
Ans616. (a)
? = 621.65 Ans626. (c)
? = 236 + 160
Ans617. (b) = 396
? = 794
Ans627. (a)
Ans618. (c) ? = 0.333 + 2.02
2 3 4 = 2.353
?   215  128  147
5 4 7
 86  96  84
Ans628. (b)
 98
? = 4.6 x 2.5 – 3. 2
Ans619. (b) = 8.3

?  56  7  64  9  4  29
Ans629. (c)
 968  116
1 1
 852 3
? 2  11
3
2

 ?  11
Ans620. (a)
?  707  148 Ans630. (e)
 855 ?  2.33  30  1.53  70  2  87
 69.9  107.1  174
Ans621. (d)  177  174  3

3
?  11 14300 Ans631. (e)
1100
 429
For Free Study Material & Quizzes visit : www.mathsbyarunsir.com
For any queries or suggestions email us @ mathsbyarunsir003@gmail.com or whatsapp @ 8881331466
?  15.68  5.67  3.75 = 5+
107
70
 17.6 37
=6
70
Ans632. (e)
639. Ans.(d) is correct
56 20
?  225  150  109 Sol.
100 100
 126  130  109  265 ?
= 67 × 7 + 73 × 5.1 – 52 × 2.2
10
Ans633. (b) ? = 7269
1.85 ? = 499 – 425
640. Ans.(a) is correct
= ? = 40
Sol.
? = 3564 + 348.6 + 574.34 – 673.85
Ans634. (d)
= 3813.09
18?  187.9 0.1 4
18?  18 641. Ans.(c) is correct
?4 Sol.
? 3 15 13 52
Ans635. (c) × × × × 544 =
5 4 17 6 7
35 1 3 1 3 18 52
 (3  1  7  5)  (     ) ⇒ ?× 52 × 3 =
7
? 7 5 5 7 35
1
⇒?=
4 2 18 21
7(   )
5 7 35
28  10  18 642. Ans.(b) is correct
7( )
35 Sol.
7
? = 86.66 + 200 + 253
= 539.66
Ans636. (e)
643. Ans.(d) is correct
36 40
?  10   245   210 Sol.
100 100
4
 10  88.20  84 3
× ? = √(256 + 473) + 504
5
 5.8 2565
?= = 641.25
4
637. Ans.(b) is correct
Sol. 644. Ans.(a) is correct
39 51 40 Sol.
?= × 450 + × 340 – × 255
100 100 100 25 28 12+? 500
= 246.9 × × =
7 5 6 9
50
638. Ans.(c) is correct ⇒ ?= – 12
3
14 2
Sol. = =4
3 3
2 5 7 2
? = (3 + 4 + 5 – 7) + ( + + – )
5 7 10 7

For Free Study Material & Quizzes visit : www.mathsbyarunsir.com


For any queries or suggestions email us @ mathsbyarunsir003@gmail.com or whatsapp @ 8881331466
645. Ans.(e) is correct 652. Ans.(d) is correct
Sol. Sol.
? = 4168 + 279 = 4447 2.4
16 × ? = 288
100
646. Ans.(c) is correct ? = 750
Sol. 653. Ans.(b) is correct
Sol.
? = 505 + 202 + 4040.4 + 4.484
= 4751.884 (−251 × 21 × (−12)) 15813
=
? 100
? = 400
647. Ans.(d) is correct
654. Ans.(d) is correct
Sol.
Sol.
49 + 81 – 64 =? – 121
130 × 130 1
⇒ 66 = ? – 121 ?= [ × 15] = 338
25 30
⇒ ? = 121 + 66 = 187 655. Ans.(e) is correct
648. Ans.(b) is correct Sol.
Sol.
? = √√4900 + √5476 = √70 + 74 = √144 = 12
3 2 1
?= 1+ + 2+ + 2+ 656. Ans.(e) is correct
4 3 2
3 2 1 Sol.
= 5+( + + )
4 3 2 ? = 24.375 − 1.955 = 22.420
23
= 5+ 657. Ans.(d) is correct
12
11 11 Sol.
= 5+1 =6
12 12 ? = 13.141 + 31.417 – 27 .118
= 17.44
649. Ans.(e) is correct 658. Ans.(b) is correct
Sol. Sol.
Expression
? = √8.0656
6 42 35
= 500 × × ×
7 100 100 80656
= 63 =√
10000
284
650. Ans.(a) is correct =
100
Sol.
= 2.84
8888 + 848 + 88 – ? = 7337 + 737 659. Ans.(b) is correct
⇒ 9824 – ? = 8074 Sol.
⇒ ? = 9824 – 8074 3.2
? = 5 × 3.2 + 4 × 4 + 3 ( )
= 1750 4
651. Ans.(c) is correct = 16 + 16 + 2.4
Sol. = 34.4
127512
660. Ans.(e) is correct
?= = 44
414×7 Sol.

For Free Study Material & Quizzes visit : www.mathsbyarunsir.com


For any queries or suggestions email us @ mathsbyarunsir003@gmail.com or whatsapp @ 8881331466
?2 − 441 = 80 × 5 ⇒?=5
668. Ans.(b) is correct
⇒ ? = √841
Sol. ? = 576.03 – 472.31
⇒ ? = 29
661. Ans.(a) is correct = 103.72
Sol.
669. Ans.(e) is correct
?2 = √162 × 128
Sol. ? = 24 ÷ 16 × 7.4 + 343 − 231
= 144
= 11.1 + 112
⇒ ? = 12
= 123.1
662. Ans.(a) is correct
Sol.
670. Ans.(c) is correct
?=6√5 × 12√5 − 361 84×84 1
Sol.7 ×? = × 12 × 24
28
= 360 − 361
? = 18
= −1
671. Ans.(b) is correct
7.9 3.4
663. Ans.(b) is correct Sol. ? = × 134 − × 79
100 100
Sol. = 7.9
?=111.1 + 25.8 + 153.5
= 290.4 672. Ans.(b) is correct
664. Ans.(b) is correct Sol.
144
Sol. ?= =9
16
?3 = 54 × 32
3
? = √1728 673. Ans.(c) is correct
? = 12 Sol.
? = 26,281
665. Ans.(e) is correct
674. Ans.(a) is correct
Sol. Sol.
5 14 13 2 9 × ? = 207.27
?= + × ×
4 9 8 13 ⇒ ? = 23.03
5 14 45+14
? = 4 + 36 = 675. Ans.(b) is correct
36
59 23 Sol.
? = 36 = 1 36
(?)³ = 180 + 156 – 120 = 216
666. Ans.(e) is correct ⇒?=6
Sol.
676. Ans.(a) is correct
? = 25 × 14 − 42 + 4? = 182
Sol.
⇒350 − 42 + 4? = 324
(?)² = 2 × 7√8 – 21 + 8 + 49 – 14√8
⇒4? = 324 − 308 = 16
= 57 – 21
?=2 = 36
667. Ans.(a) is correct ∴ ? = ±6
Sol.(6)? = (6)3 ÷ 64 × 66 677. Ans.(d) is correct
⇒ (6 )? =6 3−4+6

For Free Study Material & Quizzes visit : www.mathsbyarunsir.com


For any queries or suggestions email us @ mathsbyarunsir003@gmail.com or whatsapp @ 8881331466
Sol.
? 684. Ans.(c) is correct
× 19.07 = 359.56 – 207
100
Sol.
152.56 × 100
?= −2 −2
19.07
?× 5 = (4)−5× 5 + (7)−3× 3
? = 800 16+49
or, ? = = 13
5

678. Ans.(c) is correct


Sol. 685. Ans.(e) is correct
Sol.
(?)⁴ = 243 × 27
1.4 + 2.8
(?)⁴ = (9) ⁴ ?= × 100 = 42
10
⇒?=9
686. Ans.(c) is correct
679. Ans.(b) is correct Sol.
Sol. 7 5 1 1
? = (1 + 2 + 3 − 4) + ( + + − )
9 3 9 5
8 24 25 7
?= +
3 7
× ×
6 50
106 16
8 =2+ =4
= +2 45 45
3
2 687. Ans.(c) is correct
=4
3
Sol. ? = 6894
688. Ans.(b) is correct
680. Ans.(a) is correct
Sol. Sol. ?= 81.25 + 2.1

? = 0.0003 – 0.00003 + 0.03 = 83.35


= 0.03027
689. Ans.(a) is correct
? 1
681. Ans.(b) is correct Sol.350 × 100 × 50 = 343
Sol. ⇒ ? = 4900
?
= 354 – 425 + 240
41 690. Ans.(a) is correct
⇒ ? = 41 × 169
Sol.12 × 3842 + 100
15
× ? = 2449
⇒ ? = 6929 528 × 100
⇒?=
15
⇒ ? = 3520
682. Ans.(b) is correct
Sol.
691. Ans.(d) is correct
36864 1
√? = × 24 × = 288 448.8
64 48 Sol. ? = 24
∴ ? = 82944 ? = 18.7

683. Ans.(d) is correct 692. Ans.(b) is correct


Sol. Sol.
144
30 2 2 2 2 ?= =9
?= × × × × × 9450 = 48 16
100 7 9 5 3

For Free Study Material & Quizzes visit : www.mathsbyarunsir.com


For any queries or suggestions email us @ mathsbyarunsir003@gmail.com or whatsapp @ 8881331466
693. Ans.(c) is correct Sol.
Sol. 400 × 185 240 × 35 1648 ×?
? = 26,281 + =
100 100 100

694. Ans.(a) is correct ⇒ 74000 + 8400 = 1648 × ?


Sol. 82400
∴ ?= = 50
9 × ? = 207.27 1648
703. Ans.(c) is correct
⇒ ? = 23.03
695. Ans.(b) is correct Sol.
Sol.
(?)³ = 180 + 156 – 120 = 216 √244 + 224 = ? × 202
⇒?=6 ⇒ 24 × 24 + 224 = ? × 202
696. Ans.(a) is correct ⇒ 800 = ?× 400
Sol.
800
(?)² = 2 × 7√8 – 21 + 8 + 49 – 14√8 ∴ ?= =2
400
= 57 – 21 = 36 704. Ans.(a) is correct
∴ ? = ±6
Sol.
697. Ans.(a) is correct
Sol. 8 × 250
? = 2.8 × 1.5 +
100
19 51 11
?= × × × 490 = 4.2 + 20 = 24.2
5 7 3
705. Ans.(c) is correct
= 49,742
Sol.
698. Ans.(c) is correct
Sol. 160 × 250 120 × 400
+?
100 100
?
+ 14094 + 91 = 14,285 ⇒ 400+ ? = 480
13
? ∴ ? = 480 − 400 = 80
⇒ = 100 706. Ans.(b) is correct
13
⇒ ? = 1300 Sol.
699. Ans.(b) is correct 65 × 780 78 × 650
?= +
100 100
Sol.
= 507 + 507 = 1014
? 707. Ans.(b) is correct
= 156
0.8
Sol.
⇒ ? = 124.8
? = 1656
700. Ans.(c) is correct
708. Ans.(c) is correct
Sol.
Sol.
? = √784 + 2352 ? = 31 × 5 + 41 × 11 – 5 × 79
= √3136
= 606 – 395
? = 56
= 211
701. Ans.(e) is correct
709. Ans.(c) is correct
Sol.
Sol.
? = 0.81 + 0.49 + 0.16
? = 738 + 236 – 874
= 1.46
= 100
702. Ans.(d) is correct
710. Ans.(d) is correct

For Free Study Material & Quizzes visit : www.mathsbyarunsir.com


For any queries or suggestions email us @ mathsbyarunsir003@gmail.com or whatsapp @ 8881331466
Sol. 720. Ans.(a) is correct
3 4 18 Sol. 4.5 + 23.50 + 14.58 – 17.68 ×0.5
?= × × × 2673
4 27 11 = 42.58 – 8.84 = 33.74
= 2 × 243 721. Ans.(c) is correct
= 486 3675 7480
Sol. × = 49 × 93.5 = 4581.5
711. Ans.(b) is correct 75 80
722. Ans.(b) is correct
Ans.(b)
Sol.
Sol.
?= 1 + 11.1+ 11.11
4 7
?= × 17 + × 15 ? = 23.21
17 15
= 11 723. Ans.(c) is correct
712. Ans.(c) is correct Sol.

Sol.
63×15
+? = 61 + 235 1
5 ? = 15 −
2
? = 296 − 189 = 107 1
713. Ans.(a) is correct = 14
2
Sol.
63×15
+? = 61 + 235 724. Ans.(a) is correct
5
Sol.
? = 296 − 189 = 107
36 44
714. Ans.(d) is correct ?= × 125 + × 225
100 100
Sol.(? )2 − 312 + 289 = 5018 14,400
=
(? )2 = 5041, ? = 71 100
715. Ans.(e) is correct = 144
1411 725. Ans.(c) is correct
Sol. + 583 ×? = 2415
17
Sol.
2332
?= =4 ? = 861
583
716. Ans.(b) is correct 726. Ans.(b) is correct
3864 1 Sol.
Sol. = (? )2 1
24
1
? × 4 + 31 × 6 = 2 × 820
161 = (? )2 , ? = (161)2 = 25921 224
⇒?=
717. Ans.(b) is correct 4

Sol.668 ÷ 167 × 284 = 4 × 284 = 1136 ⇒ ? = 56


727. Ans.(b) is correct
718. Ans.(e) is correct
5
Sol.
Sol. 60% of 25% of 6 th of x = 630
1562 + 15.6% of ? = 3,590
60 25 5
Or, 100 × 100 × 6 𝑥 = 630 202,800
⇒?= = 13,000
𝑥
Or, 8 = 630 15.6
728. Ans.(b) is correct
∴ x = 5040 35 263 18
Sol. ? = − 35
719. Ans.(d) is correct 35
35×35
Sol. (85410+ 36885 + 24705) ÷ 1600 Or, ?= =5
245
= 147000 ÷ 1600 = 91.875 729. Ans.(b) is correct

For Free Study Material & Quizzes visit : www.mathsbyarunsir.com


For any queries or suggestions email us @ mathsbyarunsir003@gmail.com or whatsapp @ 8881331466
Sol. 533
=
17 90 69 52 48
?= × + ×
5 17 13 23 5
= 11
= 18 + 12 48
737. Ans.(a) is correct
= 30
730. Ans.(a) is correct Sol.
Sol. ?² = 395 + 791 – 990
? × 3 + 198 = ? × 9 – 102 ?² = 196
300 ⇒ ? =± 14
⇒? =
6
738. Ans.(c) is correct
⇒ ? = 50 150 120
731. Ans.(b) is correct Sol. × 220 + 100 × 140+? = 850
100

Sol. ? = 850 – 15 × 22 – 12 × 14
?² = 49 + 32 = 352
739. Ans.(d) is correct
⇒ ?² = 81
30 70
Sol. 100 × 340 − 100 × 110 = 65−?
⇒ ?= ± 9
732. Ans.(b) is correct ? = 65+77-102
Sol. ? = 65 – 25
? = 132.3 + 145.7 – 52.5 = 40
= 225.5 740. Ans.(a) is correct
Sol.
733. Ans.(d) is correct ? = 91 – 64
Sol. = 27
7 102 741. Ans.(e) is correct
?= × × 304
17 133 2 1 1
Sol. ? = (1+2–2) +   – 
= 96 5 7 2
734. Ans.(b) is correct (28  10 – 35) 3
Sol. =1+ =1
70 70
? = 1198 – 881 742. Ans.(b) is correct
= 317 64 + 60 ÷ (30 ÷ 3) = (?) ÷ 20
735. Ans.(c) is correct ⇒ 64 + 60 ÷ 10 = x/ 20
Sol.
⇒ 64 + 6 = x/ 20
√?− 611 + 550 = 19 ⇒ x = 1400
⇒√? = 80 743. Ans.(e774) is correct
⇒ ? = 6400 1251/3 × 271/3 × 1254/3 = 15 × 5?
736. Ans.(e) is correct
⇒ 5 × 3 × 54 = 15× 5x
Sol.
⇒ 15 × 54 = 15×5x
79 87 101
?= + − ⇒x=4
8 16 24
474 + 261 − 202 744. Ans.(a) is correct
=
48

For Free Study Material & Quizzes visit : www.mathsbyarunsir.com


For any queries or suggestions email us @ mathsbyarunsir003@gmail.com or whatsapp @ 8881331466
⇒ 125% of 3060 = 408 + 85% of (?) Sol.

⇒ 3825 – 408 = [85 × (?)]/100 33 × 272 3


⇒ × 11.25 + 45 ×
37 4
⇒ 3417 = [85 × (?)]/100
36
(?) = 3417 × 100/85 ⇒ 4 × 11.25 + 33.75
3
(?) = 4020 ⇒ 135
745. Ans.(e) is correct 751. Ans.(d) is correct
[(37.5 × 19.2) ÷ 18 + 21.50 ÷ 4.3] = ? × 15 Sol.
Applying BODMAS Rule; 539 × 12 4
⇒ +? = 404 + ×?
⇒ [(300/8 × 19.2) ÷ 18 + 21.50 ÷ 4.3] = ? × 15 16 3
4
⇒ [720 ÷ 18 + 5] = ? × 15 ⇒ 404.25 − 404 = ( − 1) ×?
3
⇒ 40 + 5 = 15 × ? ⇒ ? = 3 × 0.25
⇒ 45 = 15 × ? 3
?=
∴?=3 4
752. Ans.(b) is correct
746. Ans.(c) is correct
Sol.
338.40 ÷ 42.3 + 25% of 4200 ÷ 100 = ? + 246.5 ÷ 29
36864 1
Applying BODMAS Rule; √? = × 24 × = 288
64 48
⇒ 8 + 1050 ÷ 100 = ? + 8.5
∴ ? = 82944
⇒ ? = 8 + 10.5 – 8.5 753. Ans.(d) is correct
∴ ? = 10 Sol.
747. Ans.(e) is correct 30 2 2 2 2
?= × × × × × 9450 = 48
Sol. 100 7 9 5 3
3 754. Ans.(c) is correct
√3969 − √2209 + 9 = √68921−?
3
Sol.
⇒ 63 − 47 + 9 = √68921−?
−2 −2
⇒ ? = 41 + 47 − 72 ?× 5 = (4)−5× 5 + (7)−3× 3
16+49
= 16 or, ? = = 13
5
748. Ans.(d) is correct 755. Ans.(e) is correct
Sol. Sol.
7.9 0.1 4 ?
⇒ 18 × 18 ÷ 18 = 18 1.4 + 2.8
?= × 100 = 42
18(7.9+0.1) 10
⇒ = 18? 756. Ans.(c) is correct
184
188 Sol.
⇒ 4 = 184 ⇒? = 4
18 7 5 1 1
749. Ans.(d) is correct ? = (1 + 2 + 3 − 4) + ( + + − )
9 3 9 5
Sol. 106 16
=2+ =4
45 45
⇒ 249 + 1149 + 2 =? +1335
757. Ans.(b) is correct
⇒ ? = 1400 − 1335
Sol.
= 65
750. Ans.(a) is correct 35 ÷ 7 × 342 =?

For Free Study Material & Quizzes visit : www.mathsbyarunsir.com


For any queries or suggestions email us @ mathsbyarunsir003@gmail.com or whatsapp @ 8881331466
Or, ? = 1710 4
758. Ans.(a) is correct ⇒ 577.5 − 344 = ( − 1) ×?
3
Sol. ⇒ ? = 3 × 233.5
805 + 187 + 2? = 1000 ? = 700.5
Or, 2? = 8 766. Ans.(e) is correct
Or, ? = 3 Sol.
759. Ans.(d) is correct 40 × 3.5 80 × 3.5 (? )
⇒ + =
Sol. 100 100 10
140 + 280 ?
18? = 187.9 × 180.1 ÷ 184 ⇒ =
100 10
Or, ? = 7.9 + 0.1 - 4 = 4 ⇒ ? = 42
760. Ans.(b) is correct 767. Ans.(e) is correct
Sol. Sol.
35 22 13 36 38 18 245 ? = 540 – 392
= + + − − =
? 7 5 5 7 35 35 = 148
35×35
Or, ? = =5 768. Ans.(d) is correct
245
761. Ans.(e) is correct Sol.
Sol. ? = 81.701
88.2 − 4 = 10−? 769. Ans.(d) is correct
Or, ? = -74.2 Sol.
762. Ans.(d) is correct ? = 12.52
Sol. 770. Ans.(a) is correct
⇒ 249 + 1149 + 2 =? +1335 Sol.
⇒ ? = 1400 − 1335 ? = 475 + 608 – 900 = 183
= 65 771. Ans.(b) is correct
763. Ans.(a) is correct Sol.
Sol. 64 47 4 ? (256)2
× =( ) ×
33 × 27 3 1000 107 10 108
⇒ × 11.25 + 45 ×
3 7 4 4 ? 47+3–8
3 3 ⇒( ) =
10 102
⇒ 4 × 11.25 + 33.75 4 2
3
=( )
⇒ 37.5 10
764. Ans.(c) is correct ⇒ ?=2
Sol. 772. Ans.(d) is correct
140 × 36.5 100 Sol.
⇒ ×
100 80 × 12.5 ? = 47.22
∴ ? = 5.11 773. Ans.(a) is correct
765. Ans.(d) is correct Sol.
Sol. 45
385 × 12 4 ? = 23 ×
15
⇒ +? = 344 + ×?
8 3 ? = 69

For Free Study Material & Quizzes visit : www.mathsbyarunsir.com


For any queries or suggestions email us @ mathsbyarunsir003@gmail.com or whatsapp @ 8881331466
774. Ans.(b) is correct ⇒ ? = 57 – 34
Sol.
∴ ? = 23
4046 × 14 × 15 780. Ans.(e) is correct
?=
210 × 17
Solution
? = 238
1920/?2 + 96/6 – 10 = 1/30 + 42/7
775. Ans.(e) is correct ⇒ 1920/?2 + 16 – 10 = 1/30 + 6
Sol. ⇒ 1920/?2 + 6 = 1/30 + 6
5 1 8 ⇒ 1920/?2 = 1/30 + 6 - 6
? = (4 + 7 − 5) + ( + − )
6 2 11
⇒ 1920/?2 = 1/30
(55 + 33 − 48)
=6+ ⇒ 1920 × 30 = ?2
66
40 ⇒ 240 × 8 × 30 = ?2
=6+
66
⇒ 240 × 240 = ?2
20
=6 ⇒ 2402 = ?2
33
776. Ans.(b) is correct ⇒ 240 = ?
Sol.
∴ ? = 240
83 781. Ans.(b) is correct
?= × 2350
100 Sol.
? = 1950.5
? = 41 × 4 + 51 × 7 – 4 × 89
777. Ans.(d) is correct
= 521 – 356
Sol.
= 165
⇒ 5 × 8 + 60 ÷ 4 – 45 + 2
782. Ans.(b) is correct
⇒ 5 × 8 + 15 – 45 + 2 Sol.
⇒ 40 + 15 – 45 + 2 ? = 209 × 5 + 128 – 768
⇒ 57 – 45 = 405
⇒ 12 783. Ans.(e) is correct
778. Ans.(c) is correct Sol.
Solution 3 4
?= × × 2100
(9/2 + 4) × 8 = ? × 10 5 7
= 12 × 60
⇒ (9 + 8)/2 × 8 = x × 10
= 720
⇒ 17 × 4 = x × 10 784. Ans.(d) is correct
⇒ x = 68/ 10 Sol.
⇒ x = 6.8 2 3 5
?= × × × 1920
779. Ans.(a) is correct 5 4 8
Solution = 360
785. Ans.(e) is correct
83.33% of 90 + 78 – 168 × 8 ÷ 14 = ? + 34
2 1 1
⇒ 5/6 × 90 + 78 – 168 × 8 ÷ 14 = ? + 34 Sol. ? = (1+2–2) +   – 
5 7 2
⇒ 75 + 78 – 96 = ? + 34

For Free Study Material & Quizzes visit : www.mathsbyarunsir.com


For any queries or suggestions email us @ mathsbyarunsir003@gmail.com or whatsapp @ 8881331466
(28  10 – 35) 3 ?=
23
×
20 32
× 360 – 100 × 950
=1+ =1 5 3
70 70
= 11040 – 304
786. Ans.(c) is correct
= 10736
Sol. ?2 = 56 + 25
794. Ans.(a) is correct
 ?2 = 81
Sol.
 ?= 9 54
? = 327 × 5 + 105 – 100 × 1645
787. Ans.(a) is correct
Sol. = 851.7
795. Ans.(c) is correct
1
1+ 1 + (−2)4 Sol.
(64)2
3 2 48 36
1 ( + ) 𝑜𝑓 ? = × 550 + × 750
= 17 8 5 5 100 100
788. Ans.(a) is correct ⇒ ? = 534
Sol. 796. Ans.(c) is correct
34 48 Explanation :334 + 614 – 1354 = – 406
3 × 10 =? , ? = 2.4 797. Ans.(a) is correct
68
3 Explanation :21*58 = 1218
789. Ans.(e) is correct 798. Ans.(c) is correct
Sol. Explanation :0.12 + 6.25 = 6.37
(1.1 )2 − (4.24 × 0.04) =? 799. Ans.(d) is correct
⇒ ?=1.0404 Explanation :24+76 = 100
790. Ans.(d) is correct 800. Ans.(b) is correct
Sol. Explanation :
(280 × 16) ÷ 8 120 + (730x/100) = 354
?= = 40 (730x/100) = 234
14
X = 32%
791. Ans.(b) is correct
801. Ans.(d) is correct
Sol.
Explanation :256 + 729 – 24 = 961 = 312
700 200 50
? = 900 × 3456 + 300 × 1881 – 300 × 12354 802. Ans.(b) is correct
= 7 × 384 + 2 × 627 – 2059 Explanation :
= 2688 + 1254 – 2059 3(46-16) + (36-3)3 = 90 + 332
90+ 1089 = 1179
= 1883
803. Ans.(a) is correct
792. Ans.(c) is correct
Sol. Explanation :
22×248×122 = 665632
5 3 45 665632/22 = 30256
? = √ 4 × 108 + 160 + 2 + 100 × 460
804. Ans.(a) is correct
5 45
= √243 + 100 × 460 Explanation :
= 3 + 207 = 900 ÷ 3+300×10
= 300 + 3000 = 3300
= 210
805. Ans.(b) is correct
793. Ans.(d) is correct
Sol.

For Free Study Material & Quizzes visit : www.mathsbyarunsir.com


For any queries or suggestions email us @ mathsbyarunsir003@gmail.com or whatsapp @ 8881331466
Explanation :
645 × 16 × 85.74 =216 × 4096 = 884736
84 =4096
806. Ans.(a) is correct
Explanation :
222.5 + 441 = 663.5
663.5/6 = 110.6
807. Ans.(a) is correct
Explanation : 817. Ans.(d) is correct
230.4 + 269.01 = 100 + ?
499.41 = 100 + 399.41
808. Ans.(b) is correct
Explanation :
(14/3)+(37/5)-(38/3)+(25/7) = (490+777-1330+375)/105
= 312/105 = 2(102/105)
809. Ans.(d) is correct
Explanation :
5.39 + 4256 – 18.75
4261.39 – 18.75 = 4242.64
810. Ans.(c) is correct 818. Ans.(d) is correct
Explanation : Correct Option: D
= 2.8 ×6.9 – 96
= 19.32 – 96 73 × 8 80
⇒ 48% of 2500 – – 15% of =?
= – 76.68 √196 3
811. Ans.(b) is correct
Answer 73 × 8 80
B. 13.5 ⇒ 1200 – – 0.15 × =?
7×2 3
812. Ans.(b) is correct
Answer 12
B. 17/32 ⇒ ? = 1200 – (72 × 4) –
3
813. Ans.(a) is correct
Answer ⇒ ? = 1200 – 196 – 4
A. 0
814. Ans.(c) is correct ⇒ ? = 1000
Answer 819. Ans.(c) is correct
C. 37 Correct Option: C
815. Ans.(c) is correct ⇒ 123 + 2744 ÷ 72 – 29 + 232 + 50 = ?3
Answer
⇒ 123 + 2744 ÷ 49 – 29 + 529 + 50 = ?3
C. 208
816. Ans.(c) is correct ⇒ 123 + 56 – 29 + 529 + 50 = ?3

⇒ 758 – 29 = ?3

⇒ 729 = ?3
For Free Study Material & Quizzes visit : www.mathsbyarunsir.com
For any queries or suggestions email us @ mathsbyarunsir003@gmail.com or whatsapp @ 8881331466
Explanation :
⇒?=9 = 534.596 + 61.472 – 496.708 – 27.271
= 596.068 – 523.979 = 72.089
820. Ans.(a) is correct
⇒ 36% of 450 + √3136 – 18 × 13 = ? 830. Ans.(a) is correct
Explanation :
⇒ 162 + 56 – 234 = ? (√3 – 2)2 = ? – √12 – √36
3 + 4 – 4√3 = x – 2√3 – 6
⇒ ? = – 16 13 – 2√3 = x

Hence, option A is correct. 831. Ans.(d) is correct


821. Ans.(c) is correct Explanation :
3 4 2 (9 ÷ 2 * 27 ÷ 9)/(18 ÷ 7.5 * 5 ÷ 4) = 27/6 = 4.5
of of of 875 ÷ 5–1 = ?
5 7 3
? = 25 × 4 × 2 × 5 832. Ans.(c) is correct
? = 1000 Explanation :
822. Ans.(b) is correct (?)2 + (123)2 = (246)2 – (99)2 – 246
60516 – 27392 = 33124 = (182) 2
Sol.
833. Ans.(c) is correct
? = 1656
Explanation :
823. Ans.(c) is correct
572 ÷ 26 = 22
Sol. 22 x 12 = 264
? = 31 × 5 + 41 × 11 – 5 × 79 264 – 200 = 64 = 26
834. Ans.(b) is correct
= 606 – 395 = 211
824. Ans.(c) is correct Explanation :
[√8(3 + 1) x √8(8 + 7)] – 98
Sol.
[4√8 x 15√8] – 98
? = 738 + 236 – 874 = 100 480 – 98 = 382
825. Ans.(d) is correct 835. Ans.(b) is correct
Sol. Explanation :
3 4 18 √(2352 – 1331 + 3) = √1024 =32
?= × × × 2673
4 27 11 836. Ans.(b) is correct
= 2 × 243 = 486 Explanation :
826. Ans.(b) is correct (65/100)*56*5 = ? + 154
Explanation : 182 – 154 = 28
(0.4)? = (0.064)*(0.4)7 / (0.0256)2 = (0.4)2 837. Ans.(a) is correct
827. Ans.(b) is correct Explanation :
Explanation : 86+7 = 93
572 ÷ 26 * 12 – 200 100-93 = 7
= 22 * 12 – 200 = 264 – 200 = 64 = (2)6 838. Ans.(d) is correct
828. Ans.(b) is correct (45)2 + (21)2 = (?)2 + 257
Explanation :
350% of ? ÷ 50 + 248 = 591 or, (?)2 = (45)2 + (21)2 – 257
x = (343 * 100 * 50) / 350 = 4900
829. Ans.(c) is correct or, (?)2 = 2025 + 441 – 257 = 2209.

For Free Study Material & Quizzes visit : www.mathsbyarunsir.com


For any queries or suggestions email us @ mathsbyarunsir003@gmail.com or whatsapp @ 8881331466
86 – (86)2 + 86 × (86 + 86 ÷ 0.86) = ?
or, ? = √2209 = 47 ? = 86 – (86)2 + 86 × (86 + 100)
? = 86 – (86)2 + 86 × 186
839. Ans.(a) is correct ? = 86 (1 – 86 + 186)
? = 86 (101)
65% of 400 + √? = 44% of 800 – 12% of 400 ? = 8686
⇒ (50% of 400 + 10% of 400 + 5% of 400) + √? = (50% of 800 844. Ans.(c) is correct
– 6% of 800) – 12% of 400
57% of 350 + ?% of 170 = – 89.5 – 512 – 9
⇒ (200 + 40 + 20) + √? = (400 – 48) – 48

or, √? = 352 – 48 – 260 = 44 57 170


350 × +?× = – 89.5 – 8 – 9
100 100
∴ ? = (44)2 = 1936. 199.5 + 1.7 × ? = – 106.5
1.7 × ? = – 106.5 – 199.5
840. Ans.(a) is correct 1.7 × ? = – 306
40% of 375 ÷ 512 × 30 = 4500 ÷ ? ? = – 180
150 ÷ 8 × 30 = 4500 ÷ ?
? = 4500 × 8 ÷ 150 ÷ 30 845. Ans.(b) is correct
?=8 529 ÷ ?% of 270 = 115 × 81 ÷ ?2
41. Ans.(d) is correct 23 ÷ (2.7 × ?) = 115 × 9 ÷ ?2
Correct Option: D 23 ÷ 2.7 = 115 × 9 ÷ ?
299 ? = 115 × 9 ÷ 23 × 2.7
× 11 – 361 = ?2 + 38
169 ? = 121.5

299 846. Ans.(d) is correct


× 11 – 19 = ?2 + 38
13 Sol.
2
23 × 11 – 19 – 38 = ? ? = (6² - 3²)× (8² - 4²) ÷ (10.5² - 1.5²)
253 – 57 = ?2 (6 + 3) × (6 − 3) × (8 + 4) × (8 − 4)
?=
(10.5 + 1.5) × (10.5 − 1.5)
?2 = 196 9 × 3 × 12 × 4
? = 14 ?= = 12
12 × 9

842. Ans.(d) is correct 847. Ans.(b) is correct


2
94.5 × 512 ÷ 343 ÷ 6.4 = ? + 56 Sol.
35% 𝑜𝑓 540 + 410% 𝑜𝑓 30 = 39% 𝑜𝑓 ?
512 39
945 × = ?2 + 56 189 + 123 = 100 × ?
7 × 64
312 × 100
135 × 8 – 56 = ?2 ?= = 800
39
1080 – 56 = ?2 848. Ans.(e) is correct
?2 = 1024 Sol.
? = ± 32 5
?2 = 40% of × 352
11
2 5
843. Ans.(c) is correct ?2 = × × 352 = 64 ⇒ ? = 8
5 11
For Free Study Material & Quizzes visit : www.mathsbyarunsir.com
For any queries or suggestions email us @ mathsbyarunsir003@gmail.com or whatsapp @ 8881331466
849. Ans.(a) is correct 857. Ans.(d) is correct
Sol. Sol.
11 195 132 64 × 288 × 16
?× = × ?2 = = 32 = 25
15 108 65 256 × 36
?=5 ?=5
850. Ans.(e) is correct 858. Ans.(b) is correct
Sol. Sol.
(√1444 + √676) 38 + 26 64 90% 𝑜𝑓 ? = √576 ÷ √64 × √729
3
?2 = = = = 16
4 4 4 24 27
?=4 ?= × × 10
4 9
851. Ans.(d) is correct ? = 180
Sol. 859. Ans.(a) is correct
64 × 288 × 16 Sol.
?2 = = 32 = 25
256 × 36
43 108
?=5 ? = 40% of 320 + ×
18 8
852. Ans.(b) is correct
= 128 + 48 = 176
Sol. 860. Ans.(d) is correct
3
90% 𝑜𝑓 ? = √576 ÷ √64 × √729 Sol.
24 27 60 152
?= × × 10 × ×? = 33 × 42
4 9 100 19
? = 180 27 × 16 × 100 × 19
853. Ans.(a) is correct ⇒? = = 90
60 × 152
Sol. 861. Ans.(d) is correct
43 108 Sol.
? = 40% of 320 + × (15)3.7×3 ÷ (15)2×4.3 × (15)2.5 ÷ (3)5 = (5)?
18 8
(15)11.1−8.6+2.5 ÷ (3)5 = (5)?
= 128 + 48 = 176
(15)5 ÷ (3)5 = (5)?
854. Ans.(d) is correct
3 5 × 55 ÷ (3) 5 = (5)?
Sol. 5 5 = (5)?
60 152 ?=5
× ×? = 33 × 42 862. Ans.(c) is correct
100 19
27 × 16 × 100 × 19 Sol.
⇒? = = 90 ? = 1396 + 412 – 2704 + 961
60 × 152
855. Ans.(a) is correct ? = 65
Sol. 863. Ans.(c) is correct

11 195 132 Ans.(c)


?× = × Sol.
15 108 65
?=5 90
⇒ ? × 100 = 126+396
856. Ans.(e) is correct 522
Sol. ⇒?= × 10 = 580
9
864. Ans.(e) is correct
2
(√1444 + √676) 38 + 26 64
? = = = = 16 (142 −82 )
4 4 4 Sol. × (72–42) = ? × 400
?
?=4

For Free Study Material & Quizzes visit : www.mathsbyarunsir.com


For any queries or suggestions email us @ mathsbyarunsir003@gmail.com or whatsapp @ 8881331466
(14−8)(14+8)×3×(7+4) 871. Ans.(c) is correct
⇒ ?2 = 400
Sol.
2 6×22×11×3 33 2
⇒? = = (10)
400 5 3 45
? = √ 4 × 108 + 160 + 2 + 100 × 460
? = 3.3
5 45
= √243 + 100 × 460
865. Ans.(d) is correct = 3 + 207 = 210
Sol. 872. Ans.(d) is correct
92 121 8 Sol.
× × = 22 + 66
11 23 ? 23 20 32
352 ?= × × 360 – 100 × 950
5 3
= = 88
? = 11040 – 304
⇒?=4 = 10736
866. Ans.(b) is correct 873. Ans.(b) is correct
Sol. 874. Ans.(c) is correct

4 1 2 3 1 2 ?2 % of 11.11% of 256 × 169 ÷ 2704 = 81


7 + 3 + 5 =? +4 − 7 + 11 ?2 × 1 ÷ 900 × 16 × 169 ÷ 52 = 81
3 2 3 5 2 5
4 1 2 3 ?2 = 81 × 900 × 52 ÷ 16 ÷ 169
? = (7 + 3 + 5) + + + − 4 + 7 − 11 −
3 2 3 5 ?2 = 2025/ 16
2 1
− + ? = 45/ 4 = 11.25
5 2
875. Ans.(d) is correct
?= 9
Sol.
4
867. Ans.(c) is correct 3
× ? = √(256 + 473) + 504
5
66 5 2565
Sol. 100 𝑜𝑓 350+ ? = 8 𝑜𝑓 1256 ?=
4
⇒ 231+ ? = 785 = 641.25
∴ ? = 785 − 231 = 554 876. Ans.(b) is correct
Sol.
868. Ans.(b) is correct 3 2 1
Sol. ?= 1 + +2+ +2+
4 3 2
80 1 2 3 2 1
⇒?× = × 360 + × 120 =5+( + + )
11 3 3 4 3 2
11 23
⇒? = × [120 + 80] =5+
80 12
= 11 × 2.5 11 11
= 27.5 =5+1 =6
12 12
869. Ans.(b) is correct 877. Ans.(e) is correct
Sol. Sol.
3570 Expression
?=
1190 6 42 35
?=3 = 500 × × ×
7 100 100
870. Ans.(e) is correct = 63
878. Ans.(a) is correct

For Free Study Material & Quizzes visit : www.mathsbyarunsir.com


For any queries or suggestions email us @ mathsbyarunsir003@gmail.com or whatsapp @ 8881331466
Sol. 885. Ans.(d) is correct
8888 + 848 + 88 – ? = 7337 + 737
⇒ 9824 – ? = 8074
⇒ ? = 9824 – 8074
886. Ans.(c) is correct
= 1750
879. Ans.(c) is correct
Sol.
127512
?= = 44
414×7
880. Ans.(d) is correct 887. Ans.(d) is correct
Sol.
25 + 210 = ? × 12
25 (1 + 32)
=?
12
⇒ ? = 88 888. Ans.(a) is correct
881. Ans.(a) is correct
Sol.
√256 + √784 =?× √121
16 + 28 = ? × 11 889. Ans.(c) is correct
44
?= =4
11
882. Ans.(b) is correct
Sol.
1 1 1 1 1
? +2 + 1 = 1 + 4 + 7
2 4 8 2 4
1 1 1 1 1 890. Ans.(a) is correct
? = (1 + 4 + 7 – 2 – 1) + [ + + – – ]
8 2 4 2 4
1 73
?= 9 =
8 8
883. Ans.(e) is correct
891. Ans.(b) is correct
Sol.
318 × 5160 ÷ [(43/100)*2400] = 46 × 990 ÷ 15 + x – 2400
115% of 360 + 180% of 270 = ? × 225
318 *(5160/1032) = 46*(990/15) + x – 2400 1590 = 3036 + x – 2400
414 + 486 = ? × 225
1590 + 2400 – 3036 = x
900
?= =4 X = 954
225
884. Ans.(b) is correct 892. Ans.(c) is correct
Sol. Sol.98 ÷ 14 × 49 – 294 = (? )2
34 × 96 ⇒ 7 × 49 – 294 = (? )2
?
9 × 729 =
81 ⇒ 343 – 294 = (? )2
2
9 ×9 6 ⇒ (? )2 = 49 ⇒ ? = 7 or – 7
9? = 2 = 93
9 × 93
893. Ans.(a) is correct
⇒?=3

For Free Study Material & Quizzes visit : www.mathsbyarunsir.com


For any queries or suggestions email us @ mathsbyarunsir003@gmail.com or whatsapp @ 8881331466
46×450+84×525−55×880
Sol.(2 × 3)3 ÷ (4 × 9)2 × (27 + 8)2 = (6)? Sol. ? = 100
⇒ 63 ÷ (36)2 × (216)2 = (6)? ?=164
⇒ 63 ÷ (62 )2 × (63 )2 = (6)?
3−4 + 6
⇒6 = (6 )? ⇒ ? = 5
894. Ans.(b) is correct 903. Ans.(b) is correct
3
Sol.103.72 Sol. √256 + 320 + 4 × ? = 3×16
? = 32
895. Ans.(e) is correct
904. Ans.(a) is correct
Sol.√576 ÷ (4 )2 )3
× 7.4 + (7 – 231 = ? Sol. (−3)4 ÷ 34 = 1
⇒ 24 ÷ 16 × 7.4 + 343 – 231 = ? 905. Ans.(e) is correct
⇒ 1.5 × 7.4 + 343 – 231 = ? Sol. ? (2.5 − 1) = 48 × 2.5
⇒ 11.1 + 343 – 231 = ? 120
⇒ ? = 123.1 or, ? = = 80
1.5
906. Ans.(a) is correct
896. Ans.(a) is correct
2
Sol. 215 ÷ 5 =? −215
Sol. (√3 − 2) = ? −√12 − √36
or ? = 43 + 215 = 258
⇒ 3 + 4 − 4√3 =? −2√3 − 6
⇒ ? = 7 – 4√3 + 2 √3 + 6 ⇒ 13 – 2 √3 907. Ans.(b) is correct
Sol.
897. Ans.(a) is correct 2 1 9 188
3 +4 − =
Sol. 7 14 14 ?
30 × 30 × 30 1 23 57 9 188
[ × 24] × = 36 ×? + − =
60 25 7 14 14 ?
432 46 + 57 − 9 188
⇒ ?= =
36 14 ?
46 + 57 − 9 188
⇒ ? = 12 =
898. Ans.(c) is correct 14 ?
94 188
Sol. =
14 ?
? =12,596 – 9690 188
?= × 14 = 28
= 2906 94
899. Ans.(b) is correct
80% of 24000 – 16% of 7200 =? – 15% of 2000 = > (80/100)*24000 – 908. Ans.(e) is correct
(16/100)*7200 =? – (15/100)*2000 = > 19200 – 1152 =? – 300 = Sol.
> 19200 – 1152 + 300 =? = > 18348 =?
√15 × 222 − 40%of602 + 19 × 39 = ?2
900. Ans.(c) is correct
√15 × 484 − 40% of 3600 + 19 × 39 = ?2
292 + 52 % of 900 – (3/7) of 1750 = x - 113 841 + (52/100)*900 – √7260 − 1440 + 741 = ?2
(3/7)*1750 + 1331 = x X = 841 + 468 – 750 + 1331 X = 1890 √6561 = ?2
901. Ans.(a) is correct 81 =?2
∛35937 + √5184 ÷ 2? – 4/5 of 65 + 52 = 42 33 + (72/2x) – (4/5)*65 + ?= 9
25 = 42 (72/2x) = 42 – 33 – 25 + 52 (72/2x) = 36 2x = 72/36 = 2 909. Ans.(c) is correct
2x = 21 X = 1 Sol.
902. Ans.(a) is correct 40% of ? + 55% of 360 = 36% of 450 + 102

For Free Study Material & Quizzes visit : www.mathsbyarunsir.com


For any queries or suggestions email us @ mathsbyarunsir003@gmail.com or whatsapp @ 8881331466
2 11 36 (?)² = 2 × 7√8 – 21 + 8 + 49 – 14√8
× ? + × 360 = × 450 + 100
5 20 100 = 57 – 21
2 = 36
× ? +198 = 162 + 100
5 ∴ ? = ±6
2 917. Ans.(a) is correct
×? = 262 − 198
5 1
2 Sol. 2652 − 441 + 928 – 15 × 3375 = ? + 961
×? = 64
5 2652 − 441 + 928 – 225 – 961 = ?
? = 160 ? = 1953
910. Ans.(b) is correct
Sol.
918. Ans.(d) is correct
1 (54 )2 1
√144 × √324 ÷ 4 ( ÷ 24) = Sol. 240 × 300 × 240 = ? + 240
3 ?
(54)2 ? = 300 – 240
12 × 18 ÷ (4 ÷ 72) =
? ? = 60
54 × 54 × 4 3 919. Ans.(b) is correct
?= = = 0.75
12 × 18 × 72 4 5
911. Ans.(e) is correct Sol. 4 × 92– 16+? = 101
99+? = 101
Sol. ?=2
4 2 3
?3 920. Ans.(a) is correct
3 ÷ 36 × 24 =
2 1
34 Sol. 34 × 2 × 31 × 31 = ? −30.
× 243 × 2 =?3
362 ? = 68 + 30
?3 = 1728
? = 12 ? = 98
921. Ans.(c) is correct
912. Ans.(b) is correct Sol. 101 + 154 + 26 = 281
Sol. 922. Ans.(b) is correct
(63)? = 72.3+9.4 × 633.4 × 911.7 1 1 1
= (63)3.4+11.7 Sol. 15.5 × 28 × 112 + 1230 × 240 = ? × 5
? = 15.1 31 41 1
+ =?×
913. Ans.(d) is correct 8 8 5
Sol. 72
?= ×5
(?)² = 94,249 – 272 – 26,896 8
(?)² = 67,081 ? = 45
? = ±259 923. Ans.(d) is correct
914. Ans.(a) is correct 13
Sol. 9 × 17 + × 378 – 241 = ?
Sol. 9

9 × ? = 207.27 ? = 153 + 546 – 241


⇒ ? = 23.03 ? = 458
915. Ans.(b) is correct 924. Ans.(c) is correct
Sol. 1 1 1
(?)³ = 180 + 156 – 120 = 216 Sol. 8 × 928– 7 × 980 + 144 × 4 = ?
⇒?=6 116 – 140 + 36 = ?
916. Ans.(a) is correct
Sol. ? = 12

For Free Study Material & Quizzes visit : www.mathsbyarunsir.com


For any queries or suggestions email us @ mathsbyarunsir003@gmail.com or whatsapp @ 8881331466
925. Ans.(b) is correct = 14
Sol. √18 × 8 + 37.5% × 216−? =√1444 − √529 933. Ans.(c) is correct
Sol.
3
√144 + 8 × 216− ? = 38 – 23 55
?× 13 + × 460 = ?× 21 − 227
144 + 81 – ? = (15)2 = 225 100
? = 225 – 225 = 0 ⇒ 8 × ? = 253 + 227
926. Ans.(d) is correct ⇒ 8 × ? = 480
3 5 2 ⇒ ? = 60
Sol. ? = 5 𝑜𝑓 9 𝑜𝑓 7 𝑜𝑓 9450
934. Ans.(b) is correct
3 5 2
= × × × 9450 = 900 Sol.
5 9 7
927. Ans.(b) is correct 17 90 69 52
?= × + ×
Sol. 5 17 13 23
? = 41 × 4 + 51 × 7 – 4 × 89 = 18 + 12
= 521 – 356 = 30
935. Ans.(a) is correct
= 165
928. Ans.(b) is correct Sol.
Sol. ? × 3 + 198 = ? × 9 – 102
300
? = 209 × 5 + 128 – 768 ⇒? =
6
= 405 ⇒ ? = 50
929. Ans.(e) is correct 936. Ans.(c) is correct
Sol. Sol.
56 1 (315+105)
?= × 250 + × 963 ?= ×7
100 3 3

? = 140 +321= 461 ? = 980


930. Ans.(d) is correct 937. Ans.(a) is correct
Sol. Sol.
2 3 5 ?² = 395 + 791 – 990
?= × × × 1920
5 4 8 ?² = 196
= 360 ⇒ ? =± 14
931. Ans.(b) is correct 938. Ans.(c) is correct
Sol. Sol.
26 70 48 33
?= × + × √?− 611 + 550 = 19
7 13 11 16
= 20 + 9 ⇒√? = 80
= 29 ⇒ ? = 6400
932. Ans.(a) is correct 939. Ans.(b) is correct
Sol. Sol.
? = 3735 + 4565 + 6245 − 6432
72 200 111 400 ? = 8113
? =√ × + × 940. Ans.(c) is correct
100 3 100 3 Sol.
= √196
For Free Study Material & Quizzes visit : www.mathsbyarunsir.com
For any queries or suggestions email us @ mathsbyarunsir003@gmail.com or whatsapp @ 8881331466
? 6 21 ⇒?=0
× × × 216 = 36
3 7 36
?=1
941. Ans.(d) is correct 950. Ans.(b) is correct
Sol. Sol. ? = 12 + 28 + 36 – 8
? = 144 + 111 – 29 – 27
= 76 – 8
= 199
942. Ans.(e) is correct = 68

Sol. 951. Ans.(d) is correct


? = 27 + 87 – 48 + 39 = 105 𝟏 𝟑 𝟒 𝟑
Sol. (𝟏𝟒 + 𝟏𝟔 + 𝟏𝟒 + 𝟏𝟐) + (𝟏𝟏 + 𝟏𝟏 + 𝟏𝟐𝟏 + 𝟏𝟏) = ?
𝟏𝟏 + 𝟑𝟑 + 𝟒 + 𝟑𝟑
943. Ans.(b) is correct ? = 𝟓𝟔 + ( )
𝟏𝟐𝟏
Sol.
𝟖𝟏
44 30 = 𝟓𝟔 +
?= × 250 + × 270 + 169 𝟏𝟐𝟏
100 100
= 110 + 81 + 169 𝟖𝟏
= 𝟓𝟔
= 360 𝟏𝟐𝟏
944. Ans.(a) is correct
9–27+18 0
? = 1444 = 1444 = 0 952. Ans.(e) is correct
√ √
945. Ans.(d) is correct Sol. 𝟒𝟗. 𝟓 + 𝟗𝟖𝟕 − 𝟒𝟖 =?
2 108 21 27
? = 31 + 3 × × 36 × 42 ? = 𝟗𝟖𝟖. 𝟓
9
= 31 + 3
= 34 953. Ans.(a)

Sol.
946. Ans.(c) is correct 5
Sol. ? = 29 + 170 – 115 = 84 ⇒ 35% 𝑜𝑓 180 + 182 = (27)3 +?2
⇒ 63 + 324 = 243 +?2
947. Ans.(d) is correct ⇒ 387 − 243 =?2
𝟒𝟎 𝟒𝟒 ⇒ ?2 = 144
Sol. ?² = 𝟏𝟎𝟎 × 𝟒𝟐𝟎 + 𝟏𝟎𝟎 × 𝟐𝟎𝟎
⇒ ? = 12
= 168 + 88
= 256
954. Ans.(a)
⇒ ? = ± 16 Sol.
323
948. Ans.(b) is correct × 441 − 15 × 21 =?
357
𝟐𝟎
Sol. 𝟏𝟎𝟎 ×? = 𝟏𝟎𝟗𝟖 19 × 17
⇒ × 441 − 15 × 21 =?
⇒ ? = 5490 17 × 21
⇒ ? = 19 × 21 − 15 × 21 = 4 × 21 = 84

949. Ans.(a) is correct


955. Ans.(a)
𝟑𝟒(?+𝟐) 𝟑𝟑 ×𝟑𝟐 ×𝟑𝟑
Sol. 𝟏𝟎𝟒(?+𝟐) = 𝟏𝟎𝟑 ×𝟏𝟎𝟐 ×𝟏𝟎𝟑 Sol.
𝟑𝟖 ? = √5125 – 289 – 75
(𝟎. 𝟑)𝟒(?+𝟐) = = (𝟎. 𝟑)𝟖
𝟏𝟎𝟖 = √4761 = 69
⇒ 4(? + 2) = 8
For Free Study Material & Quizzes visit : www.mathsbyarunsir.com
For any queries or suggestions email us @ mathsbyarunsir003@gmail.com or whatsapp @ 8881331466
961. Ans.(d)
Sol.
956. Ans.(e) 23(24 + 47– 54) = 𝑥
𝑥 = 23 × 17
Sol.
𝑥 = 391
2
?3 = 64% 𝑜𝑓 150 + 7 × 3 − 92
2
962. Ans.(c)
?3 = 96 + 21 − 81 Sol.
2 6
?3 = 36 × 650 + 320 + 51 = 𝑥
3
5
⇒ ? = 362 = 216 780 + 320 + 51 = 𝑥
𝑥 = 1151

957. Ans.(c) 963. Ans.(c)


Sol. Sol.
2 2
?× 4 − 40 = 14 − 36 × 44 4 2 8 234
2 7 −3 +4 =
?× 4 = 196 + 40 − (40 − 4) × (40 + 4) 5 3 15 ?
?× 4 = 196 + 402 − 402 + 42 = 212 12 − 10 + 8 234
8+ =
212 15 ?
?= = 53 2 234
4 8+ =
3 ?
958. Ans.(b) 26 234
=
3 ?
Sol.
45 ⇒ ? = 27
× 80 + √841 + 𝑥 2 = 2121 ÷ 21
100
36 + 29 + x² = 101 964. Ans(d)
𝑥 2 = 36 Sol.
𝑥=6
96 ?
× 1050 − × 500 = 998
959. Ans.(c) 100 100
?
Sol. × 500 = 1008 − 998
100
36 + 3𝑥 10×100
+ 1 = 52 ?=
23 500
36 + 3𝑥 + 23 = 52 × 23 ?=2
3𝑥 + 59 = 1196
3𝑥 = 1196 – 59
3𝑥 = 1137 965. Ans.(d)
𝑥 = 379 Sol.
1350
960. Ans.(c) 𝑥× = √5929 + √8281
112.5
Sol.
x ×12 = 77 + 91 = 168
343 175
+ × 350 = 𝑥 2 ⇒ x = 14
2 100
𝑥 2 = 171.5 + 612.5
𝑥 2 = 784 966. Ans.(b)
𝑥 = 28
Sol.

For Free Study Material & Quizzes visit : www.mathsbyarunsir.com


For any queries or suggestions email us @ mathsbyarunsir003@gmail.com or whatsapp @ 8881331466
18750
= 36 × 11 + 59 × 6 971. Ans. e
√𝑥 𝟑
√𝟏𝟐𝟐𝟓 ÷ √𝟑𝟒𝟑 × 45% of 760 = ?
18750
⇒ = 396 + 354 35 ÷ 7 × 342 = ?
√𝑥 ? = 1710
18750
⇒ √𝑥 = = 25 972. Ans. d
750
x = 625 𝟏𝟕𝟓% 𝒐𝒇 𝟒𝟔𝟎 + 𝟏𝟏𝟎% 𝒐𝒇 𝟏𝟕𝟎 + 𝟐? = 𝟏𝟎𝟎𝟎
805 + 187 + 𝟐? = 1000
𝟐? = 8 = 23
967. Ans.(e)
?=3
Sol.
729 216 × 72 973. Ans. e
3𝑥 = × = 81 𝟏 𝟑 𝟏 𝟑 𝟏𝟖 𝟑𝟓
243 576
𝟑 +𝟐 +𝟕 −𝟓 − =
3 𝑥 = 34 ⇒ 𝑥 = 4 𝟕 𝟓 𝟓 𝟕 𝟑𝟓 ?
𝟏 𝟑 𝟏 𝟑 𝟏𝟖
(𝟑 + 𝟐 + 𝟕 − 𝟓 − 𝟏) + ( + + − − ) = 35/?
𝟕 𝟓 𝟓 𝟕 𝟑𝟓
968. Ans. b 𝟑𝟓
𝟔=
?
Q I: 𝟑𝟓
5428
?=
x = 1357 = 4 𝟔

Q II: 974. Ans. e


𝟏 𝟑 𝟐
800 × 𝑦 750 × 22 𝟓𝟐 ÷ 𝒐𝒇 𝟖 + × 𝟏 = ?
= 293 − 𝟐 𝟓 𝟑
100 100 ? = 𝟏𝟑 + 𝟏
⇒ 8 × 𝑦 = 293 − 165 = 128
? = 14
128
⇒ 𝑦= = 16
8 975. Ans. d
? % of 400 + 310% of 60 = 500% of 82
969. Ans. a 4x + 186 = 410
x = 56
Q I:
x = 9824 – 8074 = 1750 976. Ans. d
Q II: 𝟓
759 – 54 × 𝟗 = ? ³
y = 618 + 31 – 29 = 620 729 = ?𝟑
93 = ? ³
970. Ans. a ?=9
Q II: 977. Ans. b
⇒ 16 × 2.4 × 𝑥 = 288 × 100 156 + 376 − ? = 13% of 3000
288 × 100 532 – 390 = ?
⇒ 𝑥= = 750 ? = 142
16 × 2.4
Q I:
978. Ans. e
800 × 𝑦 750 × 22 115% of 4880 – 85% of 1640 = 740% of ?
= 293 −
100 100 5612 – 1394 = 7.4x
⇒ 8 × 𝑦 = 293 − 165 = 128 ? = 570
128
⇒ 𝑦= = 16
8 979. Ans. c

For Free Study Material & Quizzes visit : www.mathsbyarunsir.com


For any queries or suggestions email us @ mathsbyarunsir003@gmail.com or whatsapp @ 8881331466
25 12 85 54 [(32/100) * 250 ] - [(45/100) * 120] = 10 - (?/3)
× +?= × 80 - 54 = 10 - (?/3)
3 5 6 17
? = 45 – 20 = 25 26 = 10 - (?/3)
10 - 26 = ?/3
980. Ans. a
-16 = ?/3
(21 – 8) ? (13 - 10)
13 ? 3 ? = -48
So, ? = > 989. Ans. B
132 - 24 + 789 ÷ 263 = 46 * ? ÷ 23
981. Ans. a
(54 ÷ 3 of 6 + 9) ? [13 − (12 − 6 ÷ 3) ] => 2 * ? = 169 - 24 + 3
12 ? 3 => 2 * ? = 148
=> ? = 74
982. Ans. b
8[1 - {3 + (5 - 6 + 7)}] ? 0.8{0.75 ÷ (1.35 - 0.6)} - 0.8 990. Ans. A
8(-8) ? 0.8 – 0.8 80% of 45 ÷ 12% of 50 + (?) = 18
-64 ? 0
=> ((80/100) * 45) ÷ ((12/100) * 50) + (?) = 18
983. Ans. b => 36 ÷ 6 + (?) = 18
(37 - 6 × 4 + 32 ÷ 8) ? (64 - 48 ÷ 6 × 4 + 8) => 6 + (?) = 18
(41 – 24) ? (32 + 8)
or, (?) = 12
17 ? 40
991. Ans. E
984. Ans. b
(76 + 24 x 5 - 11 x 8) ÷ 4 + √144 = ?
(12 + 4 - 8 ÷ 2 × 3) ? (7 - 5 + 14 ÷ 2 + 6)
(12 + 4 – 12) ? (20 – 5) => (76 + 120 - 88) ÷ 4 + 12 = ?
4 ? 15 => 108 ÷ 4 + 12 = ?
=> 27 + 12 = ?
985. Ans. b
(16 + 8 ÷ 4 − 2 × 3) ? (15 + 5 ÷ 5 × 2) or, ? = 39
(16 + 2 – 6) ? (15 + 2) 992. Ans. B
12 ? 17
(3214 + 886 - 1073 - 1731)1/4 + (?) = √361
=> (4100 - 2804)1/4 + (?) = 19
986. Ans. a
=> (1296) 1/4 + (?) = 19
𝟒
(2450 ÷ 7 + 112 × 2.5) ? 𝟕of 441 ÷ 18 + 14 => (36 * 36) 1/4 + (?) = 19
630 ? 266 => (6 * 6 * 6 * 6) 1/4 + (?) = 19
=> 6 + (?) = 19
or, (?) = 13
987. Ans. A
993. Ans. D
126 ÷ 14 * 9√2 - √72= ?√2
√1225 + √5776 - √3249 = √? + √1024
9 * 9√2 - √72= ?√2
=> 35 + 76 - 57 = √? + 32
81√2 - 6√2 =? √2
=> √? = 54 - 32
75√2 =? √2 => ? = 222
75 = ? => ? = 484
988. Ans. E 994. Ans. D
32% of 250 − 45% of 120 = 10 - (?/3) 56% of 3150 - 130% of 630 = 900% of ?

For Free Study Material & Quizzes visit : www.mathsbyarunsir.com


For any queries or suggestions email us @ mathsbyarunsir003@gmail.com or whatsapp @ 8881331466
=> 1764 - 819 = 9 * ? 999. Ans. D
=> ? = 945/9 11 x ? x 23 = 572 + 2570
=> ? = 105 253 x ? = 3249 + 2570 = 5819
995. Ans. E ? = 5819/253 = 23
78 * 34 ÷ 221 - 121 * 209 ÷ 1331 = ? 1000. Ans. C
=> ? = 78 * 2/13 - 121 * 19/121 ?% of 480 = 1131 - 90% of 670
=> ? = 12 - 19 = -7 ?% of 480 = 1131 - 90 x 670/100 = 1131 - 603 = 528
996. Ans. D ? = 528 x 100/480 = 110
12.5% of 64 + 80% of 25 - ?% of 180 = 19 1001. Ans. B
=> 8 + 20 -19 = ?/100 * 180 ? = 32(0.16 + 0.04) = 320.20 = 321/5
=> ? = 9 * 100/180 Since, 321/5 = 2, ? = 21 = 2
=> ? = 5 1002. Ans. D
997. Ans. D (?)2 = 232 + 292 - 1
425 - 68% of 350 + ? = 16 2 - √2809 = 529 + 841 - 1
425 - (68/100) * 350 + ? = 256 - 53 = 1369
425 - 238 + ? = 203 ? = 37
? = 203 - 187
? = 16 1003. Ans. E
998. Ans. E
?2 = (4 x 80 x ∛125)/(√256)
√(945 ÷ 45 + 12 x 5) - ? = 156 ÷ 39 = 4 x 80 x 5/16
√(21 + 60) - ? = 4
√81 - ? = 4
?=9-4
?=5
= 100 = 102
So, ? = 10

For Free Study Material & Quizzes visit : www.mathsbyarunsir.com


For any queries or suggestions email us @ mathsbyarunsir003@gmail.com or whatsapp @ 8881331466
For Free Study Material & Quizzes visit : www.mathsbyarunsir.com
For any queries or suggestions email us @ mathsbyarunsir003@gmail.com or whatsapp @ 8881331466

You might also like